UWorld Review 1

Pataasin ang iyong marka sa homework at exams ngayon gamit ang Quizwiz!

A 35-year-old woman comes to the emergency department with nausea, vomiting, and fever. Her symptoms began 24 hours ago, and she has been unable to eat or drink anything since. She has a 3-year-old daughter who had similar symptoms 2 days earlier but is now fine. Laboratory studies show a blood glucose level of 82 mg/dL despite her 24-hour fast. Maintenance of this patient's blood glucose levels is facilitated by hepatic conversion of pyruvate into glucose. Which of the following substances directly stimulates the first enzyme involved in this process?

Acetyl-CoA - Acetyl-CoA acts as a feedback mechanism, inhibiting pyruvate dehydrogenase and activating pyruvate carboxylase to activate gluconeogenesis for the formation of glucose.

A 21-day-old boy is brought to the office by his mother because of a palpable bulge in the child's neck. He continues to feed well but appears comfortable only when held with his body sideways under the breast. He is at the 50th percentile for height, weight, and head circumference. The child favors looking toward the right and cries when his head is turned to the left. There is a firm mass on the left side of his neck that does not move when the child swallows. The remainder of the examination is unremarkable. Which of the following conditions was most likely present prenatally?

Intrauterine malposition - This baby likely has congenital torticollis, which can happen from birth trauma or malposition in utero, leading to a sternocleidomastoid muscle.

A 4-week-old infant is evaluated for frequent episodes of hypoglycemia. The patient is found to have elevated insulin levels during periods of fasting. Several members of his family have a similar condition. Genetic testing reveals a mutation involving potassium channels expressed in pancreatic beta cells. The mutated channels have a higher sensitivity to the substance that normally modulates their activity, causing increased beta cell secretory activity. Which of the following substances normally binds to and regulates the ion channels that are responsible for this patient's hypoglycemia?

ATP - It binds ATP-sensitive potassium channel, causing membrane depolarization and subsequent insulin release

A 56-year-old man comes to the emergency department due to persistent left wrist pain. While walking his dog 5 hours ago, he fell forward onto the ground and landed on his outstretched hand. Examination shows mild left wrist swelling with preserved range of motion. There is point tenderness over the dorsolateral aspect of the wrist between the tendons of the extensor pollicis longus and extensor pollicis brevis. An x-ray is shown. This patient is at greatest risk for developing which of the following complications?

Avascular necrosis - A scaphoid fracture is the most common carpal bone fracture and frequently results from a fall on an outstretched hand. It should be suspected in any patient with persistent wrist pain and tenderness in the anatomic snuffbox. Fractures can cause disruption to the retrograde flow of the dorsal scaphoid branch of the radial artery, leading to avascular necrosis of the scaphoid bone.

Special electrodes are used to detect the change in membrane potential of a specific type of cardiac cell. These changes are recorded in a graph. The deflection indicated by the arrow is most likely caused by movement of which of the following ions?

Calcium - Pacemaker depolarization is due to a large increase in calcium influx.

A 14-year-old girl is brought to the office due to intermittent shortness of breath over the past 4 months. She is unable to link the episodes to specific events or exposures. The patient has no prior medical conditions and takes no medications. She does not smoke and has no second-hand tobacco exposure. Her family has a cat that remains indoors. There is no significant family history. Blood pressure is 110/65 mm Hg, pulse is 80/min, and respirations are 16/min. On physical examination, heart sounds are normal and the lungs are clear to auscultation. Lung function tests reveal a FEV1/FVC ratio of 83%. Which of the following would be most helpful in excluding a diagnosis of asthma in this patient?

Negative methacholine challenge - A provocative stimulus (typically aerosolized methacholine, a cholinergic muscarinic agonist) is administered at increasing concentrations to induce bronchoconstriction. Patients with asthma are hyperresponsive to this stimulus and experience a decline in FEV1 at lower doses than nonasthmatics. However, the test is nonspecific and positive results may also be seen with chronic obstructive pulmonary disease, cystic fibrosis, and allergic rhinitis.

A 52-year-old man is evaluated for low-grade fever, malaise, abdominal pain, and diarrhea. The patient underwent a cadaveric renal transplant 6 months ago to treat renal failure from underlying polycystic kidney disease. He takes maintenance immunosuppressive therapy. Physical examination shows mild tenderness in the lower abdominal quadrants with no rebound tenderness. Serum immunosuppressant levels are within therapeutic range, and renal function studies are normal. Colonoscopy reveals mucosal erythema and ulcerations. Biopsy shows large cells with inclusion bodies, and a polymerase chain reaction-based test detects viral DNA. First-line therapy for the infection is initiated. This patient is at greatest risk of which of the following from the treatment?

Neutropenia - Patients on immunosuppressive therapy after organ transplant are at risk for cytomegalovirus reactivation. This patient has the classic manifestations of CMV colitis. Biopsy will usually show large cells with intranuclear and intracytoplasmic inclusions ("owl eye"). This is usually treated with ganciclovir which interferes with viral DNA replication, but this drug also impedes human DNA polymerase to a lesser degree, so you can see effects like neutropenia, anemia, and thrombocytopenia.

A 47-year-old man comes to the office due to persistent fever, night sweats, and fatigue. Thorough evaluation yields a diagnosis of chronic myeloid leukemia. While undergoing treatment for his malignancy, the patient experiences headaches, scant nasal discharge, and left eye symptoms. Physical examination reveals tenderness over the paranasal sinuses and left-sided orbital swelling and cellulitis. Mild proptosis and ptosis of the left eye are also present. Which of the following is the most likely cause of this patient's condition?

Aspergillus fumigatus - on sinus mucosa biopsy, there is a presence of septate hyphae and 45* (acute) angle branching. Aspergillus can cause invasive aspergillosis in immunosuppressed patients (treat with voriconazole), aspergillomas which are fungus balls that grow in old lung cavities (often surgically removed), and allergic bronchopulmonary aspergillosis which occurs in asthma with IgE increased in the serum and titers of antibodies are characteristic (treat with corticosteroids)

A 10-year-old boy is brought to the office for poor grades and behavioral issues. He is very smart but is struggling at school and getting bad grades due to distractability, careless mistakes, and losing homework. He often gets in trouble for disrupting the class. At home, he cannot focus on chores or listen well to his parents. His family history is significant for substance use disorder in the patient's 16-year-old brother and depression in his mother. They would prefer to try a non-stimulant option for treatment. Which of the following is the most appropriate pharmacotherapy for the patient?

Atomoxetine - This patient is consistent with the presentation of ADHD. Stimulants are usually the first-line treatment for ADHD, but have a large side effect profile and have the potential for abuse. Atomoxetine is a first-line nonstimulant that lacks abuse potential. It is a norepinephrine reuptake inhibitor. Alpha-2 adrenergic agonists can also be considered for a nonstimulant option.

An 18-month-old boy is evaluated for recurrent infections. He has been hospitalized 3 times with pneumonia since age 3 months. He has also had multiple skin infections requiring treatment with antibacterial and antifungal agents. During his last episode of pneumonia, he developed a large pleural effusion, which was drained. The pleural fluid revealed numerous neutrophils containing a larger number of intact gram-positive cocci. The microorganisms responsible for this patient's recurrent infections most likely produce which of the following virulence factors?

Catalase - The infant likely has a primary immunodeficiency disorder. The presence of a large number of intact bacteria inside the neutrophils suggests chronic granulomatous disease, an x-linked recessive immunodeficiency marked by impaired NADPH oxidase activity. Patients with CGD are most susceptible to catalase-positive organisms as they require higher levels of free radicals (I.e. greater NADPH oxidase function) for effective killing.

Researchers are interested in the association between colorectal carcinoma and nonsteroidal anti-inflammatory drug use. They first interview a group of patients with biopsy-proven colorectal carcinoma and then interview a group consisting of the patients' neighbors who are of similar age and race. The analysis is based on comparing the results of pairs of individuals (one from each of the 2 groups) who have similar characteristics. This design technique best helps address which of the following potential problems with this study?

Confounding - Matching is used in case-control studies to control confounding. They match variables that could be cofounders. In this case, neighbors are matched to account for similar variables like age and race, to avoid cofounding.

A 1-week-old newborn is seen for abnormal thyroid function tests that were performed as part of routine screening. Serum thyroxine (T4) was low and serum TSH was normal. The patient was born by spontaneous vaginal delivery at 40 weeks of gestation to a 28-year-old primigravida. During the pregnancy, the mother took acetaminophen intermittently for low back pain. Examination of the newborn shows no abnormal findings. Repeat thyroid function test results are as follows: Free T41.2 μg/dL (normal: 0.8-1.95) Total T44 μg/dLTSH3 μU/mL Which of the following is the most likely cause of the patient's abnormal thyroid function tests?

Deficiency of thyroxine-binding globulin - Congenital TBG deficiency is benign and x-linked. It will usually be detected on a newborn screen. Total T4 is low as it is not bound, but the TSH and free t4 remain normal due to physiologic feedback. The child is euthyroid and no treatment is needed

A 23-year-old graduate student comes to the emergency department after being raped by a man she met at a bar 5 days ago. The night of the incident, the patient recalls a man introducing himself and handing her a drink. Shortly afterward, she left with him and walked to her apartment, but she is unable to remember the remainder of the night. The following morning, the patient awoke to an empty apartment, and her clothes were strewn across the floor. She felt panicky and decided to take a shower to calm down. The patient mentions that she decided to come in for evaluation after talking to campus crisis services earlier today. She states she has not reported the incident to the authorities due to her hazy memory. Which of the following is the most appropriate initial statement by the physician?

"It's important that you feel safe here; please let me know if you need to stop or take a break at any point" - The statement should be supportive, nonjudgemental and patient centered, avoiding retraumatizing the patient however possible.

A 21-year-old woman comes to the office due to recurrent episodes of self-limited, colicky abdominal pain. She also had an episode of facial swelling that resolved spontaneously. The patient has no other significant past medical history and takes no medications. Examination is unremarkable. Evaluation shows that her complement protein C1, even when not attached to an antigen-antibody complex, is excessively cleaving C2 and C4. Which of the following is most likely increased in this patient?

Bradykinin - This patient likely has angioedema due to C1 inhibitor deficiency. C1INH prevents C1-mediated cleavage of C2 and C4, thereby limiting activation of the complement cascade. It also blocks kallikrein-induced conversion of kininogen to bradykinin, a potent vasodilator that also causes increased vascular permeability.

A 44-year-old man hospitalized for acute cholecystitis is being evaluated for anxiety and agitation. He underwent an open cholecystectomy without any operative complications. Two days after admission, the patient experiences anxiety and tremulousness, and becomes irritable, agitated, and verbally abusive. He has no other medical problems. He does not use tobacco or illicit drugs but drinks 6-8 beers daily for the past few years. He has no known drug allergies and family history is insignificant. BP is 160/90mmHg and pulse is 110/min. Exam shows hand tremors bilaterally. Which of the following is the most appropriate pharmacotherapy for this patient?

Chlordiazepoxide - This patient is likely going through alcohol withdrawal and the first line therapy is benzodiazepines (ex. lorazepam, diazepam, chlordiazepoxide)

A 16-year-old girl comes to the office due to malodorous vaginal discharge. The patient arrives with her mother, who stays in the examination room for the evaluation. The patient has had increased vaginal discharge for the past 2 days but no abnormal vaginal bleeding or abdominal or pelvic pain. She is sexually active with a new partner and uses a progestin-releasing subdermal implant for contraception. Her last menstrual period was 2 weeks ago. She has no chronic medical conditions and takes no medications. Vital signs are normal. The patient appears anxious. Abdominal examination shows no tenderness or palpable masses. When the pelvic examination is attempted, the patient says that she is "embarrassed and anxious" and refuses the examination. Which of the following is the most appropriate course of action?

Do not perform the examination and ask the patient if she will perform self-collection for testing - Minors have the right to refuse a pelvic examination, which can be especially anxiety provoking in minors. In this non-emergent condition, there is no strong need to perform the exam when the patient is uncomfortable, but asking the patient to perform self-collection can allow for diagnosis and appropriate treatment.

A 22-year-old man comes to the office due to progressive skin lesions. The patient noticed a nonpainful, nonpruritic skin bump on his right knee 3 months ago, which progressively increased in size and number. He has no prior health issues and takes no medications. The patient is outdoors frequently for work and recreational activities. He is sexually active. Physical examination findings are shown in the exhibit. Which of the following microbial organisms is most likely responsible for this patient's current condition?

Human papillomavirus - This patient presents with cutaneous warts, caused by HPV and direct contact. It is caused by types 2 and 4 most commonly.

A 65-year-old woman comes to the office due to progressively worsening weakness of the legs for 5 months. Examination shows increased muscle tone, brisk deep tendon reflexes, and decreased muscle strength in the lower extremities. The Babinski sign is positive in both legs. Upper extremities are normal. MRI of the spine reveals a 2-cm, round lesion with homogeneous, intense contrast enhancement compressing the spinal cord at the T12 level; the lesion is extramedullary but is surrounded by cerebrospinal fluid, as shown in the exhibit. Which of the following is the most likely cell of origin of the lesion?

Meningothelial cells - This patient has spinal cord compression due to a contrast-enhancing lesion located in the intradural extramedullary space. This is likely a meningioma from meningothelial cells.

A 45-year-old woman comes to the office with a palpable thyroid node. Medical history reveals that she was diagnosed with medulloblastoma during childhood and treated with chemotherapy and external beam radiation. Fine-needle aspiration of the thyroid shows no malignant cells. As the test results are explained to the patient, she asks, "What are the chances that I really do not have thyroid cancer?" Which of the following diagnostic test parameters would be most useful for answering this patient's question?

Negative predictive value - Negative predictive value (NPV) is defined as the probability of not having a disease when the test result is negative. NPV is calculated as the proportion of true negatives divided by the total number of negative tests (true and false negatives); therefore, it varies with the prevalence of disease in the target population.

A 30-year-old man is evaluated for 6 months of persistent pain in his right buttock and posterior thigh. He is a construction worker and has difficulty lifting heavy objects or climbing stairs. He has no history of back injury and no associated back pain. Past medical history is unremarkable. The patient does not use tobacco and drinks only moderate quantities of alcohol. On examination, the patient develops pain with forced adduction of the flexed thigh and internal rotation of the extended thigh against resistance. Further evaluation reveals possible entrapment of the sciatic nerve in the greater sciatic foramen. Which of the following structures passes through the foramen and occupies most of its volume?

Piriformis muscle - The piriformis muscle occupies most of the space in the greater sciatic foramen, which is made up of inferiorly by the sacrospinous ligament. This patient likely has piriformis syndrome.

A 70-year old man comes to the office due to persistent aspiration. Two weeks ago, he had a stroke that impacted his speech, voice, and swallowing. Modified barium swallow study reveals aspiration of liquids without nasal regurgitation. Aspiration is ameliorated when the patient flexes his neck to put his chin to his chest during swallowing. This maneuver is most likely augmenting which of the following airway-protective mechanisms?

Superior displacement of the larynx - in normal swallowing the following occurs: displacement of the larynx superiorly, tilting of the epiglottis, closing of the glottis by adduction of the vocal folds

A 34-year-old woman comes to the hospital with a 4-day history of abdominal cramps, nausea, and watery diarrhea. Today she developed dizziness on standing. Her child has had similar symptoms recently. The patient has no prior medical conditions and takes no medications on a regular basis. Blood pressure is 124/82 mm Hg while supine and 100/70 on standing; pulse is 98/min. Examination shows dry mucous membranes. The abdomen is soft and nontender. Laboratory results are as follows: Serum chemistry Sodium 144 mEq/L Blood urea nitrogen 50 mg/dL Creatinine 1.8 mg/dL Urinalysis Protein negative Red blood cells 0/hpf White blood cells 0-1/hpf Microscopy few hyaline casts Urine sodium 8 mEq/L Which of the following changes are most likely to be seen in this patient?

Vasopressin: increased Norepinephrine: increased Angiotensin II: increased Endothelin 1: Increased This patient has hypovolemia. Activation of the renin-angiotensin-aldosterone system, vasopressin, and increased sympathetics lead to these findings.

A 45-year-old woman participates in a cardiovascular health study. She undergoes 24-hour ECG monitoring with a wearable device and a pattern is found on her ECG showing a PVC at point Y, with a normal beat (X) before it and beat Z after. Which of the following represents the relative left ventricular end-diastolic volumes just prior to the heart beats?

`Z>X>Y, When a PVC occurs, it interrupts diastolic filling of the left ventricle, resulting in decreased end-diastolic volume (EDV) compared to the previous normal beat (beat X > Y). The sinus pause that occurs after a PVC allows for longer-than-normal filling time before the next ventricular contraction (ie, prolonged ventricular diastole), which creates increased EDV at the time of the post-PVC beat (beat Z > X). The greater-than-normal EDV causes a large-stroke volume ventricular ejection, which may be responsible for the palpitations that some patients experience with PVCs.

A 54-year-old woman comes to the office for a yearly preventive examination. She has no specific concerns. Blood pressure is 144/95 mm Hg and pulse is 86/min. BMI is 50 kg/m2. Examination shows severe obesity but is otherwise unremarkable. Fasting glucose is 128 mg/dL. When the subject of her weight is mentioned, the patient says, "In my family, we just like to eat. My mother is heavy, my father is heavy, and my kids are heavy. There's no point in trying to lose weight now." Which of the following is the most appropriate response to this patient's statement?

"Tell me how you think your weight might relate to your blood pressure and glucose" - This patient is in the pre-contemplation change and you need to get an idea of the current understanding of their condition and how it can affect their future health.

A study is conducted to assess the effectiveness of a new medication X added to standard antihypertensive therapy compared to standard antihypertensive therapy alone for preventing the development of proteinuria in patients recently diagnosed with essential hypertension. Five years after diagnosis, 97 of 100 patients on the medication X + standard antihypertensive therapy regimen have not developed proteinuria, and 90 of 100 patients on a standard antihypertensive therapy regimen have not developed proteinuria. According to these results, which of the following represents the approximate number of patients with essential hypertension who need to be treated with medication X to prevent an additional patient from developing proteinuria within 5 years?

15 - The number needed to treat is the number of patients who need to be treated with a specific drug to prevent an additional negative event. NNT is the inverse of the absolute risk reduction (NNT = 1/ARR). Absolute risk reduction is = to control rate - treatment rate. So in this case the control rate = 100-90/100 = 10/100 = 0.10 and the treatment rate = 100-97/100 = 3/100 = 0.03. So the absolute risk reduction = 0.10 - 0.03 = 0.07. The number needed to treat = 1/ARR = 1/0.07 = 14.3 ≈ 15

A 4-year-old boy is brought to the office due to progressively worsening cough for the past 2 days that is productive of yellow sputum. A year ago, he was found to have bilateral lower-lobe pulmonary infiltrates on chest x-ray and was diagnosed with pneumonia. The patient has since had 2 additional episodes of pneumonia, each requiring antibiotics for improvement of symptoms. His current chest x-ray reveals bilateral lower lobe infiltrates. Further testing shows a high chloride content in his sweat. Which of the following abnormalities is most likely to be seen in this patient?

Abnormal post-translational processing of a transmembrane protein - This patient likely has cystic fibrosis, an AR disease with sinopulmonary infections, pancreatic insufficiency, and malabsorption. There are several different mutations that affect the CF transmembrane conductance regulator (CFTR) gene. The most common is ^F508 which causes impaired post translational processing. (drug - lumacaftor)

A 43-year-old man comes to the emergency department due to 2 weeks of intermittent fevers, malaise, and headaches. He has a history of HIV and has not been taking his antiretroviral drugs recently. Five years ago, the patient spent a year in jail. Temperature is 38.2 C (100.8 F). Cerebrospinal fluid (CSF) analysis shows markedly elevated protein and low glucose, and CSF cultures grow Mycobacterium tuberculosis. Resistance testing shows the following: Rifampin sensitive Isoniazid sensitive Pyrazinamide sensitive Ethambutol sensitive Streptomycin resistant Moxifloxacin sensitive Which of the following best explains the resistance pattern seen in these bacteria?

Altered structure of bacterial ribosomal proteins - The patient has M tuberculosis meningitis, which requires combination therapy with isoniazid, rifampin, pyrazinamide, and either an injectable aminoglycoside or a fluoroquinolone. Streptomycin is one of the older drugs in the aminoglycoside family, so bacterial resistance to the antibiotic is widespread. It works by interfering with the 16s rRNA of the bacterial 30S ribosomal subunit, thereby preventing bacterial protein synthesis. Mutation in the genes that encode ribosomal binding sites for these drugs causes this resistance.

A 48-year-old man comes to the office due to several hours of severe right knee pain. The patient has a history of peptic ulcer disease and gastroesophageal reflux disease. His right knee is swollen, erythematous, and tender. Arthrocentesis is performed and synovial fluid analysis shows needle-shaped, negatively birefringent crystals with many neutrophils. The medication given to this patient selectively binds to an interleukin-1 inducible enzyme that is highly expressed by inflammatory cells and undetectable in the surrounding normal tissue. Which of the following is most likely the drug used in this patient's treatment?

Celecoxib - This drug is a COX-2 inhibitor, blocking its pro-inflammatory effects and minimizing the gastroduodenal affects of blocking COX-1.

A 2-year-old boy with no chronic medical conditions is brought to the emergency department after he was found unresponsive with vomitus in his mouth. The patient has had muscle aches, cough, rhinorrhea, diarrhea, vomiting, and fever for the past 3 days. His parents gave him aspirin twice daily for symptom control. Vital signs are within normal limits. On examination, the patient is somnolent but withdraws from pain. Pupils are equal in size and reactive to light. Cardiopulmonary examination shows no abnormalities. Hepatomegaly is present. Extremities are well perfused. Laboratory results are as follows: Aspartate aminotransferase (SGOT) 2,170 U/L Alanine aminotransferase (SGPT) 1,587 U/L Total bilirubin 1.7 mg/dL Direct bilirubin 1.2 mg/dL Serum ammonia 106 μmol/L (normal: 11-32) Over the next day, the patient's condition deteriorates and he dies despite supportive care. Examination of the brain would most likely reveal which of the following findings?

Diffuse astrocyte swelling - This patient has characteristic symptoms of Reye syndrome, which arises from the use of aspirin in children with a viral infection. The liver failure will show fat accumulation and there will be diffuse astrocyte swelling and cerebral edema from excess ammonia.

A 62-year-old woman undergoes hip replacement surgery. The patient has a history of advanced hip osteoarthritis that limits her daily activities. She has no other medical conditions and no known drug allergies. After appropriate preoperative evaluation, total hip arthroplasty is performed under general anesthesia. The intraoperative course is uncomplicated, and after recovery from anesthesia, patient-controlled intravenous morphine is started for pain control. Several minutes later, the patient reports generalized itching. Physical examination reveals hypotension, tachycardia, and mild bilateral wheezing but no rashes. Which of the following drug effects is most likely responsible for this patient's current condition?

Direct mast cell degranulation - Opioids (eg, morphine) can generate a pseudoallergic response by directly stimulating mast cell degranulation, releasing histamine and other vasoactive mediators. This nonimmunologic reaction can cause itching, urticarial rash, wheezing, hypotension, and tachycardia that closely mimic true IgE-mediated type 1 hypersensitivity (eg, anaphylaxis). However, true IgE-mediated allergic reaction is rare with opioids

In normal female development, non-fusion of the urethral folds forms the labia minora and the vestibule of the vagina. In males, non-fusion of the urethral folds would most likely result in which of the following?

Hypospadias - Incomplete fusion of these folds in a male would result in an abnormal opening of the urethra at a location proximal to the distal tip of the glans penis on the ventral surface.

A 24-year-old man comes to the office due to paroxysmal episodes of breathlessness and wheezing for the past 6 months. He does not recognize any triggers for these episodes and reports they are not associated with exercise or stress. The patient has no history of recent illness. He had eczema as a child. The patient has no other medical conditions and takes no medications. Vital signs are within normal limits. Lung examination shows good air movement and no wheezing. Sputum microscopy reveals many granule-containing cells and crystalloid masses. The sputum findings in this patient are a direct result of which of the following cytokines?

IL-5 - This patient likely has atopic asthma. It is an excessive TH2-mediated reaction to environmental aero-antigens. These cells secrete IL-5, a critical cytokine for eosiniophilic activation, recruitment, and prolonged survival in the bronchial mucosa.

A 64-year-old woman comes to the emergency department due to a 2-day history of worsening abdominal pain. She has a history of gallstones but has had no surgery due to high operative risk. Medical history includes chronic obstructive pulmonary disease, hypertension, and remote cervical cancer treated with surgery and chemotherapy. Abdominal CT scan shows multiple dilated small bowel loops with a decompressed distal ileum, and an abnormal bowel indicated by the arrow. Which of the following is the most likely cause of this patient's current condition?

Incarcerated hernia - This patient's CT is concerning for small bowel obstruction. The CT shows a small bowel loop extending outside the peritoneal cavity into the left inguinal area, indicative of an inguinal hernia that has become trapped.

An outbreak of hepatitis in New Delhi was characterized by high incidence of fulminant hepatitis in pregnant women, who experienced a mortality rate of 20%. The virus responsible for the outbreak:

Is an unenveloped RNA virus - Hepatitis E is an unenveloped, single-stranded RNA virus spread through the fecal-oral route. It has a high mortality rate in pregnant women.

A 23-year-old man comes to the office due to chest discomfort that usually occurs during exercise, such as jogging or climbing stairs. The symptoms go away 5 to 10 minutes after he stops. The patient has not had syncope but mentions some shortness of breath that accompanies the chest pain. Family history includes an uncle who died suddenly at age 35. Blood pressure is 122/70 mm Hg and pulse is 70/min and regular. The apical impulse is strong and sustained. While supine, the patient has a soft crescendo-decrescendo systolic murmur at the apex and left sternal border; it becomes quite pronounced when he stands. Which of the following medications should be avoided while treating this patient's condition?

Isosorbide dinitrate - This patient's presentation is most consistent with hypertrophic cardiomyopathy. The clinical manifestations of HCM result mostly from enlargement of the intraventricular septum that leads to left ventricular outflow tract obstruction. Medications that decreased LV blood volume should be avoided, including preload reducers (nitrates and diuretics), afterload reducers (hydralazine), and balanced vasodilators (dihydropyridine calcium channel blockers, ACE inhibitors)

How would you calculate negative predictive value?

NPV = true negative / (true negative + false negatives)

A 22-year-old woman comes to the office due to 2 weeks of profound fatigue, which has prevented her from attending college classes. The fatigue was preceded by fevers and sore throat, but these have now improved. The patient is an exchange student from Turkey and has no prior medical conditions. She does not use tobacco, alcohol, or illicit drugs. Vital signs are within normal limits. Oropharyngeal examination shows no abnormalities. There are several enlarged lymph nodes posterior to the sternocleidomastoid muscle bilaterally. Cardiopulmonary examination is unremarkable. The spleen is palpable 1 cm below the left costal margin. No skin rashes or other lymphadenopathy is present. Peripheral blood evaluation shows increased numbers of abnormal white blood cells as shown in the exhibit. The observed cytoplasmic granules of these cells are most likely to contain which of the following substances?

Perforin - The blood smear reveals a reactive atypical lymphocyte, which is large, scalloped, and has abundant cytoplasm. This is in response to infectious mononucleosis, along with cytotoxic T cells and natural killer cells. Reactive lymphocytes are effector cells that contain cytotoxic granules composed of perforin and granzymes.

A 4-year-old girl is brought to the clinic due to an inward deviated right eye. Examination shows a pale red reflex in the right eye relative to the left eye. Dilated funduscopic examination reveals a well-circumscribed white mass within the retina. Genetic analysis shows a germline mutation in the patient's cells leading to development of the mass. Which of the following is the most likely function of the protein affected by this patient's gene mutation?

Prevention of the G1/S cell cycle transition - This presentation is indicative of retinoblastoma, which is an intraocular tumor caused by inactivating mutations affecting the RB1 tumor suppressor gene, which regulates the G1/S checkpoint

A 5-year-old boy is brought to the emergency department due to accidental drug ingestion. His mother states the patient was playing by himself earlier today and 2 hours later she found him unresponsive in the bathroom. There was an empty bottle of hydrocodone-acetaminophen next to him, but she does not know how many pills were in it. The emergency medical team found the patient stuporous and with bradypnea. His mental status and respirations promptly improved after 1 dose of intravenous naloxone was administered, and he was transported to the hospital. On arrival, the patient is sleepy but arouses easily to voice and follows simple instructions. Vital signs, including respirations, are normal. One hour later, he has worsening lethargy, bradypnea, and miosis. Which of the following most likely accounts for this patient's current clinical deterioration?

Short half life of naloxone - Naloxone is a short-acting opioid antagonist with a short half life of less than an hour, so recurrent symptoms can occur.

A 37-year-old woman is evaluated for progressive shortness of breath and dry cough. The patient has seasonal allergies and takes no medications. There is no significant family history. On physical examination, the lungs are clear to auscultation. Chest imaging studies reveal pulmonary infiltrates and hilar adenopathy. The patient undergoes lung biopsy; the findings are shown below. Which of the following set of immune cells and cytokines is most likely responsible for the development of this patient's pathologic findings?

Th1, IL-2, interferon-γ - This patient's presentation suggests pulmonary sarcoidosis. The biopsy shows multiple non-caseating granulomas (central collection of tightly clustered epithelioid macrophages with abundant pink cytoplasm surrounded by a rim of mononuclear cells). Sarcoidosis is thought to result from a dysregulated cell-mediated immune response to an unidentified antigen. Cell-mediated immunity is stimulated by production of IL12 from activated antigen presenting cells, which stimulated CD4+ helper T cells to differentiate into the Th1 subtype, which secrete IL-2 and interferon-γ.

A 53-year-old man comes to the office due to double vision. The patient lives in a two-story house and has had difficulty walking down stairs because he sees duplicates of every step and doesn't "know which ones are real." He does not have significant problems walking up stairs. The patient is also frustrated because he has trouble reading certain things, such as the morning newspaper and work-related documents. He has had no eye trauma, headache, focal weakness, or numbness. The patient has a history of hypertension and type 2 diabetes mellitus. He exercises regularly and does not use tobacco, alcohol, or illicit drugs. A lesion affecting which of the following structures is most likely responsible for this patient's visual symptoms?

Trochlear Nerve - This patient likely have trochlear nerve (CN IV) palsy. The trochlear nerve innervates the superior oblique muscle, which causes the eye to intort and depress while adducted. This nerve is particularly susceptible to injury due to its small size and long course. These patients typically present with vertical diplopia which is most noticeable when a patient tries to look down and in (reading up close, walking down stairs)

A 46-year-old woman comes to the emergency department due to sudden-onset, severe occipital headache and nausea. The patient has a history of hypertension and cigarette smoking. Physical examination shows mild disorientation but no focal weakness, sensory loss, or cranial nerve abnormalities. Noncontrast CT scan of the head reveals diffuse hyperdensity in the subarachnoid space. CT angiography reveals a ruptured anterior communicating artery aneurysm. The patient undergoes endovascular coiling of the aneurysm. Four days later, there is sudden deterioration in her level of consciousness and new, right-sided hemiparesis is observed. Repeat CT scan shows no significant changes. Which of the following is the most likely cause of this patient's neurological deterioration?

Vasospasm - This patient had a subarachnoid hemorrhage from the rupture of a berry aneurysm. If a patient survives, there are a number of complications that may occur, with the most common being arterial vasospasm. This will cause delayed cerebral ischemia with changes in mental status and focal neurological deficits, but a CT usually reveals no significant change.

A 61-year-old woman comes to the office for insomnia. She has had issues falling asleep since her divorce was finalized 2 months ago and she relocated closer to her daughter and grandchildren. Her sleep is restless and she is frequently awakened by household noises. She is feeling anxious and tense. She is also overwhelmed by financial worries and responsibilities of living on her own. She was previously outgoing, but now only socializes with her daughter. Her medical history is only significant for hypothyroidism well controlled with levothyroxine (TSH 1.6 microU/mL). What is the most likely diagnosis?

Adjustment disorder - The patient is developing anxiety symptoms in the presents of a recent stressor with no other history of anxiety symptoms. It is within 3 months of this stressor and is distressing and impairing, but insufficient for a diagnosis of another mental disorder.

A 72-year-old man comes to the office due to poor sleep. He is concerned that he is sleeping less since he retired a year ago. He says, "I used to sleep a solid 8 hours, but now I wake up several times a night. And I'm up for the day between 5:00 and 6:00 AM, which is a full hour before I want to get up. I'm probably only getting about 6 or 7 hours of sleep total." The patient typically falls asleep while watching television at 10:00 PM. He sometimes awakens to use the bathroom in the middle of the night but typically falls back asleep within 15 minutes. The patient likes to do crosswords and play golf in the mornings but occasionally takes 30-minute naps in the afternoon. The patient's wife says that he snores softly, but she has not witnessed any breathing pauses. Medical problems include hypertension and a history of depression. The patient does not use alcohol or illicit drugs. Which of the following is the most likely cause of this patient's symptoms?

Age related sleep changes - Age related sleep changes include decreased total sleep time, fragmented sleep, earlier wake-up time, and daytime napping.

A 13-year-old boy with cystic fibrosis comes to the clinic with a persistent cough over the past 2 weeks despite treatment with several oral antibiotics. Temperature is 36.7 C (98.1 F). Pulmonary examination reveals diffuse wheezing. Pulmonary function testing is consistent with an exacerbation of the underlying disease, and the patient is admitted for broad-spectrum intravenous antibiotics and aggressive respiratory therapy. Induced sputum is collected for cultures. In addition to blood, chocolate, and MacConkey agar, this patient's sputum should also be cultured on which of the following media?

Burkholderia cepacia-selective agar - The common bacterial agents responsible for CF exacerbations include Staph aureus, nontypeable Haemophilus influenzae, and Pseudomonas aeruginosa. With progressive disease, patients are also at risk for colonization with Burkholderia cepacia complex, a fastidious gram-negative bacteria that is often difficult to eradicate and associated with increased morbidity and mortality.

A 55-year-old man comes to the emergency department with severe right foot pain that started suddenly in the middle of the night. He has never had such symptoms before. The patient has a history of diet-controlled type 2 diabetes mellitus. BMI is 32 kg/m2. Physical examination shows swelling and tenderness of the first metatarsophalangeal joint. Leukocyte count is 13,500/mm3 and serum creatinine is 0.8 mg/dL. Joint aspiration is performed, and synovial fluid microscopic findings are shown. Which of the following is the best initial treatment for this patient?

Cyclooxygenase inhibitor - This patient is experiencing acute gouty arthritis. NSAIDS are the first line treatment for this exacerbation

A 27-year-old woman in her 2nd trimester of pregnancy comes to the office due to constipation. Her stools have become increasingly hard and pellet-like over the past few weeks; bowel movements before then were always regular. She also has mild crampy abdominal pain and bloating, both of which improve after defecation. Medical history and family history are both unremarkable. This is the patient's first pregnancy and has so far been uncomplicated. On physical examination, the abdomen appears gravid but nontender. Which of the following best explains this patient's symptoms?

Decreased colonic smooth muscle activity - Constipation is fairly common in pregnant patients and is largely due to the fact that progesterone has been shown to reduce colonic smooth muscle activity as it prevents the release of motilin.

The mother of a 7-year-old boy calls the clinic due to a 3-day history of diarrhea. The patient initially had nausea with vomiting on the first day but has since been able to keep down crackers soaked in apple juice. He now has voluminous watery diarrhea several times per day. Several other children in his classroom have also been ill with similar symptoms. The physician schedules the family to come to the office in 2 days and recommends an oral rehydration solution in the meantime. The solution contains dextrose and sodium in equimolar amounts, as well as a small amount of potassium and citrate. Which of following best describes the role of dextrose?

Enhancement of sodium absorption - Sodium is absorbed at the gut brush border by 1:1 cotransport with glucose via sodium-glucose luminal cotransporter-1 (SGLT-1).

A 28-year-old woman, gravida 2 para 1, at 18 weeks gestation comes to the office for a routine prenatal appointment. She is feeling well and has had no complications during this pregnancy. The patient has no chronic medical conditions and has had no previous surgeries. Her only medications are a daily prenatal vitamin and an iron supplement. Blood pressure is 110/70 mm Hg and pulse is 62/min. BMI is 24 kg/m2. Cardiopulmonary examination is unremarkable. The uterus is consistent with 18 weeks gestation and fetal heart tones are normal. There is mild bilateral pedal edema to the shins. Compared to a nonpregnant state, which of the following changes are most likely expected in this patient?

Fibrinogen level: increased Protein S level: decreased Fibrinolysis activity: decrease Pregnancy is a mild prothrombic state

A 19-year-old woman comes to the office for evaluation of amenorrhea. The patient underwent menarche at age 12, and she previously had menses regularly every 25-28 days. However, for the past year, her menses have been irregular, and her last menstrual period was 3 months ago. The patient has no significant medical history, eats mainly fruits and vegetables, and exercises regularly at a local gym. On further questioning, she expresses concern about being overweight and asks for suggestions on cutting down her caloric intake. BMI is 18 kg/m2. The patient is thin and has dry skin covered by fine, downy hair. A pregnancy test is negative. Which of the following laboratory findings are most likely present in this patient?

GnRH - decreased FSH - decreased Estradiol - decreased This patient has secondary amenorrhea, likely due to anorexia nervosa. This causes functional hypothalamic amenorrhea.

A 57-year-old Caucasian male with severe pyelonephritis is admitted to the hospital. His past medical history is significant for diabetes, hypertension, and two episodes of transient ischemic attacks. His serum creatinine level is 3.2 mg/dL; therefore, he needs to be started on an antibiotic that depends mainly on non-renal clearance. Which of the following characteristics should the antibiotic also have if hepatic metabolism and clearance is desired?

High lipophilicity - Drugs with high intrinsic hepatic clearance tend to have high lipophilicity and a high volume of distribution. These drugs can rapidly cross tubular cell membranes after filtration and reenter the tissue, making kidney clearance unlikely. High lipophilicity allows drugs to cross cellular barriers more easily and enter hepatocytes, allowing for an excretion not through the kidneys and a higher amount of distribution.

A 62-year-old man comes to the office due to chest pain over the last 6 months. He describes pressure-like pain in the substernal area when walking fast or climbing stairs that gradually subsides when he stops. The patient does not smoke cigarettes or use illicit drugs. He has no family history of heart disease or sudden death. Physical examination shows a mid-systolic murmur at the upper sternum and slow-rising carotid pulses. After initial evaluation, cardiac catheterization is performed and reveals 30% stenosis of the mid-left anterior descending artery. A 50 mm Hg pressure gradient is seen when a catheter is passed across the aortic valve. Which of the following is the most important contributor to this patient's presenting symptoms?

Increase in left ventricular wall stress - This patient presents with symptoms consistent of severe aortic stenosis. This outflow obstruction leads to higher chamber pressures and increased wall stress during systole and diastole. The rise in systolic wall stress increases myocardial oxygen demand while the increased diastolic wall stress reduces the coronary perfusion pressure gradient, leading to decreased myocardial perfusion.

A 44-year-old woman with a history of breast cancer comes to the office due to persistent back pain. The patient was recently started on radiation treatment for several vertebral metastases. She has taken acetaminophen and nonsteroidal anti-inflammatory drugs but continues to have severe pain. On physical examination, she has tenderness over several lumbar vertebrae. Neurologic examination is unremarkable. The patient is started on oral morphine therapy. Which of the following is the most likely direct effect of this medication on the spinal cord neurons of this patient?

Increased potassium efflux out of the cells - Morphine is an opiate analgesic. In the spine, opiates bind to mu receptors on the primary afferent neuron, resulting in closure of voltage gated calcium channels, reduced calcium influx, and decreased excitatory neurotransmitter release from the presynaptic terminal. Opiates also bind mu receptors on the postsynaptic membrane, which opens potassium channels and leads to membrane hyper polarization due to potassium efflux.

A 47-year-old woman comes to the office with a red rash on her breast that has been present for approximately 2 months. The rash is itchy, and the skin feels rough to the touch. The itching has worsened in the past few weeks, and moisturizers have provided no relief. The patient has a history of surgery for bilateral silicone breast implants 10 years ago. Temperature is 37 C (98.6 F). Physical examination shows redness and swelling of the right breast without any discrete masses. The overlying skin is indurated with a dimpled texture, and several enlarged, hard lymph nodes are palpated in the right axilla. The left breast is normal. Which of the following is the most likely cause of this patient's skin changes?

Lymphatic obstruction - This patient has a classing peau d'orange rash. This rapidly progressive dermatological finding and breast edema are consistent with the characteristic presentation of inflammatory breast cancer, a poorly differentiated breast malignancy with an unfavorable diagnosis. This cancer can obstruct lymphatic drainage after spreading to the dermal lymphatic spaces.

A 4-year-old boy is brought to the clinic by his mother due to decreased appetite, abdominal cramps, and diarrhea over the last 3 days. The stools were initially watery but have now become bloody. Temperature is 38 C (100.4 F). Mucous membranes are dry, and the abdomen is mildly distended and diffusely tender to palpation. Stool studies are positive for fecal leukocytes and occult blood. Stool cultures grow non-lactose fermenting, gram-negative rods on MacConkey agar. The bacteria ferment glucose without gas production, do not generate hydrogen sulfide when grown on triple sugar iron agar, and cannot replicate at refrigeration temperatures. Which of the following bacterial factors is most essential for the pathogenesis of this patient's disease?

Mucosal invasion - This patient likely has Shigella, which survives in M cells and spreads to adjacent cells.

A 4-year-old boy is being evaluated for failure to thrive, shortness of breath, and exercise intolerance. The parents first became aware of his symptoms after the patient started preschool a few weeks ago. His teacher mentioned that the patient seemed to tire faster than the other children when playing outside. Cardiac examination shows bounding peripheral pulses and a palpable thrill below the clavicle near the left uppersternal border. On auscultation, a continuous murmur is best heard over the same region. After discussion with the parents, a thoracotomy is scheduled to correct the patient's condition. During the procedure, the surgeon should plan on ligating a derivative of which of the following embryologic structures?

Sixth aortic arch - This patient likely has a PDA. 1st - part of maxillary artery 2nd - hyoid artery, stapedial artery 3rd - common carotid artery, proximal internal carotid artery 4th - on the left aortic arch, on the right proximal right subclavian artery 6th - proximal pulmonary arteries, on the left ductus arteriosus

A 42-year-old man is brought to the hospital after a motor vehicle collision. The patient was driving while intoxicated with alcohol and collided with another car. Evaluation reveals a left tibia and fibula fracture but no other major injuries. He reports no prior medical conditions. The patient is hospitalized and operative repair of the fracture is performed. On the third day in the hospital, he becomes agitated and demands to leave. Temperature is 37.2 C (99 F), blood pressure is 162/94 mm Hg, and pulse is 125/min. On physical examination, the patient is diaphoretic,tremulous, and disoriented. There are mild hand tremors but no other neurologic abnormalities. Increased activity of which of the following central nervous system receptors is the most likely cause of this patient's condition?

This patient is likely undergoing alcohol withdrawal. Chronic ethanol use causes compensatory downregulation of GABA A receptors. Chronic exposure also leads to a compensatory upregulation of NMDA receptors. Abrupt cessation leads to significantly decreased GABA activity and increased glutamate activity due to the increased number of NMDA receptors, resulting in CNS overexcitation.

A 62-year-old woman comes to the physician after finding a lump in her breast. Physical examination reveals a firm, immovable, irregularly shaped mass. Mammography is performed and shows a spiculated, calcified lesion that was not seen on prior studies. A tissue biopsy is diagnostic for infiltrating ductal carcinoma. Malignant tumors such as infiltrating ductal carcinoma are typically associated with varying degrees of disordered differentiation and maturation. Which of the following findings is most characteristic of an anaplastic tumor?

Brain tumor cells forming giant cells - Anaplastic tumors typically demonstrate the following features: loss of cell polarity with complete disruption of normal tissue architecture, significant variation in the shape and size of cells and nuclei, disproportionately large nuclei that are often deep-staining with abundant, coarsely clumped chromatin and large nucleoli, numerous, often abnormal, mitotic figures, and giant, multi nucleated tumor cells.

An 82-year-old man is brought to the emergency department after a syncopal episode. He has had no chest pain or dyspnea but he has constipation of recent onset. He was hospitalized 2 weeks ago for atrial fibrillation with rapid ventricular response and was discharged home with oral medications after appropriate management. Medical history is also significant for hypertension and severe chronic obstructive pulmonary disease requiring home supplemental oxygen. Blood pressure is 105/60 mm Hg and pulse is 50/min. Examination reveals bilaterally decreased breath sounds, no wheezing, and normal heart sounds. ECG shows new-onset second-degree atrioventricular block. Which of the following drugs is the most likely cause of his current condition?

Diltiazem - The constipation and new-onset second-degree AV block are likely adverse effects of therapy with a nondihydropyridine calcium channel blocker (diltiazem or verapamil). They block the L-type calcium channels, decreasing phase 0 depolarization and conduction velocity in the SA and AV nodes. This will slow the sinus rate and sinus conduction. They also have a negative inotropic event.

A 34-year-old man comes to the clinic with a 3-week history of difficulty hearing. He finds it increasingly difficult to tolerate everyday sounds. He also complains of ear pain and often avoids public places as a result. The patient has no past medical history and takes no medications. Injury to which of the following cranial nerves is most likely responsible for his condition?

Facial - The middle ear cavity contains 3 auditory ossicles (malleus, incus, and stapes) and 2 skeletal muscles (tensor tympani and stapedius) that participate in the transmission of sound from the tympanic membrane to the cochlea. The stapedius nerve innervates the stapedius muscle. This nerve is a branch of the facial nerve (CN VII). Paralysis of this muscle causes the stapes to oscillate more widely, producing hyperacusis. This ipsilateral hyperacusis can be associated with Bell's palsy.

A 24-year-old woman comes to the office with a pruritic rash on her arms and legs; it has been present on and off for most of her life. Examination of the posterior left leg reveals erythematous patches and papules, as shown in the exhibit. A similar rash is present on the right leg and bilateral antecubital fossae. Which of the following cytokines primarily initiated her current exacerbation?

IL 4 and IL 13 - This patient has atopic dermatitis, an inflammatory condition characterized by acute exacerbation of dry, pruritic, erythematous patches, papules, and/or vesicles. As in this patient, flexural regions are commonly affected in adults. The pathogenesis involves immune dysregulation and genetically mediated skin barrier dysfunction. Exposure to environmental allergens induce a Th2-predominant immune response. Key Th2 cytokines that trigger acute inflammation in atopic dermatitis are IL-4 and IL-13 and mechanisms include: stimulation of plasma cell IgE production, suppressed expression of epidermal barrier components, and inhibition of Th1-type immune response and antimicrobial peptide production.

A 45-year-old man comes to the office for an annual medical visit. The patient has had prediabetes for the last 2 years, He feels well and takes no medications, but has gained weight in the past year. There is a strong family history of type 2 diabetes mellitus. BP is 124/78mmHg and BMI is 32kg/m2. Lab results show a fasting blood glucose of 157mg/dL and serum creatinine of 0.7mg/dL. Hemoglobin A1c is 7.4%. Urine assay shows no detectable albuminuria. Which of the following renal changes is most likely present in this patient at this time?

Increased glomerular filtration rate - Diabetic nephropathy can occur with any form of diabetes mellitus and is the most common cause of end-stage renal disease in the US> Increased glomerular filtration rate is one of the the earliest pathogenesis due to increased glucose loads.

A muscle biopsy obtained from a healthy volunteer consists mainly of myoglobin rich, glycogen-poor fibers with many mitochondria. Which of the following is the most likely biopsy site?

Paraspinal - Type 1 fibers perform actions requiring low-level sustained forced and function primarily through aerobic metabolism, meaning they have high myoglobin and mitochondrial concentrations (ex. posture muscles). Type two are specialized for rapid forceful movements, but fatigue quickly due to their use of anaerobic glycolysis

A 28-year-old woman, gravida 1 para 1, comes to the office for a 6-week postpartum checkup following an uncomplicated delivery. The patient reports feeling increasingly fatigued and having little energy for the past few weeks. She feels very anxious about being a new mother and has been getting up multiple times at night to check on the baby when he is sleeping. She tends to skip meals and says, "I don't have time to sit and eat—I can't even find the time to shower." Although the baby is healthy and doing well, the patient berates herself for being a "terrible mother." She becomes tearful during the interview and says, "I don't know why I keep crying when I should be so happy." Physical examination is normal. The patient has no suicidal ideation or thoughts of harming the baby. Which of the following is the most likely diagnosis?

Postpartum depression - PPD is diagnosed using the same criteria as major depressive episodes (more than 2 weeks of 5 symptoms including depressed mood, sleep disturbance, loss of interest, guilt/worthlessness, low energy, impaired concentration, change in appetite, psychomotor retardation or agitation, and suicidal thoughts; SIGECAPS). All mothers should be screened at the 6-week postpartum visit, because if left untreated both the mother and the babies health can be impacted.

In an animal experiment, murine proerythroblasts are cultured in 2 different growth media; the first medium is folate deficient, whereas the second (control) is supplemented with additional folic acid. Both media contain high concentrations of erythropoietin. Over 48 hours, cells in the control media proliferate and differentiate into reticulocytes, whereas in the folate-deficient media, cell proliferation is minimal, with the majority of cells undergoing apoptosis. In another experiment, a substance is added to the folate-deficient media; this substance prevents apoptosis and permits proliferation of some of the proerythroblasts. Which of the following is the most likely substance added to the growth medium?

Thymidine - folate derivatives are crucial in the synthesis of nucleic acids, conversion of homocysteine to methionine, and generation of one-carbon carriers for methylation reactions. They require the reduced tetrahydrofolate form of the vitamin, made by dihydrofolate reductase. Thymidylate synthase is responsible for converting deoxyuridine monophosphate to deoxythymidine monophosphate. De novo thymidine synthesis is particularly susceptible to folate-deficient conditions because tetrahydrofolate must be continuously regenerated by dihydrofolate reductase. In this experiment, inhibition of thymidylate synthase due to deficient folate increases the ration of dUMP to dTMP, causes incorporation of uracil into the DNA. Adding thymidine can prevent apoptosis in these folate deficient cells.

A healthy 31-year-old woman comes to the office as she and her husband desire a second child. The husband is infertile and the patient's son, who was conceived via donor insemination, was recently diagnosed with glycogen storage disease type II (Pompe disease). This rare autosomal recessive disease is known to affect 1 in 40,000 of the general population. Genetic testing confirms that the patient is a carrier for the disease. A different sperm donor is selected with no personal or family history of Pompe disease; however, his carrier status is unknown. What is the probability of the patient having an affected child with the new sperm donor?

1/400 - In this case, 1 in 40,000 individuals is affected by the condition in the general population, so q2 = 1/40,000 and q = 1/√40,000 = 1/200. Thus, the probability of the sperm donor being a carrier = 2q = 2 ×(1/200) = 1/100. Given that a carrier sperm donor would have a 50% chance of passing on the mutant allele (a), the probability of the child being affected is: P (mother gives recessive allele) × P (donor is a carrier) × P (donor gives recessive allele) = (1/2 × 1/100 ×1/2) = 1/400

An infant born to a 26-year-old woman is evaluated shortly after delivery. Birth weight and length are at the 10th and 15th percentiles, respectively. Vital signs are normal. Physical examination shows a protruding tongue, excessive skin at the nape of the neck, and upslanting palpebral fissures. The startle reflex is symmetric and weak. Cardiac auscultation reveals a harsh, III/VI systolic murmur heard best over the lower left sternal border. The patient has normal external female genitalia. Review of maternal medical records shows a past history of 2 spontaneous abortions in the last 3 years. Echocardiography confirms the presence of a ventricular septal defect. Which of the following karyotypes is most likely to be found in this infant?

46, XX, t(14;21) - This patient has translocation down syndrome, which is less common, and can be inherited from an unaffected parent with a balanced translocation. The infant with have an unbalanced Robertsonian translocation with 46 chromosomes and 3 effective copies of chromosome 21.

A large cohort study is conducted to assess the association between smoking and squamous cell carcinoma of the esophagus among middle-aged Chinese men. During 10 years of follow-up, smokers have 5 times the risk of esophageal carcinoma compared to non-smokers (relative risk = 5.0, 95% confidence interval = 2.9-7.1). According to the study results, what percentage of squamous cell carcinoma of the esophagus in smokers can be attributed to smoking?

80% - The attributable risk percent in the exposed = 100 x [(risk in exposed-risk in unexposed)/risk in exposed] = 100 x [(relative risk - 1)/relative risk] ARP of exposed = 100 x [(5-1)/5] = 100 x (4/5) = 100 x 0.8 = 80%

A prospective study evaluates the relationship between regular antioxidant supplement use (vitamins C and E) and the risk of stroke in healthy and physically active men age 40-60. The study compares the risk of stroke among men who consumed antioxidant supplements for ≥5 years and among men who consumed antioxidant supplements for <5 years, as compared to a reference group of men who never consumed antioxidant supplements. According to the study results, men who consumed antioxidant supplements for <5 years and men who consumed antioxidant supplements for ≥5 years have stroke relative risks of 0.95 (p = 0.45) and 0.75 (p < 0.01), respectively, when compared to the reference group. The results of the study were adjusted to account for baseline differences related to healthy behaviors and overall health. Which of the following factors most likely explains why the relative risk of stroke is lower with longer antioxidant use?

Accumulation effect - Long term exposure leads to a change in effect.

A 35-year-old man is admitted to the hospital with a 2-day history of abdominal pain. The patient's condition deteriorates throughout the course of hospitalization, and he dies 5 days after admission. Autopsy reveals chalky white lesions in the mesentery. Histologic evaluation of the lesions reveals adipose cell destruction and calcium deposition. Which of the following is the most likely cause of this patient's autopsy findings?

Acute pancreatitis - The release of lipase and other digestive enzymes from the inflamed pancreas damages nearby adipose cells. Liberated fatty acids bind calcium ions, precipitating as insoluble calcium salts (ie, saponification) that have a chalky white appearance grossly. Microscopically, necrotic adipocytes (ie, anucleate adipocytes with necrotic debris) and blue calcium deposits are seen (compared to a normal pancreas).

A 53-year-old woman comes to the office due to an itchy rash she has had on her left breast for the past month. The patient has applied over-the-counter corticosteroid ointment with no relief of symptoms. The patient's last menstrual period was 2 years ago. She has a history of hypertension that has improved with weight loss and exercise, and she takes no prescription medications. Physical examination shows an eczematous plaque on the left nipple and areola. The remainder of the physical examination is unremarkable. Histologic examination of the skin lesion would most likely show which of the following?

Adenocarcinoma - This patient likely has Paget disease of the breast. Paget cells are malignant, intraepithelial adenocarcinoma cells.

A 32-year-old man with HIV is diagnosed with pulmonary tuberculosis and started on a 4-drug combination therapy. On a follow-up visit 3 weeks later, he reports red urine and red staining of his contact lenses. A drug susceptibility test of his sputum isolates of Mycobacterium tuberculosis shows resistance to several antimycobacterial agents. Which of the following best explains the bacterial resistance to the drug responsible for this patient's current symptoms?

Altered structure of enzymes involved in bacterial RNA synthesis - This patient with pulmonary tuberculosis. Rifamycins (rifampin) cause harmless red-orange discoloration of body fluids. They inhibit DNA-dependent RNA polymerase.

A 52-year-old man is brought to the emergency department after being found unresponsive on the street. The patient's medical history is unknown. Temperature is 36.2 C (97.2 F), blood pressure is 108/62 mm Hg, and pulse is 72/min. On physical examination, he is unresponsive to verbal and tactile stimuli but moans to deep sternal rub. An arterial blood gas analysis on room air shows a partial pressure of oxygen of 60 mm Hg. The partial pressure of oxygen in his alveoli is calculated to be 68 mm Hg. Which of the following is the most likely cause of this patient's symptoms?

Alveolar hypoventilation - This patient has low PaO2 and PAO2 with a normal A-a gradient (68 − 60 = 8), indicating that his low PaO2 is directly due to low PAO2. Possible causes of hypoxemia in the setting of a normal A-a gradient include alveolar hypoventilation and low partial pressure of inspired oxygen (PiO2) (ie, high altitude). Common causes of alveolar hypoventilation include suppressed central respiratory drive (eg, sedative overdose) and diseases that decrease inspiratory capacity (eg, myasthenia gravis, obesity).

A previously healthy 34-year-old woman comes to the emergency department due to 3 days of severe headache. The patient has had a global headache that has progressively worsened and has now become intolerable. She has also had nausea but no fever, emesis, or diarrhea. The patient works as an international reporter and, over the past month, has traveled extensively in Europe and Asia. CT scan of the head is normal. Cerebrospinal fluid results from lumbar puncture are as follows: Opening pressure 280 mm H2O Glucose 60 mg/dL Protein 110 mg/dL Leukocytes 1,000/mm3 Neutrophils 5% Lymphocytes 10% Eosinophils 85% Red blood cells 10/mm3 Which of the following is the most likely cause of this patient's illness?

Angiostrongylus cantonensis - This patient has eosinophilic meningitis, most commonly caused by this helminth.

A 57-year-old female presents to the emergency department after vomiting a substance that resembled "coffee grounds." She reports that she now feels lightheaded as well. Her past medical history is significant for deep venous thromboses, for which she takes warfarin, and occasional joint pain, for which she takes aspirin. Her blood pressure is 90/60 mm Hg and her pulse is 110 beats per minute. Which of the following substances would provide the fastest reversal of warfarin's effects?

Fresh frozen plasma - While vitamin K can help reverse the action of warfarin, it takes time, so in cases of life-threatening bleeding, rapid reversal is achieved through fresh frozen plasma.

A 67-year-old man is found dead in his home. The cause of death is not apparent; he had a long history of hypertension and he had a myocardial infarction a year ago. An autopsy is performed. Gross examination of the heart shows white scarring and enlargement of the left ventricle. Histologic findings are shown. The type of collagen seen in the autopsy sample is most likely to be extensively found in which of the following normal body tissues?

Tendon - Type 1 collagen, is typically found in tendons, ligaments, skin, and bones, but also plays a major role in wound healing and scar formation.

Researchers are studying mechanisms of human infection by animal viruses. The investigators induce random mutations in the genome of an avian influenza virus that is unable to infect humans but is structurally similar to human influenza A virus. A mutated isolate is found to be able to infect human upper respiratory tract epithelial cells. Alteration in which of the following viral components most likely enabled this novel strain to cause cross-species infection?

Antigenic glycoprotein - Interaction with the host cell is dependent upon a viral surface glycoprotein called hemagglutinin. Influenza is prone to antigenic changes. Antigenic changes to hemagglutinin can alter the tissue tropism of the virus to infect humans.

A 78-year-old man is found unresponsive and is brought by ambulance to the emergency department. Upon arrival, the family says he has a complicated medical history and takes multiple medications but has no known drug allergies. Noncontrast CT scan of the head reveals intracranial hemorrhage. The patient is given a recombinant biologic agent that has antigen homology with factor Xa but no catalytic effect. The agent is most likely to antagonize the effects of which of the following drugs?

Apixaban - Apixaban and rivaroxaban both are factor Xa inhibitors

A 30-year-old man comes to the office due to a 4-month history of fatigue that is worse at the end of the day. The patient has also had abdominal bloating and intermittent loose stools after meals. His appetite is normal, and he has had no weight loss. Vital signs are normal. Examination shows pale mucosa. The abdomen is mildly distended and tympanic; there is no tenderness or organomegaly. Rectal examination reveals brown stool that is negative for fecal occult blood. Laboratory results are as follows: Hematocrit 28% Mean corpuscular volume 75 μm3 Leukocytes 7,100/mm3 Platelets490,000/mm3 C-reactive protein level is normal. Which of the following pathologic gastrointestinal tract findings is most likely to be present in this patient?

Atrophic mucosa with loss of villi of the small intestines - This patient's loose stools, abdominal cramps, and microcytic anemia are most likely due to celiac disease, which presents with loss of small-bowel intestinal villi and mucosal atrophy. Iron is normally absorbed in the duodenum, so iron deficiency leads to microcytic anemia.

An 8-year-old boy is brought to the office due to acute facial puffiness. His mother reports that for the preceding 24 hours he has been easily fatigued and has had dark urine. The patient was treated for a skin infection 3 weeks ago but has no chronic medical conditions. Temperature is 36.1 C (97 F) and blood pressure is 140/94 mm Hg. Physical examination shows periorbital edema and mild pitting edema along the ankles. The remainder of the examination shows no abnormalities. A representative renal biopsy sample is shown in the below image. The fluorescent areas on the slide more likely indicate the presence of which of the following substances?

C3 - This patient likely has post-streptococcal glomerulonephritis. These immune complexes visible on immunofluorescence microscopy are granular deposits of IgG, IgM and C3 on the GBM and mesangium, producing a "starry sky" appearance.

A 3-year-old boy is being evaluated for persistent diarrhea. Although the patient seemed healthy at his 12-month well child visit, since then he has experienced 4 episodes of otitis media and 3 episodes of pneumococcal pneumonia. He was at the 50th percentile for weight and height at 12 months but is now at the 25th percentile for height and 10th percentile for weight. The patient is referred for upper gastrointestinal endoscopy, and Giardia lamblia is isolated from duodenal aspirates. Further workup shows very low serum levels of all immunoglobulin types. Flow cytometry of this patient's peripheral blood is most likely to show a near absence of cells bearing which of the following markers?

CD19 - This patient's presentation is suggestive of X-linked agammaglobulinemia, which is a mutation in the Bruton tyrosine kinase gene causing failure of bone marrow pre-B cells to develop into mature B cells, a step necessary for B cells to leave the bone marrow and enter the peripheral circulation. Flow cytometry can be used to assess the number of circulating B cells by using fluorescent tags that bind to specific B cell surface proteins such as CD19, CD20, and CD21.

A 62-year-old man comes to the office due to an intensely pruritic facial rash for the past 3 days. The patient uses no facial cosmetic products but has frequently dyed his hair during the past year; he last dyed his hair 5 days ago and also recalls developing a similar rash the previous time he used hair dye. The patient has a history of asthma, hypertension, and diabetes mellitus. He does not use tobacco, alcohol, or illicit drugs. Vital signs are within normal limits. Physical examination findings are shown in the exhibit. Which of the following are primarily involved in the pathogenesis of this patient's rash?

CD8+ T cells and interferon gamma - This patient is experiencing allergic contact dermatitis, a type IV hypersensitivity reaction, which occurs in 2 phases. Sensitization: cutaneous Langerhans cells take up haptens and present the complexes to naive CD4+ and CD8+ T cells in the lymph nodes, resulting in clonal expansion and this phase takes 10-14 days with no cutaneous lesions. Elicitation: on re exposure, activated CD8+ T cells release cytotoxins and amplify the inflammatory response by releasing cytokines (interferon gamma), taking about 2-3.

A 28-year-old woman comes to the office due to right hand tremors for the past several weeks. The patient has difficulty performing daily activities and feels embarrassed in social gatherings. She has a history of the remitting-relapsing form of multiple sclerosis. There is no family history of tremors. On physical examination, no abnormal hand movement is observed at rest. When the patient is instructed to touch an object on the table, a coarse tremor is observed that gradually increases as the hand moves closer to its target. Dysfunction of which of the following structures is the most likely cause of this patient's tremor?

Cerebellum - this presentation suggests a cerebellar tumor (action tumor which increases as hand reaches target). These tremors are usually low frequency and high amplitude.

A 9-year-old girl is brought to the emergency department by her parents due to severe headache, lethargy, and vomiting. Her symptoms began a few weeks ago as mild intermittent headaches with fatigue and have progressively worsened. CT scan of the head reveals a mass lesion, enlarged lateral and third ventricles, and a normal-sized fourth ventricle. Which of the following is the most likely site of obstruction in this patient?

Cerebral aqueduct - This child's tumor is causing obstructive hydrocephalus limited to the lateral and third cerebral ventricles, which is indicative of an obstruction at the level of the cerebral aqueduct

A 22-year-old woman comes to the office for a routine prenatal visit. She is 16 weeks pregnant with her first child. Two months ago, she had a mononucleosis-like illness with fever, myalgia, and fatigue, and she was diagnosed with cytomegalovirus infection. The patient recovered with symptomatic treatment and is currently asymptomatic. Her other medical problems include episodic migraine headaches and benign choroidal nevus. She takes prenatal vitamins and does not use tobacco, alcohol, or illicit drugs. The patient has a family history of hypertension and glaucoma. Physical examination findings and prenatal ultrasound are unremarkable. This patient's unborn infant is at greatest risk for which of the following eye conditions?

Chorioretinitis - The highest risk of transmission of CMV from mother to baby is during the first trimester. CMV-related complications for infants exposed in utero include chorioretinits, sensorineural deafness, seizures, jaundice, hepatomegaly, splenomegaly, and microcephaly

A 47-year-old woman comes to the physician with progressive joint pain and swelling in her hands for the past several months. She also has easy fatigability that has gradually worsened over the same period. Morning activities are especially difficult due to stiffness lasting 1 to 2 hours after waking. Examination shows warmth, swelling, and tenderness involving the proximal interphalangeal joints, metacarpophalangeal joints, and wrists bilaterally. A blood sample is obtained for laboratory analysis. Autoantibodies against which of the following components are most specific for this patient's condition?

Citrullinated peptides - This patient presents with symptoms highly suggestive of RA. The presence of anti-cyclic citrullinated peptide antibodies is helpful in confirming this diagnosis.

A 46-year-old woman is admitted to the hospital with dehydration secondary to excess output from an ileostomy. Five years ago, the patient had a total colectomy with a diverting ileostomy for colon cancer. For the last 6 months, she has had increased output from the ileostomy and has been admitted to the hospital twice with similar episodes of dehydration. On the second day of admission, she reports right flank pain. X-ray of the abdomen reveals a nonspecific bowel gas pattern with no evidence of renal calculi. Ultrasound of the abdomen shows a 4-mm stone in the distal right ureter. The patient is treated with analgesics and the stone passes spontaneously. Microscopic analysis of the stone reveals a pure uric acid stone. Which of the following is the most likely underlying mechanism leading to stone formation in this patient?

Concentrated acidic urine - This patient has dehydration which can be causing a chronic metabolic acidosis. The kidneys will compensate by excreting more hydrogen ions and absorbing more bicarbonate. This lowers urine pH and increases the conversion of soluble urate ions into insoluble uric acid.

A resident physician is finishing up her shift when she receives a page that reads, "Patient in Room 121 asking to leave." The patient was admitted 2 hours ago for worsening dyspnea. On entering the room, the resident attempts to engage in a discussion about why the patient is requesting to leave. In response, the patient pulls out his nasal cannula and breathlessly says, "You don't know anything; you're not even a real doctor!" The resident is reminded of her father, who has frequently belittled her accomplishments. She angrily informs the patient, "You can't leave and I'm ordering haloperidol to calm you down." Which of the following best explains the resident's response to the patient?

Countertransference - A providers reaction based on past personal relationships that can interfere with medical judgment.

A 43-year-old man comes to the office due to shortness of breath and fatigue. Over the last 2 weeks, his fatigue has been so profound that he has "little energy, even to get out of bed." The patient has no chills but has experienced recent weight gain and ankle swelling. He has no prior medical conditions and takes no medications. Blood pressure is 168/94 mm Hg, and pulse is 95/min and regular. The patient has bilateral lower extremity pitting edema limited to the ankles. Urinalysis reveals 2+ protein, white blood cell count of 5-7/hpf, and red blood cell count of 75-100/hpf. He undergoes a kidney biopsy; immunofluorescent microscopy findings are shown. Which of the following would be the most likely finding on light microscopy in this patient?

Crescent formation - The immunofluorescence pattern has a linear appearance, indicating anti-glomerular basement membrane disease (Goodpasture disease) This results in a form of rapidly progressive (crescentic) glomerulonephritis.

A 65-year-old man comes to the office due to pain, redness, and swelling in his right calf. The patient is diagnosed with cellulitis and clindamycin is initiated. A few days after starting treatment, he develops watery diarrhea and abdominal cramps. The patient is hospitalized, and a complete blood count reveals leukocytosis. The toxin responsible for this patient's condition most directly affects which of the following components of intestinal mucosal cells?

Cytoskeleton integrity - These strains produce toxin A and toxin B, which act synergistically, although toxin B is significantly more virulent. The toxins bind specific receptors on intestinal mucosal cells and are internalized, allowing them to inactivate the Rho-regulatory proteins involved in the maintenance of actin cytoskeletal structure. The result is loss of cytoskeleton integrity, leading to cell rounding/retraction, disruption of intercellular tight junctions, and increased paracellular intestinal fluid secretion (eg, watery diarrhea). Both toxins also have inflammatory effects (eg, neutrophil recruitment) and can induce apoptosis, resulting pseudomembrane formation.

A 19-year-old man comes to the office with an ulcer on his penis. The patient first noticed the lesion 3 days ago. His temperature is 37.1 C (98.8 F). Physical examination shows an indurated and painless ulcer near the glans penis, with no surrounding erythema and no inguinal lymphadenopathy. The patient is a college student. He has no significant past medical history and takes no medications. He has no known drug allergies. The first-line treatment for this patient has structural similarities with which of the following?

D-alanine-D-alanine - This patient likely has a chancre due to primary syphilis. Spirochetes are universally sensitive to penicillin, which works by covalently binding and inhibiting transpeptidase. Penicillin is structurally similar to D-alanine-D-alanine

A 34-year-old man is found to have dyslipidemia. His other medical problems include a myocardial infarction 1 week ago. His father died of myocardial infarction at the age of 48 years. Several other family members have had myocardial infarctions and hypertension. The patient smoked cigarettes and drank alcohol occasionally but quit after his myocardial infarction. He is started on simvastatin and cholestyramine. Which of the following best describes the independent effects of simvastatin and cholestyramine, respectively, on hepatic cholesterol synthesis?

Decreased and increased - Statins are fist line for most patients with hypercholesterolemia, which inhibit HMG CoA reductase, which in turn decreases hepatic cholesterol synthesis. Bile acid-binding resins work by binding bile acids, causing increased bile acid excretion, resulting in hepatic synthesis of new bile acids. This depletes cholesterol stores therefore causes an increased hepatic cholesterol synthesis.

A 51-year-old woman comes to the office because it feels like the right side of her face has been getting heavier for the past 12 hours. Medical history is significant for type 2 diabetes mellitus. Physical examination reveals asymmetry of her face when she smiles, puffs out her cheeks, raises her eyebrows, and tries to close her eyes tightly. There is also effacement of the right nasolabial fold, and her lips are drawn toward the left side. The remainder of the neurologic examination is normal. Which of the following additional findings is most likely associated with this patient's condition?

Decreased tear production from the right eye - This patient is exhibiting signs of a VII nerve palsy, often referred to as Bell palsy or peripheral facial nerve palsy. CN VII consists of: motor output to the muscles controlling movement of the face, somatic sensation afferents from portions of the pinna and external auditory canal, special sensation afferents for taste from the anterior two-thirds of the tongue, parasympathetic innervation of the submandibular and lacrimal glands, and motor innervation of the stapedius muscle, which causes sound dampening.

A 29-year-old previously healthy man comes to the hospital with a right leg injury after a cycling accident. He reports pain in the lower leg and inability to bear weight. X-ray reveals right tibial and fibular shaft fractures. The patient undergoes closed reduction and immobilization of the fractures. Several hours later, he has increasingly severe pain in his right leg. Physical examination shows increased tension within the anterior compartment. Which of the following structures is most likely to be compromised by this patient's acute complication?

Deep peroneal nerve - This patient likely has acute compartment syndrome, which is a complication possible from long-bone fractures, crush injuries, thermal injuries, or nontraumatic cases. Early diagnosis and surgical decompression are needed to avoid permanent injury. The anterior compartment, which is the most common site for ACS, includes the foot extensor muscles, anterior tibial artery, and the deep peroneal (fibular) nerve

A 15-year-old boy develops severe cardiomyopathy following myocarditis from Coxsackie virus and is placed on the cardiac transplant list. Two weeks following his cardiac transplantation from a matched donor, he develops dyspnea on exertion. Extensive evaluation is undertaken including cardiac catheterization and endomyocardial biopsy to assess for acute allograft rejection. Which of the following histologic findings is most consistent with this diagnosis?

Dense interstitial lymphocytic infiltrate - Acute cardiac transplant rejection typically occurs within 6 months of transplantation and can be diagnosed via endomyocardial biopsy. Acute rejection can be cell mediated, or less commonly, antibody mediated. In ACR, host T lymphocytes are sensitized against foreign human leukocyte antigens in the cardiac allograft, which leads to inflammation and injury. Microscopic features include an interstitial lymphocytic infiltrate and damaged myocytes.

A 15-year-old boy is brought to the emergency department due to hemoptysis. He has a history of amputation of the right lower extremity due to bone cancer. Chest imaging reveals a lung mass. Excisional biopsy of the mass shows sheets of uniform, round, small cells slightly larger than lymphocytes with scant, clear cytoplasm. The cellular deposits are interrupted by vascular fibrous septa, with areas of hemorrhage and an abrupt transition from viable to necrotic cells. A representative sample is shown. Which of the following is the most likely diagnosis?

Ewing sarcoma - Ewing sarcoma has uniform, small, round cells and is the second most common primary malignant bone tumor in children and typically occurs in the diaphysis of long bones or in the pelvis. It often metastasizes to the lungs and pulmonary complications are a leading cause of death.

A 43-year-old man comes to the emergency department due to a 3-day history of persistent headaches. The patient has a history of hypertension and has had poor medical follow-up. Blood pressure is 224/115 mm Hg and pulse is 67/min. He appears mildly confused during the physical examination, but no focal neurologic deficits are noted. Funduscopic examination shows bilateral papilledema. Serum creatinine is 1.4 mg/dL. An intravenous medication is initiated that causes arteriolar dilation while also improving renal perfusion and increasing natriuresis. Which of the following agents is most likely being used in this patient?

Fenoldopam - This patient has severe hypertension and clinical evidence of secondary end-organ damage. This is a consistent presentation with a hypertensive emergency, which requires aggressive treatment. Fenoldopam is a short-acting, selective, peripheral dopamine-1 receptor agonist. Dopamine-1 receptor stimulation activates adenylyl cyclase and raises intracellular cyclic AMP, resulting in vasodilation of most arterial beds with a corresponding decrease in systemic blood pressure. Renal vasodilation is particularly prominent, and leads to increased renal perfusion, increasing urine output and natriuresis.

A 62-year-old man is hospitalized with severe abdominal pain and diarrhea after a recent urinary tract infection. Six months ago, the patient had an episode of Clostridioides difficile colitis after being treated for pneumonia. Medical history is also significant for diverticulitis and upper gastrointestinal tract bleeding. The patient is allergic to penicillin. An appropriate work-up confirms C difficile colitis. He is administered an oral macrolide antibiotic that inhibits the sigma subunit of RNA polymerase. Which of the following agents was most likely initiated in this patient?

Fidaxomicin - This drug is a macrolide antibiotic that inhibits the RNA polymerase

A 30-year-old woman comes to the office due to a 6-month history of increasingly severe fatigue and malaise. For the past 5 years, she has lived on a small farm and eats only vegetables, fruits, and grains she grows herself. Medical history is unremarkable. The patient takes no medications or nutritional supplements. She does not use tobacco or alcohol. This patient is at increased risk for deficiency involving which of the following nutrients?

Folic Acid - No Iron - Yes Thiamine - No B12 - Yes A vegan diet is at risk for cobalamin, calcium, vitamin D, and iron deficiencies.

A 4-month-old girl is brought to the office due to a rash on her cheeks for the past 2 weeks. The rash has not spread, and the patient is often seen scratching her face. There have been no changes in bath soaps and detergents; no animals or plants are in the house. Vaccinations are up to date. The patient is exclusively breastfed, with height and weight tracking along the 50th percentiles. Vital signs are normal. Skin examination is shown in the exhibit. The remainder of the examination is unremarkable. Which of the following conditions is this patient at increased risk for developing?

Food allergy - This infant shows patches on her cheeks, common of atopic dermatitis. Patients with atopic dermatitis and elevated IgE levels are at increased risk for other atopic diseases, such as allergic rhinitis, asthma, and food allergies.

A 43-year-old woman comes to the emergency department due to low-grade fevers and malaise. She has a history of lung transplantation due to cystic fibrosis and had 2 episodes of cytomegalovirus (CMV) viremia over the past few months, which were treated with ganciclovir. Temperature is 37.6 C (99.7 F). Physical examination is unrevealing. Chest x-ray and urinalysis are negative for infection. Polymerase chain reaction testing for CMV in blood shows elevated CMV levels consistent with CMV viremia. Given the recurrent episodes of viremia, genotype analysis is performed and demonstrates ganciclovir-resistant CMV. The patient is started on an alternate intravenous antiviral agent. She develops hypocalcemia and hypomagnesemia. Which of the following agents is the most likely cause of these side effects?

Foscarnet - Foscarnet is a pyrophosphate analog that is sometimes used for ganciclovir-resistant CMV infections. Foscarnet can chelate calcium. Foscarnet-induced renal wasting of magnesium may lead to hypomagnesemia and a reduction in the release of parathyroid hormone which contributes to the hypocalcemic state. Both of these states can promote seizures.

A 54-year-old woman is evaluated for progressive constipation, anorexia, and a 5.4-kg (12-lb) weight loss over the past several months. Physical examination is unremarkable. Stool guaiac test is positive, and a colonoscopy is performed. An exophytic mass is identified in the sigmoid colon. The patient undergoes a left hemicolectomy, and histopathology of the surgical specimen is positive for adenocarcinoma. Molecular testing of the cancer cells reveals a mutation in the KRAS gene that results in constitutive activation of the Ras protein. Under normal circumstances, this protein is only active when bound to which of the following substances?

GTP - RAS genes code for a family of small G-proteins involved in signal transduction in the Ras-MAPK pathway. It exists in an inactive GDP-bound state and an active GTP-bound state. It becomes activated when a growth factor ligand binds to a receptor tyrosine kinase located on the cell membrane cause autophosphorylation of the receptor. Activated Ras begins a phosphorylation cascade that results in the activation of mitogen-activated protein kinase (MAPK)m which enters the nucleus to influence gene transcription.

A 56-year-old man comes to the office due to difficulty swallowing for the past several months. He has the most trouble with solid foods and says, "They seem to get stuck in my throat if I don't chew a lot." The patient has no chest pain or heartburn and has lost 4.5 kg (10 lb) in the last 3 months. He has been an avid hunter for many years and frequently cures the meat he eats with sodium nitrite. Physical examination is unremarkable. Endoscopy shows an ulcerated mass in the distal third of the esophagus, and biopsy samples are obtained from the mass and adjacent normal mucosa. Analysis of the samples shows accelerated cytosine deamination of chromosomal DNA in both normal and malignant epithelial cells. This damage is most likely to be repaired through which of the following enzymatic sequences?

Glycosylase, endonuclease, lyase, polymerase, ligase - Base excision repair is responsible for repairing various non-bulky DNA base alterations. Excessive consumption of dietary nitrates can promote the deamination of cytosine, adenine, and guanine to form uracil, hypoxanthine, and xanthine. If these changes are not fixed, mutations and carcinogenesis may result. The process begins with recognition of abnormal bases by specific glycosylases, which cleave the altered DNA bases, leaving an empty sugar-phosphate site (AP). An endonuclease then cleaves the 5' end of the AP site before lyase enzyme subsequently completes extraction of the AP site from the DNA molecule by removing the remaining sugar-phosphate group. DNA polymerase then fills the gap with the correct sugar-phosphate base and the final nick is sealed by ligase.

A 42-year-old weightlifter is preparing for an upcoming strength competition. After lifting a series of progressively heavier weights, he decides to increase the weight on the exercise bar to 175 kg (385 lbs), the heaviest lift he has ever attempted. He is able to hold the weight over his head for several seconds. However, his arms suddenly and involuntarily give way and he drops the weight to the ground. Which of the following structures was most likely responsible for the sudden muscle relaxation?

Golgi tendon organ - GTOs are sensory receptors at the junction of muscle and tendon, innervated by group Ib sensory axons. When the muscle contracts actively against resistance, there is an increase in tension in the tendon. When a muscle exerts too much force, the GTOs inhibit contraction of the muscle, causing sudden muscle relaxation.

A 68-year-old woman comes to the emergency department due to several hours of severe upper back pain. The patient developed the pain after experiencing a sudden jolt while driving over a pothole. She has no previous history of back pain or major trauma and has not seen a doctor in many years. The patient is retired and lives a sedentary lifestyle. She drinks a glass of wine daily and eats a healthy, balanced diet. BMI is 18.4 kg/m2. On examination, there is point tenderness over the T10 vertebra. Neurologic examination is unremarkable. X-ray of the spine reveals a T10 compression fracture. What is the patient's likely serum calcium and parathyroid hormone levels?

High parathyroid hormone level and low serum calcium - This patient has osteoporosis

Researchers are investigating the use of immune checkpoint inhibitors for the treatment of advanced colorectal cancer. They find that this therapy is more effective in patients with DNA mismatch-repair deficiency compared to patients with an intact mismatch-repair mechanism. Which of the following most likely explains the improved efficacy of the drug in the first group of patients?

Higher burden of tumor neoantigens - DNA mismatch repair is a highly conserved pathway that maintains genomic integrity by correcting base-base mismatches and insertions/deletions during DNA replication, recombination, or repair. Unrepaired oncogenic mutations accumulate. Large quantities of neoantigens accumulate due to frameshift mutations.

A 38-year-old man comes to the office due to pain in multiple joints. He has a 5-year history of lumbar pain and a 2-year history of bilateral knee pain. The patient works in construction and his pain is worst after a long day on his feet. He has taken ibuprofen intermittently, but the pain is no longer tolerable. The patient has a paternal aunt with osteoarthritis. Physical examination shows blue-black spots on his sclerae and diffuse darkening of the auricular helices. Which of the following is the most likely cause of this patient's arthritis?

Homogentisic acid dioxygenase deficiency - This patient is presenting with alkaptonuria which is relatively benign but can present with severe arthritis later in adult life. It is an autosomal recessive disease caused by deficiency of homogentisic acid dioxygenase

A 45-year-old woman is evaluated for numbness and painful tingling of the right hand. The symptoms began 2 months ago and are especially prominent after she plays tennis. On examination, there is diminished sensation in the lateral palm, thenar eminence, and palmar aspect of the first 3½ digits of the right hand. This patient most likely has an injury involving a nerve that courses between which of the following structures?

Humeral and ulnar heads of the pronator teres muscle - This patient has diminished sensation over the distribution of the median nerve, exacerbated by repetitive, forceful pronation. The median nerve arises from the medial and lateral cords of the brachial plexus and travels with the brachial artery in the medial bicipital groove (ie, longitudinal hollow between the biceps brachii and triceps brachii muscles) of the upper arm. In the forearm, the median nerve courses between the humeral and ulnar heads of the pronator teres muscle and between the flexor digitorum superficialis and flexor digitorum profundus muscles before entering the hand through the carpal tunnel.

A 65-year-old man comes to the office due to a few weeks of progressive exertional dyspnea and fatigue. He has had no chest pain, fever, or cough. The patient has a history of mild aortic stenosis, gastroesophageal reflux disease, obesity, hypertension, and hypercholesterolemia. Temperature is 36.7 C (98 F), blood pressure is 130/78 mm Hg, pulse is 82/min, and respirations are 16/min. BMI is 28 kg/m2. Pulse oximetry shows 96% on room air. Mucosal pallor is present, but there is no jaundice, lymphadenopathy, or jugular venous distension. Cardiopulmonary examination reveals a 2/6 systolic ejection murmur at the second right intercostal area. The abdomen is soft and nontender; there is no palpable hepatosplenomegaly. Peripheral pulses are normal. Laboratory results are as follows: Complete blood count Hemoglobin 7.8 g/dL Platelets 98,000/mm3 Leukocytes 3,800/mm3 A peripheral blood smear is shown. Which of the following is most likely responsible for the patient's current condition?

Impaired DNA synthesis - This patient has macrocytosis and a hyper-segmented neutrophil, which likely indicate megaloblastic anemia. This could likely be due to B12 deficiency due to antacid use. This lack of vitamin B12 impacts purine synthesis, which in turn impaired DNA synthesis

A 48-year-old homeless man is brought to the emergency department after being found unresponsive on the sidewalk. Medical history is unknown. Temperature is 38.3 C (101 F), blood pressure is 116/70 mm Hg, pulse is 108/min, and respirations are 26/min. Oxygen saturation is 88% on 100% oxygen via face mask. The patient is ill-appearing, thin, and disheveled with poor dentition. There are a few white patches on the oral mucosa, but no skin rash is present. Lung auscultation reveals bilateral crackles. There are healed abdominal scars from a previous surgery. The abdomen is soft with no hepatomegaly or splenomegaly. Chest x-ray reveals bilateral interstitial infiltrates. A silver-stained bronchoalveolar lavage specimen is shown in the image. specimen is shown in the image. Which of the following most likely predisposed this patient to his current lung condition?

Impaired cell-mediated immunity - This presentation indicates likely Pneumocystis pneumonia, an atypical fungal infection cause by P. jiroveccii. Inhalation leads to inoculation of the alveolar space. Typically alveolar monocytes and macrophages are stimulated by the cell-mediated immune response to rapidly clear the organism. However, in patients with impaired cell-mediated immunity, such as those with AIDS, they often cannot eliminate the pathogen.

A 5-week-old boy is brought to the emergency department after having tonic-clonic movements of his left upper and lower extremity that lasted approximately 3 minutes. The patient was born at 38 weeks gestation to a 23-year-old primigravid woman. The mother received normal prenatal care, took prenatal vitamins, and had a healthy diet during pregnancy. The infant's newborn screen was positive for cystic fibrosis, and confirmatory testing is pending. The infant has been breastfeeding exclusively. He has never taken any medications and has received no vaccinations. On examination, the infant is postictal. A CT scan of the head shows a right-sided intracranial hemorrhage. Which of the following is the most likely cause of this infant's presentation?

Impaired gamma carboxylation - This infant who has not received any medications and is being exclusively breast fed is likely deficient in vitamin K, leading to bleeding. Vitamin K is an essential cofactor for gamma-glutamyl carboxylase, an enzyme that carboxylates coagulation factors II, VII, IX, and X. Vitamin K prophylaxis is important in newborns.

A 66-year-old man with poorly controlled type 2 diabetes is admitted to the hospital due to a 2-day history of fever and confusion. Blood pressure is 110/50 mm Hg. Pulse oximetry shows an oxygen saturation of 97% on room air. Examination reveals warm extremities with full peripheralpulses, an infected neuropathic foot ulcer with surrounding cellulitis, and normal lung sounds. A central venous catheter terminating in the superior vena cava is placed. Blood aspirated from the catheter appears red. Blood gas analysis of this venous sample reveals an oxygen saturation of 87% (normal: 65%-70%), and lactate is moderately elevated. Which of the following is the most likely cause of this patient's abnormal blood gas findings?

Impaired mitochondrial oxidative respiration in the vital organs - This patient has sepsis due to an infected diabetic foot ulcer with surrounding cellulitis. Sepsis can lead to multiple organ system dysfunction. Free radicals from bacterial components and acute phase cytokines damage the mitochondria, which leads to decreased oxidative phosphorylation, which will lead to a compensatory shift to glycolysis leading to lactic acidosis.

A 3-year-old boy is brought to the physician with jaundice and pallor. He was adopted and his family history is unknown. Laboratory studies show anemia, reticulocytosis, and increased indirect bilirubin. A peripheral blood smear shows red blood cells without central pallor. This patient is most likely to have which of the following additional findings?

Increased mean corpuscular hemoglobin concentration - The patient's presentation with anemia, reticulocytosis, and increased indirect bilirubin indicate hemolytic anemia. The lack of central pallor on red blood cells suggests acquired or hereditary spherocytosis. Hereditary is caused by defective binding of the red cell cytoskeleton to the plasma membrane due to mutations involving ankyrin, band 3 or spectrin proteins, whereas acquired is most often caused by autoimmune hemolytic anemia. These cells are smaller and have more intensely staining cytoplasm due to membrane loss and red cell dehydration. You will see an elevation in mean corpuscular hemoglobin concentration.

A 48-year-old woman with metastatic cervical cancer is brought to the emergency department due to worsening lethargy for the past several days. A year ago, the patient was diagnosed with cervical cancer, which has progressed despite chemotherapy treatment and metastasized to the liverand lungs. She has also developed bilateral hydronephrosis and renal failure due to ureteral compression by the tumor. The patient is receiving palliative care, and her pain has been adequately controlled with a stable dose of oral morphine. Temperature is 36.7 C (98 F), blood pressure is 110/62 mm Hg, pulse is 92/min, and respirations are 10/min. On physical examination, the patient is somnolent and responds to painful stimuli only. The pupils are small and sluggish to react. The lungs are clear to auscultation, and heart sounds are normal. The abdomen is soft and nondistended. Arterial blood gas analysis shows respiratory acidosis. Which of the following medication-related events most likely precipitated her current condition?

Increased metabolite accumulation - Due to this patients renal dysfunction, metabolites are accumulating.

A 60-year-old man comes to the office for follow-up due to persistent atrial fibrillation. He reports occasional palpitations and poor exercise tolerance. For the past year, the patient's atrial fibrillation has been managed with a rate-control strategy using metoprolol. Three months ago, his metoprolol dosage was increased for improved rate control, but today in the office, he reports that he was unable to tolerate the new dosage due to dizziness. Blood pressure is 110/70 mm Hg and pulse is 105/min and irregular. Examination shows no abnormalities. Echocardiography reveals left atrial enlargement, an ejection fraction of 59%, and no significant valvular disease. The patient is advised to remain on his previously tolerated metoprolol dose, and digoxin is added to the medication regimen. Four weeks later, he reports symptom improvement. Resting pulse is 84/min and irregular. Which of the following best explains digoxin's effect on heart rate in this patient?

Increased parasympathetic tone - Rate control is important in atrial fibrillation because it minimizes symptoms and decreases the risk of tachycardia-induced cardiomyopathy. Beta blockers slow conduction through the AV node. Digoxin was added to slow conduction through the AV node through increased parasympathetic tone.

A 2-year-old, previously healthy boy is brought to the emergency department due to a 4-day history of poor feeding, diarrhea, and abdominal pain. The patient has had 5-7 bowel movements every day for the past 4 days. Temperature is 36.8 C (98.2 F) and blood pressure is 65/42 mm Hg. On physical examination, the patient is lethargic. Mucous membranes are dry, and the eyes are sunken. The lungs are clear to auscultation. No murmurs are heard. Abdominal examination shows distension and tenderness; bowel sounds are hyperactive throughout. Which of the following changes is most likely present in this patient?

Increased ventilatory rate - this patient is showing signs of hypovolemia and shock. There is poor organ and tissue perfusion with shock that leads to tissue hypoxia and lactic acidosis. This leads to compensatory respiratory alkalosis to blow off CO2.

A 4-year-old girl is brought to the clinic for her routine yearly visit. She has no chronic medical conditions and has received all previously recommended vaccinations. The patient is due to receive several vaccinations today including a second measles vaccine. Which of the following IgM and IgG trends is expected to occur after initial and subsequent measles vaccinations?

Initially will see a peak in IgM around day 10 and IgG around day 15-20, on secondary exposure, will see a very large peak in IgG at day 5-10 and a similar response in IgM as to first exposure

A 34-year-old woman with polycystic ovary syndrome comes to the office with her husband for treatment of infertility. The patient has been unable to conceive despite having unprotected sexual intercourse several times a week for the past 2 years. Her menses are irregular and occur every 2-3 months, consistent with chronic anovulation. She does not use tobacco, alcohol, or illicit drugs. The patient takes no medications and has no allergies. BMI is 32 kg/m2. Physical examination shows coarse hair on her chin and abdomen. The patient receives ovulation induction therapy with a short course of gonadotropins followed by a single injection of hCG. The use of hCG therapy in this patient primarily mimics the normal physiologic increase in which of the following hormones?

LH - The hormones hCG, LH, FSH, and TSH share a common alpha subunit; in addition, the beta subunit of hCG is also structurally similar to the beta subunit of LH. Due to this beta subunit similarity, hCG can bind to and activate the same receptors as LH. Therefore, the hCG injection mimics the LH surge and acts as the ovulation trigger.

A 64-year-old man with a long-standing history of hypertension is brought to the emergency department with a dry cough and shortness of breath. The patient has been unable to sleep in the flat position for the past 2 days. Blood pressure is 192/102 mm Hg and pulse is 92/min and regular. Physical examination reveals an S4 and bibasilar crackles. He is started on intravenous nitroglycerin infusion and soon after reports significant symptomatic relief. Which of the following physiologic changes are most likely to occur following administration of this medication?

Left ventricular end-diastolic pressure - decreased Peripheral venous compliance - increased Systemic vascular resistance - decreased Nitrates are primarily venodilators that increase peripheral venous capacitance, thereby reducing cardiac preload and left ventricular end-diastolic pressure and volume. They also have a modest effect on arteriolar dilation.

A 17-year-old boy comes to the office due to a skin rash that has been present for a day. One week ago, he had a sore throat that improved with oral penicillin therapy. Skin examination shows several violaceous, raised, nonblanchable lesions distributed over the bilateral lower extremities. Histologic findings of the biopsied rash are shown in the image belowWhich of the following is the most likely diagnosis in this patient?

Leukocytoclastic vasculitis - Cutaneous small vessel vasculitis is a vasculitis that only affects the skin and typically arises due to drug or pathogen exposure (eg, hepatitis B or C virus). Drugs known to cause this condition include penicillins, cephalosporins, sulfonamides, phenytoin, and allopurinol. On skin examination, nonblanching palpable purpura is usually present and often involves the lower extremities. When biopsied, these skin lesions histologically demonstrate markedly inflamed small blood vessels with fibrinoid necrosis. In the first 24 hours, the perivascular inflammatory cell population consists primarily of neutrophils and fragmented neutrophilic nuclei (leukocytoclastic vasculitis), with mononuclear cells predominating in older lesions.

Researchers are studying the mechanisms that lead to acute kidney injury in septic shock. In an experiment, the renal perfusion pressure of laboratory mice is reduced to assess changes in renal tubular function and morphology. Perfusion pressure is decreased until renal tubular dysfunction occurs due to sublethal cellular ischemia; recovery of function is achieved within several hours of reperfusion. Which of the following is most likely the earliest change to occur within the renal tubules during the ischemic period?

Loss of epithelial cell polarity - The mice are observed to have rapidly reversible renal tubular dysfunction after exposure to sublethal ischemia, which is consistent with acute tubular necrosis. Loss of epithelial cell polarity occurs early in ATN due to alterations in the actin cytoskeleton. The actin cytoskeleton is very sensitive to loss of ATP, so ischemia causes rapid cytoskeleton disruption resulting in blunting of apical microvilli, loss of cell-cell adhesion, redistribution of integrins, and redistribution of Na-K-ATPases.

A newborn girl is evaluated in the nursery after an uncomplicated spontaneous vaginal delivery to a 29-year-old primigravida. The mother declined prenatal testing and ultrasound examination during the pregnancy. Her pregnancy was otherwise uneventful, and she took prenatal vitamins throughout. Examination of the neonate shows a posterior neck mass and bilateral nonpitting edema of the hands and feet. Femoral pulses are diminished. Neck ultrasound reveals a mass composed of cystic spaces separated by connective tissue. Which of the following is the most likely underlying mechanism responsible for this patient's condition?

Loss of paternal chromosome X - This patient likely has Turner syndrome. The patient's posterior neck mass is most likely a cystic hygroma and the bilateral extremity swelling is consistent with lymphedema. These findings in the context of diminished femoral pulses (suggestion of an aortic coarctation) in a newborn is a classic presentation of Turner syndrome, which is caused by a loss of paternal chromosome X.

A group of researchers is studying population-based screening interventions to reduce mortality from abdominal aortic aneurysm. This disorder develops primarily in elderly patients and is often asymptomatic until an acute rupture event, which is frequently fatal. Screening of high-risk patients for abdominal aortic aneurysm is found to reduce mortality. Which of the following risk factor combinations would likely define the highest-risk group for screening purposes?

Male, smoking - Abdominal aortic aneurysm involves transmural inflammation of the aortic wall with apoptosis of smooth muscle cells and degradation of matrix proteins, which causes thinning of the all. Major risk factors include age >65, smoking, and male sex.

A 5-year-old girl is brought to the office by her mother due to 2 weeks of anorexia, nausea, epigastric discomfort, and loose bowel movements. She has had no fever or bloody stools. The patient recently returned from rural Brazil, where she spent the summer with family. She had no gastrointestinal symptoms during the trip but developed intensely pruritic eruptions between the toes of her right foot. The skin rash spontaneously resolved within a few days and was attributed to insect bites from walking barefoot in the fields. The patient has no prior medical conditions, takes no medications, and has received all age-appropriate vaccinations. On physical examination, the abdomen is soft and nontender with normoactive bowel sounds. Stool microscopy reveals smooth, thin-walled eggs. If left untreated, this patient's condition can lead to which of the following complications?

Microcytic anemia - This patient likely has a hookworm infection, which commonly occurs in tropic and subtropical regions. Eggs are found in human feces and larvae spread occurs when coming in contact with human skin. This worm feeds on human blood by lacerating capillaries and can be in the GI tract for up to 14 years, so microcytic anemia due to chronic blood loss is a possibility.

A 1-hour-old girl born to a 40-year-old woman is brought to the nursery for evaluation. The pregnancy and delivery were uncomplicated. Physical examination shows mid-face hypoplasia with a flat nasal bridge, up-slanting palpebral fissures, a small mouth, and a single palmar crease bilaterally. Cardiac auscultation reveals a blowing holosystolic murmur heard best along the sternal border. Which of the following abnormalities is most likely to be present in this patient?

Mosaicism - Mosaicism accounts for <2% of DS cases. Affected individuals have 2 distinct cell lines as a result of nondisjunction during mitosis: one with a normal genotype and one with trisomy 21. The proportion of affected cells determines the severity of DS features. Of the available answer options, only mosaicism is consistent with a third copy of chromosome 21 existing in at least a portion of the patient's cells.

A 60-year-old farmer is brought to the emergency department due to acute-onset confusion, muscle weakness, and difficulty breathing. The patient has vomited twice and has had 3 loose bowel movements over the past 2 hours. His medical history is insignificant and he currently takes no medications. Blood pressure is 110/70 mm Hg, pulse is 58/min, and respirations are 26/min. His pupils are constricted, symmetric, and reactive to light. Examination reveals excessive sweating, lacrimation, and wheezing. He is incontinent of urine. Intravenous atropine is administered. Which of the following findings is most likely to persist in this patient without additional therapy?

Muscle weakness - This man has likely been exposed to a cholinesterase inhibitor (Muscarinic - diarrhea, urination, miosis, bronchospasm and bradycardia, emesis, lacrimation, salivation; nicotinic - muscle weakness, paralysis, and fasiculations) Atropine only works on muscarinic receptors, so muscle weakness due to inactivation at nicotinic receptors will remain.

A 62-year-old woman is admitted to the hospital for a living-donor kidney transplant. She has a history of end-stage kidney disease due to diabetic nephropathy and has been undergoing hemodialysis for the last 2 years. The transplant surgery is performed without complication, and the patient demonstrates good urine output afterward. To help prevent rejection, she is given a medication that inhibits the conversion of inosine monophosphate to guanosine monophosphate primarily in lymphocytes, causing reduced proliferation of activated lymphocytes. Which of the following medications is most likely being used in this patient?

Mycophenolate - this drug is an immunosuppression drug used to prevent organ transplant rejection. It functions via the inhibition of inosine 5'-monophosphate dehydrogenase (IMPDH), an enzyme in the de novo purine synthesis pathway. It has relatively specific suppression of lymphocyte suppression. Its common adverse effects include lymphopenia and gastrointestinal disturbances.

A 15-year-old girl is brought to the office due to amenorrhea. She has never menstruated, but her mother underwent menarche at age 14. The patient has no chronic medical conditions and takes no medications. She does not use tobacco, alcohol, or illicit drugs. The patient plays violin for her high school orchestra and is the captain of her junior varsity tennis team. She is not sexually active. Height is 175.3 cm (5 ft 9 in) and weight is 65 kg (143.3 lb). BMI is 21.2 kg/m2. Examination shows fully developed secondary sexual characteristics. Pelvic ultrasound shows a shortened vaginal canal with a rudimentary uterus. Which of the following is the most likely diagnosis?

Müllerian agenesis - This patient is most likely a 46,XX female with müllerian agenesis. This results in variable uterine development with no upper vagina. The patient will present with primary amenorrhea. They will have normal ovaries and regular development of secondary sexual characteristics. (Up to 50% will have a coexisting urologic abnormality)

A 75-year-old man is brought to the emergency department after he was found unresponsive at home. The patient has a history of poorly controlled hypertension. Blood pressure is 240/120 mm Hg and pulse is 104 /min. He has extensor posturing and pinpoint pupils. CT scan of the head without contrast reveals an acute pontine hemorrhage with associated mass effect. The patient dies several hours later. Autopsy reveals disruption of all pigmented neurons in the posterior rostral pons at the lateral floor of the fourth ventricle. These neurons normally produce which of the following?

Norepinephrine - The patient had a hypertensive hemorrhage in the pons at the level of the locus ceruleus, a paired pigmented brain stem nucleus located in the posterior rostral pons near the lateral floor of the fourth ventricle. This area functions are the principle site for norepinephrine synthesis in the brain. It is involved in the control of mood, arousal, sleep-wake states, cognition, and autonomic functioning. Patients with bilateral pontine hemorrhage typically present with coma due to disruption of the reticular activating system. Other features include extensor posturing and pinpoint pupils.

A 32-year-old man comes to the emergency department due to sudden onset of severe right flank pain that radiates toward the groin. He also has gross hematuria but no fever or dysuria. The patient has no significant medical conditions and has never experienced similar symptoms. He takes no medications. Temperature is 36.7 C (98.1 F), blood pressure is 120/80 mm Hg, and pulse is 88/min. The right flank is tender to palpation. There is no costovertebral angle tenderness. Imaging shows a stone in the middle of the right ureter. Which of the following is most likely to be seen on laboratory evaluation of this patient?

Normocalcemia, hypercalciuria - Hypercalciuria is the most common risk factor for calcium stones in adults. In most patients the hypercalciuria is idiopathic. Most patients remain normocalcemic due to regulation of plasma calcium levels by vitamin D and parathyroid hormone.

A 19-year-old man comes to the emergency department after being stabbed in the right arm during a fight. The patient has no chronic medical conditions. Vital signs are within normal limits. Examination of the right upper extremity shows a 4-cm laceration oriented transversely over the volar (flexor) surface of the proximal forearm. The patient is taken to the operating room for surgical repair. While exploring the wound, a nerve that courses between the flexor digitorum superficialis and profundus muscles is found to be transected. Which of the following movements is most likely to be impaired in this patient?

Opposition of the thumb - The median nerve courses between these two muscles, making it most likely to be damaged in this patient. The median nerve is most commonly injured by compression within the carpal tunnel (ie, carpal tunnel syndrome), leading to pain and numbness in the first 3 digits and lateral half of the fourth, along with weakness of thumb flexion and opposition. However, the nerve can be injured more proximally by penetrating trauma to the upper forearm, supracondylar fracture, or entrapment between the heads of pronator teres. In addition to the above symptoms, proximal injury causes decreased sensation over the thenar eminence and weakness of flexion in the wrist and second and third digits (ie, preacher or benediction hand).

A 56-year-old previously healthy man comes to the office due to decreased hearing in both ears. He reports difficulty understanding conversations in crowded rooms. His wife adds that they often argue about the volume of the television set. The patient cannot remember precisely when he first noticed hearing loss but says it has been present at least 6 months and is getting worse. For the past 12 years, he has worked in a factory where he has to shout to communicate with coworkers and has seldom worn hearing protection. An audiogram is obtained as shown in the image below. Which of the following is most likely abnormal in this patient?

Organ of Corti - The patient has high-frequency hearing loss due to chronic noise exposure. Noise-induced hearing loss results from trauma to the stereocilliated hair cells of the organ of Corti.

A 68-year-old woman comes to the office due to a burning sensation in her chest and throat for the past 2 weeks. Associated symptoms include trouble swallowing. Medical history is significant for osteoporosis, and she has no known drug allergies. The patient has smoked half a pack of cigarettes daily for 50 years, and does not use alcohol or illicit drugs. Temperature is 36.7 C (98 F), blood pressure is 110/70 mm Hg, and pulse is 70/min. BMI is 20 kg/m2. Cardiopulmonary examination shows clear lungs and normal S1 and S2. The abdomen is soft and nontender. Laboratory results are normal. It is determined that the patient's current symptoms are caused by one of her medications, which is discontinued. Her symptoms subsequently resolve. The medication responsible for this patient's presentation is also associated with which of the following side effects?

Osteonecrosis of the jaw - Medication induced esophagitis is a common adverse effect of bisphosophonates (alendronate, risedronate). These drugs are also associated with osteonecrosis of the jaw and atypical bone fractures. Other drugs that can induce esophagitis include antibiotics (tetracyclines), anti-inflammatories (aspirin and many NSAIDS), potassium chloride, and iron

A 54-year-old hospitalized man develops new-onset slurring of speech and extremity weakness. Medical history is significant for alcohol abuse. Physical examination reveals confusion, dysarthria, dysphagia, and quadriparesis. MRI of the brain shows symmetric areas of demyelination in the pons. Which of the following most likely predisposed this patient to his current condition?

Overly rapid correction of electrolyte abnormalities - This patient with a history of alcohol abuse has new-onset pseudobulbar palsy (dysarthria, dysphagia, dysphonia) and quadriparesis suggesting a brainstem insult. In association with the focal, symmetric demyelination of the pons noted on MRI, this presentation suggests osmotic demyelination syndrome (ODS), also known as central pontine myelinolysis. Other classic clinical features include a horizontal gaze paralysis, obtundation, and "locked in" syndrome, in which patients are aware but unable to move or communicate verbally.

A 78-year-old man with severe dementia is brought to the emergency department due to a daylong history of lethargy, fever, and vomiting. The patient lives in a nursing home, and his caregivers report that he has had poor oral intake over the last week. Medical history is significant for emphysema, heart failure with reduced ejection fraction, hypertension, and diabetes mellitus type 2. On physical examination, the patient is lethargic but arousable. Blood pressure is 60/30 mm Hg and pulse is 120/min and regular. The extremities are warm. Coarse rhonchi are heard over the right lower lung. After receiving several intravenous fluid boluses, the patient is given an intravenous infusion of an agent that increases peripheral vascular resistance, increases blood pressure, and decreases heart rate. Which of the following agents is most likely being described?

Phenylephrine - Phenylephrine is a selective alpha-1 agonist that results in increased systemic vascular resistance and increased blood pressure with no direct effect on the heart. The heart rate will decrease due to the baroreceptor response.

A 6-month-old boy is brought to the office by his mother out of concern that he is not developing normally. He has been feeding regularly and has had no medical problems other than a mild respiratory infection a month earlier. However, the mother says, "he doesn't seem to be as interactive as my other children were at his age." Physical examination reveals delayed developmental milestones and hypotonia. Two years later, the child is found to have involuntary movements and demonstrates a tendency to aggressively bite his own lips and fingers. Laboratory analysis shows an elevated blood uric acid level. Activity of which of the following enzymes is most likely increased as a result of this patient's condition?

Phosphoribosyl pyrophosphate amidotransferase - This patient has Lesch-Nyhan syndrome, an X linked recessive disorder, characterized by dytonia, choreoathetosis, self-mutilation, and hyperuricemia. The condition is caused by deficiency of hypoxanthine-guanine phosphoribosyltransferase (HGPRT) that normally functions in the purine salvage pathway.

A 78-year-old man was found to have a perihilar mass on screening CT scan. The patient's medical history is remarkable for chronic obstructive pulmonary disease, for which he takes albuterol inhalers as needed. He has smoked a pack of cigarettes daily for the last 50 years and does not drink alcohol or use illicit drugs. The patient is admitted for bronchoscopy and is premedicated with intramuscular atropine and becomes acutely restless, disoriented, and combative. Temperature is 38.1 C (100.5 F), blood pressure is 116/72 mm Hg, pulse is 110/min, and respirations are 15/min. Oxygen saturation is 99% on room air. On physical examination, his pupils are widely dilated and nonreactive to light. ECG shows sinus tachycardia. Which of the following agents will reverse all of this patient's signs and symptoms?

Physostigmine - Atropine is an anticholinergic, which has a major toxidrome (hot as a hare, dry as a bone, red as a beet, blind as a bat, mad as a hatter, full as a flask, fast as a fiddle). Cholinesterase inhibitors can overcome this toxicity by inhibiting the degradation of acetylchoine, thereby increasing the concentration at the synaptic cleft. Tertiary amines (physostigmine, galantamine, donepezil, rivastigmine) can reverse central and peripheral symptoms, whereas quaternary amines (neostigmine, edrophonium, pyridostigmine) can only reverse peripheral symptoms.

A 55-year-old, right-handed man comes to the emergency department due to recent onset of severe, throbbing, right-sided headache and double vision. His medical history includes poorly controlled hypertension and chronic tobacco use. Neurologic examination shows that he is awake, alert, and oriented and can follow commands. Visual fields and optic fundi are normal. The position of the right eye is down and out with ipsilateral ptosis, and the right pupil is dilated and nonreactive to both light and accommodation. Left eye examination is normal. Based on this patient's neurologic deficits, CT angiography of the head is most likely to reveal an aneurysm arising from which of the following locations in the image below?

Posterior communicating artery and the internal carotid artery junction. This patient has a right oculomotor nerve palsy, which presents with diplopia, ptosis, and down and out deviation of the ipsilateral eye.

A 65-year-old immigrant from Eastern Europe comes to the physician because of a four-week history of low-grade fever, multiple joint pains, and a well-demarcated erythematous rash on his face and trunk. The patient denies hair loss, mucosal ulceration, or photosensitivity. His serum test is positive for the presence of anti-nuclear antibody (ANA) in very high titers. The patient reports a past medical history of coronary artery disease, congestive heart failure, and "cardiac rhythm abnormalities." This patient should be specifically questioned about the intake of which of the following medications?

Procainamide -This patient is showing signs of drug-induced lupus erythematous (DILE). Anti-nuclear antibodies (ANA) and anti-histone antibodies are present >95% of the cases. Anti-dsDNA antibodies are rarely seen. Drugs exposures include hydralazine, procainamide, isoniazid, minocycline, quinidine

A 44-year-old woman comes to the clinic after several near collisions over the past month while driving at night. She also has generalized pruritus. The patient has not seen a physician for several years and has missed several scheduled appointments. She takes no medications. On physical examination, she has patchy areas of coarse, thickened skin on the extremities. Which of the following processes is most likely responsible for this patient's findings?

Prolonged biliary obstruction - This patient's symptoms are suggestive of a vitamin A deficiency. Vitamin A deficiency is characterized by night blindness, eye dryness, abnormal conjunctival keratinization (Bitot spots, foamy white plaques in the eye) and hyperkeratosis. A patient presenting with pruritus and signs of a vitamin A deficiency should be evaluated for primary biliary cholangitis, as vitamin A is a fat soluble vitamin and pruritus can be due to bile accumulation in the skin.

A 72-year-old woman comes to the office for a routine follow-up appointment. She has no symptoms and her past medical history is insignificant. Her temperature is 36.7 C (98 F), blood pressure is 110/80 mm Hg, and pulse is 76/min and irregular. ECG shows atrial fibrillation with no ischemic changes. Anticoagulation therapy with warfarin is initiated for stroke prevention. Two days later, the patient is hospitalized with severe skin and subcutaneous fat necrosis. Drug effects on which of the following processes are most likely responsible for this patient's skin findings?

Protein C activity - This patient has warfarin-induced skin necrosis, a rare but important complication. It is thought to be due to a transient hypercoagulable state that can occur during the first few days of therapy. On top of inhibition of the gamma-carboxylation of clotting factors II, VII, IX, and X, warfarin also decreases carboxylation of proteins C and S, which normally exert an anticoagulant effect. Protein C has a short half-life, so its anticoagulant activity is reduced quickly when warfarin therapy is initiated. Often warfarin will be started with heparin initially to decrease this skin induced necrosis.

A 54-year-old postmenopausal woman comes to the office due to increasing vaginal pressure for the past 3 months. The patient has had no vaginal bleeding or abnormal discharge. BMI is 18 kg/m2. Vital signs are normal. On speculum examination, the anterior vaginal wall bulges with the Valsalva maneuver. On bimanual examination, the uterus is small and mobile. Which of the following is the most likely underlying cause of this patient's presentation?

Pudendal nerve injury - This patient is probably experiencing pelvic organ prolapse, the herniation of pelvic organs into the vagina due to pelvic floor weakness. An injury to the pudendal nerve is most common due to a vaginal delivery. It is usually due to slow degeneration of the pelvic floor.

A 32-year-old man is hospitalized due to nausea, vomiting, and severe abdominal pain. He has a history of heavy alcohol use and was admitted to the hospital for acute pancreatitis a year ago. He has continued to drink alcohol since his last hospitalization and had a party last weekend, during which he consumed an entire 750-mL bottle of liquor. Temperature is 38.3 C (100.9 F), blood pressure is 110/80 mm Hg, pulse is 104/min, and respirations are 20/min. Abdominal examination is notable for marked tenderness in the epigastric region. Serum lipase is 2,392 U/L. The patient is admitted and started on intravenous fluids, analgesics, and antiemetics with subsequent improvement in his symptoms. His vital signs remain stable with adequate urine output. However, on the second day of hospitalization, the patient develops progressive shortness of breath and hypoxemia. Chest x-ray reveals new bilateral opacities. Which of the following parameters is most likely to be normal in this patient?

Pulmonary capillary wedge pressure - Noncardiogenic pulmonary edema (eg, ARDS, high-altitude pulmonary edema) can be distinguished from cardiogenic pulmonary edema (eg, decompensated left ventricular failure, volume overload) by a normal pulmonary capillary wedge pressure (6-12 mm Hg). Other distinguishing features include the clinical history (pneumonia/sepsis or pancreatitis suggests ARDS) and physical exam (peripheral edema and jugular venous distension suggest a cardiogenic source).

A 44-year-old man comes to the office due to several weeks of difficulty walking and frequent falls. He has had no headache or visual impairment but reports episodes of sharp, stabbing pain in his extremities. The patient has HIV acquired from unprotected sexual contact and is nonadherent with treatments. Temperature is 37 C (98.6 F). Neurologic examination shows 5/5 motor power in all extremities. Deep-tendon reflexes are absent at the knee and ankle bilaterally. Proprioception and vibration sensation are reduced throughout the lower extremities. He has a wide-based gait and a positive Romberg sign. Which of the following diagnostic findings is most likely associated with this patient's current symptoms?

Reactive VDRL test on CSF samples - This patients presentation is suspicious for tabes doralis, a late form of neurosyphilis. It is characterized by slow degradation of nerve fibers in the dorsal column of the spinal cord and dorsal nerve roots, which leads to diminished vibratory, proprioception and discriminative touch sensation. Common findings include sensory ataxia, absent deep-tendon reflexes, and lancinating pains.

A 42-year-old woman comes to the office due to frequent episodes of burning in her chest and small amounts of regurgitation after meals and at nighttime. Medical history includes hypertension. Vital signs are within normal limits. BMI is 30 kg/m2. There is no abdominal tenderness and the remainder of the physical examination is normal. The patient shows the clinician an over-the-counter antacid that she has been taking to relieve her symptoms. The preparation contains a combination of magnesium and aluminum hydroxide. Which of the following is the most likely rationale for combining both mineral salts in this antacid preparation?

Reduce adverse affects - Magnesium salts and aluminum hydroxide are weak alkali mineral salts that temporarily increase the gastric pH by neutralizing hydrochloric acid, helping to relieve gastroesophogeal reflux symptoms. Aluminum hydroxide tends to cause constipation, which magnesium salts cause osmotic diarrhea, so the combination offsets the other side effects.

A 21-year-old man comes to the emergency department due to several days of dizziness and extreme fatigue. The patient has been drinking a lot of water and urinating more than usual for the past 2 weeks. He developed nausea and vomiting a day ago and has had difficulty keeping food and liquids down since then. The patient has no other medical conditions and takes no medications. Blood pressure is 90/60 mm Hg, pulse is 120/min, and respirations are 20/min. Urinalysis is positive for glucose and ketones. The patient is given intravenous fluids and an intravenous infusion of recombinant human insulin without any amino acid modification. After his condition is stabilized, the insulin infusion is stopped, and the same insulin formulation is administered subcutaneously. Which of the following drug concentration curves is most likely to be obtained after the subcutaneous insulin injection?

Regular insulin given subcutaneously begins to work in 30 minutes and peaks in 2-4 hours.

A 27-year-old man comes to the emergency department due to 2 weeks of low-grade fever, malaise, and loss of appetite. The patient has no prior medical conditions and takes no medications. He does not smoke cigarettes or drink alcohol but regularly injects illicit drugs. The patient is sexually active and uses condoms inconsistently. Physical examination reveals scleral icterus. A smooth and mildly tender liver edge is palpable 4 cm below the right costal margin. Laboratory results are as follows: Liver function studies Total bilirubin 4.3 mg/dL Alkaline phosphatase 220 U/L Aspartate aminotransferase (SGOT) 1,600 U/L Alanine aminotransferase (SGPT) 2,100 U/L Hepatitis panel Hepatitis A virus antibody, IgM negative Hepatitis B surface antibody (HBsAb) negative Hepatitis B core antibody (HBcAb), IgM positive Hepatitis B surface antigen (HBsAg) positive Hepatitis C virus antibody negative Which of the following is an essential step in the replication cycle of the virus infecting this patient?

Reverse transcription of template RNA to viral DNA - This patient has acute hepatitis B virus. HBV is an encapsulated virus with a partially double-stranded DNA genome. It binds to and enters hepatocytes using a host cell bile salt transporter and then sheds its coat in the host cytoplasm. The partially double-stranded genome is "repaired" and transcribed by host cell machinery, yielding a + sense single-stranded mRNA template. This serves as a pre-genome for future progeny; it is converted by viral reverse transcriptase into the partially double-stranded DNA genome of budding viral particles (which are subsequently released from the cell). The mRNA template is also translated by host ribosomes into the proteins that compose the virus.

A 38-year-old man is undergoing treatment for acute myeloid leukemia. Initially, he had fevers and reported being lethargic and fatigued all the time. However, of late, he has noted increasingly severe right-sided headaches. He describes the pain to be throbbing in nature and located behind his right eye. He also has associated nasal stuffiness. The patient has no prior history of allergies. Physical examination reveals right-sided proptosis and periorbital tenderness. Biopsy of the right maxillary sinus mucosa is shown, showing nonseptate hyphae with right-angle branching. Which of the following is the most likely cause of this patient's condition?

Rhizopus species - It is trasmitted by spore inhalation and causes mucormycosis, which is strongly associated with diabetic ketoacidosis. Immunosuppressed individuals are also high risk. Mucormycosis tends to affect the paranasal sinuses.

In an experiment, laboratory animals are subjected to a toxic insult that specifically targets the protein kinesin. Which of the following is most likely to be absent from tissues on histological examination as a result?

Secretory vesicles in nerve terminals - Kinesin is a microtubule associated motor protein whose function is anterograde transport of intracellular vesicles and organelles toward the plus (rapidly growing) ends of microtubules. Kinesin uses energy derived from ATP hydrolysis to move along the microtubule. The plus ends of microtubules typically lie at sites distant from the nucleus; thus, in neurons, kinesin carries vesicles and organelles away from the cell body and toward the nerve terminal. A targeted toxic insult to this protein would be expected to impair anterograde transport resulting in a deficiency of synaptic vesicles at the nerve terminal.

A 7-year-old boy is brought to the office due to sudden onset of facial swelling 2 hours ago. He has had no itching or pain other than a sore throat over the last 2 days, for which he has taken acetaminophen. The patient has had similar episodes of facial swelling that resolved spontaneously after a few days. Temperature is 37 C (98.6 F), blood pressure is 100/78 mm Hg, pulse is 95/min, and respirations are 24/min. Examination shows nonpitting edema of the cheeks, lips, and tongue; there is no tenderness or erythema. Which of the following studies is most likely to be abnormal?

Serum C4 level - This recurrent facial swelling is most likely due to hereditary angioedema, which is characterized by a deficiency or dysfunction of C1 inhibitor. Poor C1 inhibitor function leads to elevated bradykinin, resulting in edema. These attacks are usually precipitated by minor trauma or emotional distress. Low C4 is characteristic because in the absence of C1 inhibitor, unregulated activation of C1 leads to excess activated C1 and in turn unchecked cleavage of C4.

A 46-year-old man is diagnosed with resistant hypertension. He had a comprehensive workup for secondary hypertension, which was unrevealing. The patient has tried multiple antihypertensive medications without significant effect. Past medical history is otherwise unremarkable. He agrees to enroll in a trial of a new long-acting medication that causes selective direct relaxation of the smooth muscle of arterioles but does not affect the veins. Which of the following adverse effects is most likely to be caused by the drug during the clinical trial?

Sodium and fluid retention - Selective arteriolar vasodilators (eg, hydralazine, minoxidil) lower blood pressure by reducing systemic vascular resistance. However, this effect is limited by subsequent stimulation of baroreceptors with resulting reflex sympathetic activation. This leads to increased heart rate, contractility, and cardiac output. In addition, sympathetic stimulation of the renin-angiotensin-aldosterone axis results in sodium and fluid retention with peripheral edema. These effects offset much of the blood pressure lowering effect of these drugs and limit their long-term efficacy. These agents are rarely used as monotherapy for chronic management of hypertension. However, they are useful acutely for patients with severely elevated blood pressure. They can also be given in combination with sympatholytics and diuretics to mitigate the side effects and provide synergistic blood pressure lowering in patients with resistant hypertension.

A newborn is taken to the neonatal intensive care unit after delivery due to respiratory distress. The neonate was born full-term, and there were no significant complications during pregnancy or delivery. Once the patient is stabilized, a detailed physical examination is performed. The neonate has an underdeveloped mandible and hypoplastic zygomatic bones. Genetic testing confirms a gene mutation that results in abnormal development of the first and second pharyngeal arches. Which of the following structures is also likely abnormal in this patient?

Stapes - The first pharyngeal arch is associated with the trigeminal nerve V. Its bony derivatives include the maxilla, zygoma, mandible, incus, and malleus and its muscles include the muscles of mastication. The second pharyngeal arch is associated with the facial nerve VII and gives rise to the styloid process of the temporal bone, lesser horn of the hyoid, and stapes and muscular derivatives include the muscles of facial expression. This child presents with Treacher-Collins Syndrome, a genetic disorder of an abnormality of the first and second pharyngeal arches.

A 62-year-old man comes to the office for the evaluation of jaundice. Medical history is significant for uncontrolled type 2 diabetes mellitus and morbid obesity. He does not use tobacco, alcohol, or illicit drugs. There is no family history of liver disease. Vital signs are within normal limits. BMI is 47 kg/m2. Mild scleral icterus is present. Heart and lung sounds are normal. The abdomen is nontender and nondistended. Trace bilateral lower extremity edema is present. Laboratory studies reveal elevated transaminases. Which of the following cells is directly responsible for the histologic findings indicated?

Stellate (Ito) cells - This patient has hepatic cirrhosis, likely due to long-standing non-alcoholic fatty liver disease. Cirrhosis can be visualized histologically as thickened collagenous bands, which stain blue separating clusters of hepatocytes. Fat infiltration is also commonly seen. In response to chronic injury, stellate cells in the space of Disse are activated and transform into myofibroblasts, which produce collagen in large quantities.

A 48-year-old woman with metastatic cervical cancer is brought to the emergency department due to worsening lethargy for the past several days. A year ago, the patient was diagnosed with cervical cancer, which has progressed despite chemotherapy treatment and metastasized to the liverand lungs. She has also developed bilateral hydronephrosis and renal failure due to ureteral compression by the tumor. The patient is receiving palliative care, and her pain has been adequately controlled with a stable dose of oral morphine. Temperature is 36.7 C (98 F), blood pressure is 110/62 mm Hg, pulse is 92/min, and respirations are 10/min. On physical examination, the patient is somnolent and responds to painful stimuli only. The pupils are small and sluggish to react. The lungs are clear to auscultation, and heart sounds are normal. The abdomen is soft and nondistended. Arterial blood gas analysis shows respiratory acidosis. Which of the following medication-related events most likely precipitated her current condition?

Stress-induced cardioinhibitory and vasodepressor response - This is likely vasovagal syncope, which involves a pain-induced or an emotional distress-induced cardioinhibitory and vasodepressor response.

A 24-year-old man comes to the office due to severe pain in the posterior aspect of the right foot that interferes with walking. For the last several months, the patient has also experienced a dull backache and morning stiffness that is relieved by over-the-counter ibuprofen. Family history is insignificant, and he does not use tobacco, alcohol, or illicit drugs. Which of the following is the most probable location of the pathologic process responsible for this patient's foot pain?

Tendon insertion site - this patient has inflammatory bak pain which is likely due to ankylosing spondylitis (AS). There is an increase in inflammatory cytokines (primarily tumor necrosis factor-alpha and IL-17) which cause migration of immune cells to the skeleton. Skeletal manifestations are most prominent in areas of mechanical stress, primarily at the insertion of tendons, ligaments, and joint capsules. This is Achilles tendinitis (enthesitis)

A 66-year-old woman with rheumatoid arthritis and atrial fibrillation is scheduled for an elective right total knee arthroplasty in a tertiary care hospital. The patient speaks mostly Spanish and understands limited English.Informed consent is obtained during a preoperative clinic visit using a certified interpreter. On the day of the procedure, the operating room is running behind schedule, and the patient's procedure is delayed by 3 hours. When the surgeon and circulating nurse visit the patient to confirm the site and side of the procedure, the certified interpreter is in the emergency department assisting with another patient and will be unavailable for an hour. The patient talks rapidly and has several questions for the team. The surgeon has taken a 3-week course in medical Spanish, and the nurse can speak short sentences in Spanish. The surgeon converses with the patient in a mixture of Spanish and English. The patient nods frequently as the surgeon speaks. Which of the following best describes the patient-provider communication in this encounter?

The surgeon's actions are inappropriate as they potentially violate the patient's rights - Legally patients with limited English proficiency have a legal right to healthcare access in their preferred language with professional interpretation.

As part of a research study investigating enzymatic activity in both normal and diseased liver tissue, hepatocytes are isolated from biopsy samples obtained from patients undergoing routine care at a local tertiary referral center. The cells are homogenized and centrifuged to remove membrane components and organelles. Following subsequent rounds of centrifugation, the remaining supernatant contains only cytosol and cytosolic proteins. Activity of which of the following enzymes will most likely be detectable in the supernatant of healthy liver cells?

Transketolase - The cytosol is home to enzymes necessary for glycolysis, fatty acid synthesis, and the pentose phosphate pathway. Transketolase is an enzyme of the pentose phosphate pathway that uses thiamine (vitamin B1) as a cofactor to shuttle 2-carbon fragments between sugar molecules. Other processes such as heme synthesis, the urea cycle, and gluconeogenesis rely on a complex interplay between the mitochondria and cytosol to function optimally; enzymes present in both cellular compartments are required for these metabolic pathways

A 74-year-old man comes to the office for a follow-up visit for hypertension. His last visit was a year ago. He has no chest pain, shortness of breath, leg swelling, or dizziness. The patient is compliant with his medications and can tolerate a moderate level of physical activity. Blood pressure is 145/75 mm Hg and pulse is 67/min and regular. Auscultation findings at the base of the heart are depicted in the exhibit below as an extra sound between S1 and S2. Which of the following is the most likely cause of this patient's physical findings?

Valvular calcification- The diagram above depicts a crescendo-decrescendo systolic murmur peaking in midsystole, characteristic of aortic or pulmonic stenosis. the murmur of valvular AS is best heard over the right second interspace.

A 23-year-old man with a history of type 1 diabetes mellitus is brought to the emergency department due to confusion and weakness. His symptoms began 2 days ago after he started having mild diarrhea. He has missed several doses of insulin because his appetite has been poor. On examination, his breath has a fruity odor. This patient is most likely to demonstrate which of the following urine chemistry patterns?

pH: ↓ HCO3-: ↓ H2PO4-: ↑ This patient will show increased HCO3- reabsorption, increased H+ secretion, and increased acid buffer excretion due to his diabetic ketoacidosis.

A 38-year-old man comes to the emergency department with severe abdominal pain and vomiting. The pain radiates to his back and improves by bending forward. Physical examination shows tenderness over the epigastrium and decreased bowel sounds. The patient is admitted to the hospital and treated with intravenous fluids and pain medication, but his condition fails to improve. An abdominal CT scan reveals diffuse pancreatic enlargement with areas of necrosis. The inappropriate activation of which of the following most likely initiated this patient's condition?

Trypsinogen - The pathogenesis of acute pancreatitis begins with either a toxic or an ischemic injury to the acinar cells that leads to premature activation of trypsin inside the pancreatic acini. Trypsin then activates the other proteolytic enzymes and starts a self-sustaining cycle of pancreatic inflammation and autodigestion with further release of digestive enzymes. In severe cases, this process can result in necrotizing pancreatitis, which is characterized by gross areas of parenchymal necrosis with a high propensity for secondary bacterial infection.

A 55-year-old Caucasian male is found on colonoscopy to have a solitary mass in his sigmoid colon. Biopsy is consistent with colon cancer, and surgery is scheduled. Which of the following features would carry the worst prognosis in this patient?

Tumor penetration into the muscularis propria - The expansion determines the stage, which is more important than grade in determining prognosis.

A 15-year-old boy is brought to the office because of 3 months of sudden, brief jerking movements involving both arms. The episodes usually occur shortly after awakening in the morning and are aggravated by sleep deprivation. The movements are neither suppressible nor preceded by an urge to make a movement. The patient has never lost consciousness and has no known medical conditions. Family history includes a seizure disorder in his uncle. Vital signs are within normal limits. Physical examination shows normal development with no abnormalities. Which of the following is the best initial treatment for this patient?

Valproic acid - This patient likely has myoclonic seizures due to juvenile myoclonic epilepsy (JME), a form of generalized onset epilepsy. It presents in otherwise heathy adolescents with myoclonic seizures. It usually involved both upper extremities with preserved consciousness and is more prominent in the first hour of awakening and provoked by sleep deprivation. Treatment should include a broad-spectrum anticonvulsant (valproic acid, levetiracetam) to treat generalized and focal seizures.

A 56-year-old man is brought to the emergency department due to weakness and difficulty speaking. He has a history of hypertension, cigarette smoking, and polysubstance use. The patient experienced a headache while smoking crack cocaine the previous night. This morning, he was found on the bed unable to move or speak. On physical examination, the patient is awake with normal pupillary responses. Vertical eye movements are intact, but horizontal eye movements are impaired. There is loss of motor strength in the upper and lower extremities bilaterally. Ischemic infarction from cocaine-induced vasospasm is suspected. A lesion involving which of the brain regions is the most likely cause of this patient's current condition?

Ventral pons - This patient is displaying locked-in syndrome (quadriplegia, loss of speech, absent horizontal eye movements, preserved consciousness, and preserved sensation), which can be caused by ischemic injury to the bilateral ventral pons.

A 57-year-old unconscious man is brought to the emergency department by his wife. Thirty minutes prior, the patient developed severe chest pain and asked to be taken to the hospital. On the way, he became unresponsive. According to his wife, the patient had been having intermittent, short-lived episodes of chest pain over the past several days. The patient's medical history is significant for hypertension, type 2 diabetes mellitus, and hypercholesterolemia. He is not adherent with medications or follow-up. In the emergency department, the patient has no spontaneous respirations or circulation and is pronounced dead. An autopsy examination is performed, and a cross-section of the mid-right coronary artery is shown in the exhibit. Which of the following is the most likely immediate cause of death in this patient?

Ventricular fibrillation - Acute MI is a common cause of sudden cardiac death (SCD) (the abrupt cessation of organized cardiac activity leading to hemodynamic collapse and inadequate tissue perfusion). SCD due to acute MI usually results from a malignant ventricular arrhythmia (ie, ventricular fibrillation or ventricular tachycardia degenerating to ventricular fibrillation) that is triggered by electrical instability in the ischemic myocardium.

A 66-year-old man comes to the clinic due to numbness and tingling in his hands and feet that developed over the last few weeks. The patient was healthy until he was diagnosed with pulmonary tuberculosis 3 months ago. He takes only isoniazid and rifampin. Temperature is 37.1 C (98.8 F), blood pressure is 126/80 mm Hg, pulse is 90/min, and respirations are 16/min. Cardiopulmonary examination is unremarkable. Neurologic examination shows ataxia and decreased pain sensation in the distal extremities. This patient's condition is most likely due to which of the following?

Vitamin-dependent neurotransmitter deficiency - This patient with TB has developed paresthesia and ataxia consistent with isoniazid-induced peripheral neuropathy. It is structurally similar to pyridoxine (B6), a cofactor for enzymes involved in synthesis of certain neurotransmitters. It should be treated with supplemental pyridoxine.

A researcher is investigating the structure of the sarcomere using skeletal muscle obtained from an experimental animal. He develops monoclonal antibodies directed against a specific skeletal muscle protein and finds that these antibodies disrupt the binding of actin to structural support elements within the sarcomere. Electron microscopy of the sarcomere is shown in the image below. Which of the following labeled regions do these antibodies most likely bind?

Z-Line - a single sarcomere is defined as the distance between two Z lines. Thin (actin) filaments are bound to structural proteins at the Z line.

A 27-year-old man comes to the office due to recurrent episodes of muscle weakness. He has no other significant past medical history. The patient's weight has been stable for the past few years, and his current BMI is 23 kg/m2. His blood pressure is 190/110 mm Hg supine and 195/110 mm Hg standing. His heart rate is 70/min supine and 72/min standing. The rest of the physical examination is unremarkable. Laboratory evaluation shows very low plasma renin activity. Overactivity of which of the following structures is most likely responsible for this patient's symptoms?

Zona glomerulosa of the adrenals - This patient presents with features consistent with primary hyperaldosteronism. It is usually due to increased secretion of mineralcortacoids from bilateral nodular hyperplasia of the adrenal zona glomerulosa or aldosterone-producing adrenal adenoma. (remember GFR)

A 66-year-old man comes to the emergency department due to several episodes of lightheadedness and a recent fall. The symptoms are especially pronounced in the morning when he gets up from bed. The patient takes medications for hypertension, stable angina pectoris, gout, and benign prostate hyperplasia. He also has osteoarthritis of the right knee and takes occasional nonsteroidal anti-inflammatory drugs. His blood pressure is 120/80 mm Hg and pulse is 70/min in the supine position, and 90/60 mm Hg and 87/min on standing, respectively. Cardiopulmonary examination is normal. Blockade of which of the following receptors is most likely contributing to this patient's current condition?

α1-adrenergic - This patient has orthostatic hypotension. The α1-adrenergic antagonists (terazosin, doxazosin) are commonly used for symptomatic benign prostate hyperplasia and can be associated with orthostatic hypotension, as they can cause a decrease in systemic vascular resistance.

A 56-year-old woman with unknown medical history is brought to the emergency department in an unresponsive state. Paramedics were called for sudden-onset, left-sided weakness, and when they arrived at the patient's house she was having a generalized tonic-clonic seizure. Antiseizure medications were given and the patient was subsequently intubated. Evaluation shows a comatose woman with left hemiplegia. Neuroimaging reveals a large infarction in the right middle cerebral artery territory with cerebral edema. Despite aggressive interventions, the patient dies from brain herniation. Autopsy reveals multiple small, nondestructive masses attached to the edges of the mitral valve leaflets. Microscopy shows that these masses are composed of platelet-rich thrombi, but no organisms are present. Which of the following is most closely associated with this patient's heart valve findings?

Advanced malignancy - Sterile platelet-rich thrombi are found attached to the mitral valve leaflets, which is characteristic of nonbacterial thrombotic endocarditis. The pathogenesis of NBTE is thought to begin with valvular endothelial injury caused by circulating inflammatory cytokines, which trigger platelet deposition in the presence of an underlying hypercoagulable state. NBTE is most commonly associated with advanced malignancy and systemic lupus. It commonly presents with systemic embolization.

A 60-year-old man comes to the emergency department with dizziness and palpitations. He woke up this morning with an intermittent pounding sensation in his chest. The patient has occasionally experienced a similar sensation in the past and did not think it was serious. Later, while mowing the lawn, he became lightheaded and had to sit down to avoid passing out. After receiving the appropriate workup, the patient is diagnosed with a cardiac arrhythmia and started on a medication known to prolong the QT interval. However, the drug is associated with a lower incidence of torsade de pointes than other QT-prolonging agents. Which of the following medications was most likely used in this patient?

Amiodarone - This drug is a class III anti-arrhythmic drug used for the management of a variety of supraventricular and ventricular arrhythmias. Class III drugs predominantly block potassium channels and inhibit the outward potassium currents during phase 3 of the cardiac action potential, prolonging repolarization and total action potential duration. Amiodarone, while prolonging the QT interval, has very little risk of inducing torsades de pointes.

A 77-year-old man comes to the office after he was found to have high blood pressure during a health fair. The patient denies any medical problems and feels proud that he has not needed to see a doctor for many years. His blood pressure is 170/70 mm Hg and pulse is 74/min. Other physical examination findings are normal. The patient is started on a medication to treat his hypertension. During his follow-up visit 3 weeks later, he reports bilateral leg swelling. He has no chest pain, shortness of breath, or abdominal symptoms. His blood pressure is 135/65 mm Hg and pulse is 80/min. Cardiopulmonary examination is normal, but there is bilateral, symmetrical, 2+ pitting lower extremity edema. Laboratory studies show serum creatinine of 0.8 mg/dL, and urinalysis is negative for proteinuria. Which of the following was most likely prescribed to treat this patient's hypertension?

Amlodipine - Dihydropyridine calcium channel blockers (amlodipine, nifedipine) are effective antihypertensives, but can have side effects such as headache, flushing, dizziness, and peripheral edema

A previously healthy 24-year-old man is hospitalized due to 3 days of fever, chills, myalgia, and severe headache. The symptoms started on the last day of his camping trip in the Appalachian Mountains and have rapidly progressed. Temperature is 38.9 C (102 F), blood pressure is 100/56 mm Hg, and pulse is 120/min. On examination, the patient is lethargic and confused. There is no neck rigidity. The lungs are clear on auscultation and heart sounds are normal. The abdomen is soft and nontender. There is a prominent erythematous maculopapular rash on the distal forearms. No lymphadenopathy is present. The patient should be immediately initiated on a medication that targets which of the following?

Bacterial protein synthesis - this patient likely is suffering from Rocky Mountain Spotted Fever (RMSF) caused by Rickettsia rickettsii. This infection is treated first line with doxycycline which inhibits protein synthesis by binding the 30s bacterial ribosomal subunit.

A 14-year-old girl is brought to the office for a routine physical examination. The patient will be starting her freshman year of high school soon. She plays the clarinet in the school band but does not play any sports. The patient says that she is not sexually active and does not use tobacco, alcohol, or illicit drugs. Height is at the 5th percentile, and weight is at the 25th percentile. Her temperature is 36.7 C (98 F), blood pressure is 120/80 mm Hg, pulse is 88/min, and respirations are 16/min. Physical examination shows a short and thick neck, a broad chest, and shortened fourth metacarpals bilaterally. A murmur is heard on cardiac auscultation. Which of the following would most likely be seen in an echocardiogram?

Bicuspid aortic valve - This patient's short stature, short and thick neck, broad chest, and shortened fourth metacarpals are characteristic of Turner syndrome. Bicuspid aortic valve is the most common congenital abnormality in TS. A non-stenotic bicuspid aortic valve presents with an aortic ejection sound, which is an early systolic, high-frequency click heard over the right second interspace. These valves are at risk for stenosis, insufficiency, and infection.

A patient is suspected of having an inherited disorder. Pedigree analysis shows a pattern of autosomal recessive inheritance. The patient most likely has which of the following conditions?

Classic galactosemia - Classic galactosemia is the most common and most severe of the galactosemic disorders. It is am autosomal recessive disorder leading to complete enzymatic absence of galactose-1-phosphate uridyl transferase. Newborns present within days of birth with jaundice, vomiting, and hepatomegaly. In general, most enzyme deficiency conditions follow an autosomal recessive inheritance pattern whereas diseases due to defective noncatalytic proteins follow and AD pattern.

A 4-year-old Caucasian boy is evaluated for difficulty walking. Past medical history includes frequent respiratory infections. Cultured cells from this patient demonstrate a high rate of radiation-induced genetic mutation. This patient is most likely to experience which of the following?

Cerebellar atrophy - Ataxia-telangiectasia is an autosomal recessive disorder. Cerebellar atrophy leads to the ataxia that occurs in the first years of life. They also have a severe immunodeficiency with repeated sinopulmonary infections. There is a significantly increased risk of cancer.

A 64-year-old man comes to the office due to 3 months of constant, dull, low back pain that is worse with movement. The patient has also had intermittent episodes of difficult urination over the last year. He has no other medical conditions and takes no medications. The patient smoked a pack of cigarettes daily for 15 years but quit 10 years ago. Vital signs are within normal limits. On physical examination, the lungs are clear and heart sounds are normal. The abdomen is soft and nontender with no hepatosplenomegaly. Digital rectal examination shows a diffusely enlarged prostate with no nodules. Laboratory testing reveals a serum creatinine level of 1.9 mg/dL. Urine dipstick testing is within normal limits, but 24-hour urinary protein excretion is elevated. Urine microscopy shows waxy, laminated casts. Imaging of the spine demonstrates diffuse osteopenia and multiple irregular, radiolucent lesions in the thoracic and lumbar vertebrae. Which of the following histopathologic findings is most likely associated with the development of this patient's bone lesions?

Clusters of mature plasma cells and plasmablasts - An older patient with back pain and several irregularly-shaped vertebral lesions raises a strong suspicion for metastatic disease. To determine the most likely underlying tumor is assisted by looking at if the lesion is sclerotic (osteoblastic) or radiolucent (osteolytic). Multiple myeloma, a clonal plasma cell malignancy, is one of the most common causes of radiolucent bone lesions in adults. Myeloma cells also produce excessive monoclonal immunoglobulin that can clog the renal tubules and subsequently cause light-chain nephropathy.

A 75-year-old man is hospitalized due to respiratory distress. The patient developed fever, cough, and muscle aches 4 days prior to admission. He is otherwise healthy and has no chronic medical conditions. The patient has received all recommended vaccinations, including a yearly flu vaccine. Temperature is 39 C (102.2 F), blood pressure is 110/65 mm Hg, pulse is 115/min, and respirations are 29/min. Chest x-ray shows bilateral infiltrates. Reverse transcriptase PCR of a specimen from a nasopharyngeal swab reveals a strain of influenza A virus that was included in the seasonal trivalent flu vaccine. The patient lives with his 50-year-old son, who received the same vaccine but did not develop the infection. Which of the following factors most likely increased this patient's risk of vaccine failure compared with that of his son?

Decreased production of naive B lymphocytes - The risk of primary vaccine failure is increased in patients with altered immune function, including those with atopic disorders, steroid use, or age-related immune decline (ex. immunosenescence). The loss of telomere length during normal aging particularly affects rapidly dividing immune cells, resulting in decreased production of naive B and T lymphocytes. Aging is also associated with chronic low-grade inflammation that causes much of the remaining naive lymphocyte pool to differentiate into memory lymphocytes against previously encountered antigens.

A 22-year-old man comes to the emergency department with abdominal pain and vomiting. His symptoms began acutely 24 hours ago with nausea and decreased appetite. His medical history is unremarkable. He drinks 3-4 beers awake and uses no tobacco or illicit drugs. BMI is 72 kg/m2. Physical exam is notable for moderate epigastric tenderness with decreased bowel sounds but no rebound tenderness, guarding, or rigidity. Fasting lab studies show a serum triglyceride concentration of 1500mg/dL (normal <200). Once the patient is stabilized, which medication would be most helpful in preventing recurrences of his condition?

Fenofibrate - The patient likely has acute pancreatitis due to severe hypertriglyceridemia. Fibrates (ex. fenofibrate, gemfibrozil) work by activating peroxisome proliferator-activated receptor-alpha (PPAR-a) which increases the synthesis of lipoprotein lipase. (Decrease triglycerides, minor increase in HDL, decrease VLDL synthesis, can have muscle toxicity or cause gallstones)

A 32-year-old man is brought to the emergency department due to weakness and tingling that started in his feet and is now at his knees. A week ago, the patient noticed that his legs felt weak while climbing stairs. Since then, the weakness has progressively worsened every day. Today, he fell twice when attempting to walk. Medical history is insignificant, except for a mild respiratory infection 2 weeks ago that resolved spontaneously. The patient does not use tobacco, alcohol, or illicit drugs. He takes no medications. Vital signs are within normal limits. Physical examination shows marked weakness in the distal muscles and moderate weakness in the proximal muscles in both legs. Deep tendon reflexes are absent at the knees. This patient's condition is most likely associated with which of the following?

Endoneurial inflammatory infiltration - This patient has ascending symmetric weakness following a febrile illness, that is likely due to Guillain-Barre syndrome. It is an acute, immune-mediated, demyelinating polyneuropathy. Histology demonstrated an inflammatory infiltrate located within the endoneurium. Macrophages strip the myeline sheath from the axon, and lipid laden macrophages are seen following the engulfment of myelin.

A 16-year-old obese primigravida girl comes to the emergency department in active labor. She has not received prenatal care or taken prenatal vitamins. A fetal heartbeat is undetectable and she delivers a stillborn boy. Examination of the stillborn shows several dysmorphic features including closely set eyes and a midline mass consistent with a proboscis. A fetal autopsy reveals fused cerebral hemispheres with an absent forebrain fissure and a single intracranial ventricle. What is the most likely mechanism for these findings?

Field Defect - holoprosencephaly which is a spectrum of abnormalities in the face and brain due to incomplete division of the forebrain, which normally occurs around 5 weeks gestation. It is a developmental field defect, which is when an initial embryonic disturbance leads to multiple malformations by disrupting the development of adjacent tissues and structures in a region.

An 18-year-old girl with a history of type 1 diabetes mellitus comes to the ER after several hours of nausea, vomiting, and abdominal pain. She has not taking insulin in the past two days. Physical exam shows a breath with fruity odor and dry mucous membranes. Labs show a blood glucose of 42mg/dL and high anion gap metabolic acidosis. Urinalysis shows ketones. It is determined that the patient's insulin deficiency is causing increased production of gluconeogenic precursors that are subsequently converted to glucose in the liver. Which of the following enzymes is most likely to provide these precursor substrates?

Glycerol kinase - the patient is in diabetic ketoacidosis and is experiencing increased triglyceride breakdown. Triglycerides are metabolized to FFAs and glycerol by hormone-sensitive lipase. Glycerol is the secreted into circulation where in the liver it is phosphorylated to glycerol-3-phosphate by glycerol kinase. This can go on to become glucose through gluconeogenesis.

A 2-month-old girl is brought to the clinic due to perinatal hepatitis B exposure. Her mother is age 22 and immigrated to the United States shortly after giving birth. The pregnancy was complicated by maternal hepatitis B infection. However, the mother received no prenatal or perinatal care and was not diagnosed with chronic hepatitis B until the immigration process. Maternal HBsAg, HBeAg, and anti-HBc are all positive; anti-HBs is negative. The infant has not been evaluated previously and has not received any immunizations or other medications. She is exclusively breast fed and has been growing normally. Vital signs are normal and physical examination is unremarkable. No hepatomegaly or jaundice is seen. The infant's laboratory results will most likely show which of the following?

HBeAg - This infant was exposed to hepatitis B virus during delivery. Mother-to-child transmission most commonly occurs during delivery but transplancental infection is possible. Maternal viral load and HBeAg status are the strongest risk factors for infant infection. Mothers who are HBeAg + have a >90% chance of passing it to the baby. Infants will then have high loads of HBeAg and enter an immune-tolerant phase of chronic HBV infection where they are typically asymptomatic. Over time they are at high risk to progress to cirrhosis and hepatocellular carcinoma, but this can be possibly prevented with administration of the HBV vaccine and immunoglobulin as soon as possible after delivery.

A 29-year-old woman comes to the office with malaise, fever, and extremely painful genital lesions. She has never had such symptoms before. She is sexually active with a new partner. On examination, the patient has several tiny, thin-walled vesicles on an erythematous base and exquisitely tender, shallow ulcers on the labia. Tender inguinal lymphadenopathy is present. Tzanck smear of the genital lesions shows multinucleated giant cells. A drug is prescribed that can be used to manage this acute episode and prevent future episodes of this patient's condition. This drug most likely works by which of the following mechanisms?

Incorporation into newly replicating viral DNA - This patients presentation is consistent with a primary genital herpes simplex virus infection, likely due to HSV-2. Antiviral drugs for the primary treatment of genital herpes include acyclovir, valacylovir, and famciclovir. In infected host cells, acyclovir (a nucleoside analog) is converted into acyclovir monophosphate principally via virus-encoded thymidine kinase. It is then transformed into acyclovir triphosphate with competes for incorporation into the viral DNA. When incorporated, viral DNA synthesis is terminated. It does not have a large impact on uninfected cells.

A 71-year-old man comes to the office due to a 2-month history of progressive exertional dyspnea. The patient is unable to walk half a block without resting and can no longer climb stairs without stopping every few steps. He also has had difficulty sleeping due to repeatedly waking up at night feeling like he is suffocating. Physical examination shows bilateral lower extremity edema and distended neck veins. Cardiac auscultation findings over the apex are given below. This patient's auscultatory findings most likely reflect which of the following?

Increased left ventricular end-systolic volume - Cardiac auscultation reveals a low-frequency sound occurring just after S2, known as S3. It can be normal in young and pregnant patients, but in those over 40 it is typically a sign of LV volume overload or failure.

A 58-year-old man is hospitalized due to sudden onset of chest pain. Blood pressure is 160/110 mm Hg and pulse is 90/min. BMI is 26.9 kg/m2. A baseline ECG shows nonspecific ST-segment and T-wave abnormalities, and serial troponin measurements are normal. The patient's fasting plasma glucose level is 160 mg/dL, although he has not been diagnosed previously with diabetes mellitus. Serum triglyceride level is elevated, and the HDL level is low. Which of the following additional findings would be most suggestive of increased insulin resistance in this patient?

Increased waist circumference - An adipose body habitus is commonly associated with insulin resistance and type 2 diabetes, especially excess visceral fat. Measuring waist circumference or waist-to-hip ratio are effective as an indirect measure of visceral fat. Increased waist circumference is associated with a higher risk of insulin resistance and metabolic syndrome.

A 15-year-old girl is brought to the hospital due to sudden onset of severe left leg pain. She was practicing for an upcoming gymnastics competition when she felt a popping sensation after landing a jump and developed anterior left leg pain. She was unable to bear weight on the leg following the injury. Physical exam shows swelling of the left knee with focal tenderness just above the patella. She is unable to raise the left leg against gravity while in the supine position. Imaging confirms a complete tear of the quadriceps tendon but no fractures are identified. Surgical repair is planned. Lidocaine injection near which of the following sites is most likely to provide adequate anesthesia for the procedure?

Inguinal ligament - The femoral nerve innervates the quadriceps muscles and the nerve emerges between the psoas and iliacus muscles and passes under the inguinal ligament. The optimal site for a femoral nerve block is right below the inguinal ligament at the lateral boarder of the femoral artery.

A 23-year-old man comes to the emergency department with fever, severe headache, and vomiting. He has not felt well for the past 2 weeks due to fatigue, intermittent low-grade fevers, and headaches. The patient is HIV positive and spent several months in prison 3 years ago. Temperature is 38.6 C (101.5 F). Neck stiffness is present on physical examination. A CT scan reveals no intracranial lesions. Cerebrospinal fluid analysis is notable for the following: Glucose 24 mg/dLProtein180 mg/dL Leukocytes120/mm3 Cerebrospinal fluid cultures grow Mycobacterium tuberculosis with significantly decreased activity of intracellular catalase peroxidase. The isolates would most likely exhibit resistance to which of the following agents?

Isoniazid - This presentation is consistent with tuberculous meningitis. Isoniazid works through the inhibition of mycolic acid synthesis by M tuberculosis. Isoniazid must be processed by mycobacterial catalase peroxidase. Through non-expression of the catalase-peroxidase enzyme or through genetic modification of the isoniazid binding site the bacteria can be resistant.

A 52-year-old man is brought to the emergency department for evaluation of fever, chills, and malaise. Cardiopulmonary examination reveals a new holosystolic heart murmur that radiates toward the axilla. Blood cultures are obtained, and the patient undergoes transesophageal echocardiography. The ultrasound probe is placed in the midesophagus facing anteriorly, and the cardiac chambers are interrogated. Which of the following chambers is closest to the probe?

Left Atrium - TEE using sound waves from within the esophagus to visualize nearby cardiac structures. The left atrium makes up the majority of the heart's posterior surface, with the esophagus passing immediately posterior to the heart, so the esophagus lies closest to the left atrium and will be best visualized, along with the atrial septum and mitral valve.

A 58-year-old man comes to the office due to progressive fatigue, dyspnea on exertion, and orthopnea. Medical history is significant for hypertension, hyperlipidemia, and a myocardial infarction 2 years ago. The patient says he has not taken his medications in several months because he was "feeling well." Blood pressure is 160/95 mm Hg and pulse is 94/min and regular. Physical examination shows bilateral lung crackles and lower extremity edema. The point of maximal impulse is displaced toward the axilla, and an S3 is heard. Serum creatinine is 0.9 mg/dL and serum potassium is 4.2 mEq/L. After this patient's condition has stabilized, long-term use of which of the following medications will most likely reduce mortality?

Lisinopril - This patient has most likely developed chronic heart failure with reduced ejection fraction (HFrEF). Pharmacologic management can reduce mortality through a reduction in deleterious cardiac remodeling. ACE inhibitors (lisinopril), angiotensin-receptor blockers (losartan), and mineralocorticoid receptor antagonists (spironolactone, epleronone) reduce mortality through reduction in angiotensin II-mediated cardiac remodeling. Beta blockers (metoprolol, carvedilol) also reduce mortality in these patients.

A 33-year-old homeless man is hospitalized due to recent weight loss, odynophagia, and progressive cough. He is found to be HIV-positive and his CD4 count is 45/mm3. The patient's condition improves on a newly instituted treatment, but 4 weeks later at outpatient follow-up he is found to have significantly elevated fasting blood glucose. Which of the following medications is most likely responsible for his hyperglycemia?

Lopinavir - This drug is a protease inhibitor. They reversibly inhibit viral protease, an enzyme responsible for HIV polyprotein cleavage to form mature viral proteins. They are never used as monotherapy as viral resistance to an entire class can form quickly. PIs adverse effects include: lipodystrophy (buffalo hump), hyperglycemia (insulin resistance), and inhibition of cytochrome p450 (drug interactions- ex. rifampin)

A 6-year-old boy is brought to the office due to frequent nighttime awakenings. Several times a week over the past month, he has come to his parents' bed around 3 AM, frequently crying, appearing frightened, and saying that he became lost in a crowd and couldn't find his parents. Although he is comforted by their reassurance, he refuses to return to his room and eventually falls back asleep in their bed. The boy is about to start first grade at a new school. He is healthy and developmental milestones are normal. Examination shows no abnormalities. The sleep stage associated with this child's frightening experience is characterized by which of the following?

Muscle paralysis - This boy's presentation is consistent with nightmares which occur in REM sleep. REM periods are more frequent in the final third of the night and is characterized by rapid eye movements, vivid dreaming, and voluntary muscle atonia due to inhibition of motor neurons.

A 10-year-old boy is brought to the clinic due to stiffness and weakness in his hands. A few weeks ago, the parents first noticed that the child was having trouble letting go of his toothbrush after brushing his teeth. The patient's teacher has also mentioned that the boy seems to have difficulty maintaining a firm grip on his pencil when doing schoolwork. He has a history of intellectual disability and takes no medications. Examination shows bilateral cataracts. Laboratory evaluation shows unstable cytosine-thymine-guanine trinucleotide repeats, resulting in the transcription and accumulation of untranslatable mutant mRNAs, which negatively affect cellular function. Which of the following cell types is likely to be most affected by this patient's condition?

Myocardial cells - This patient has myotonic dystrophy type 1 (DM1), which is an autosomal dominant muscular dystrophy characterized by unstable expansion of cytosine-thyamine-guanine (CTG) trinucleotide repeats in the DMPK gene. It is notably unstable in non-dividing cells, including skeletal muscle, the brain, and myocardium. These cells that remain senescent tend to accumulate more mutant mRNAs over time.

A 68-year-old man comes to the office due to a weak urinary stream, hesitancy, and straining on micturition. These symptoms have been present for the past 2 years but have gradually become more severe and are beginning to affect his quality of life. The patient recently started noticing continuous urine leakage, nocturia, frequent urination, and urgency. He has no other medical problems and takes no medications. The patient does not use tobacco or alcohol. He is a business management consultant and motivational speaker. Vital signs are normal. BMI is 27 kg/m2. The patient's kidneys are most likely to demonstrate which of the following findings?

Parenchymal pressure atrophy - The patient's symptoms are likely due to benign prostatic hyperplasia. As it progresses, there is worsening of bladder outlet obstruction, leading to incomplete bladder emptying and overflow incontinence. Increased hydrostatic force is needed to overcome the obstruction, which will lead to hypertrophy of the bladder wall and dilation of the ureters, renal pelvis, and calyces (hydronephrosis). If left untreated, urinary reflux can lead to significant pressure-induced parenchymal atrophy with scarring and eventual chronic kidney disease.

A 33-year-old woman, gravida 2 para 1, comes to the office due to a positive home urine pregnancy test. Her last menstrual period was 6 weeks ago, and pelvic ultrasonography confirms an intrauterine pregnancy. She has idiopathic pulmonary hypertension complicated by Eisenmenger syndrome. She is aware that pregnancy poses a major mortality risk to herself and the fetus due to the hemodynamic changes of gestation, labor and delivery, so she consents to pregnancy termination with mifepristone and misoprostol regimen. Which of the following is the MOA for mifepristone in this clinical situation?

Progesterone antagonist - It binds progesterone receptors with greater affinity than the natural hormone, which then results in apoptosis and necrosis of the uterine decidua and prevents further development of a first trimester pregnancy (promotes placental separation and uterine contractions). Misoprostol is a prostaglandin E1 analog (softens cervix and promotes uterine contraction)

A 52 year old woman comes into the office with a hand tremor for several months. It is most prominent when carrying out daily activities, such as drinking from a glass. It somewhat subsides when she drinks small amounts of alcohol. Family history shows similar symptoms in her mothers. What is the best treatment for this patient?

Propranolol - The patient presents with an essential tremor, which will often be worse with maintaining a certain position. They will also likely report improvement with alcohol consumption. It is often autosomal dominant. Propranolol is the first-line treatment for an essential tremor (nonselective beta-adrenergic antagonist)

A 40-year-old man with end-stage renal disease due to type 1 diabetes mellitus is hospitalized for initiation of hemodialysis. A tunneled dialysis catheter is inserted into the right internal jugular vein. Before he is able to undergo dialysis treatment, the patient develops bleeding around the catheter exit site that is difficult to control. He has not been treated recently with anticoagulants. Further evaluation of this patient would most likely show which of the following lab abnormalities?

Prothrombin - Normal Activated partial thromboplastin time - normal Platelet count - normal Bleeding time - prolonged This is common in patients with significant renal dysfunction due to accumulation of uremic toxins in circulation with impair platelet aggregation and adhesion, causing qualitative platelet disorder and a prolonged bleeding time.

A 43-year-old man comes to the office due to difficulty walking. The patient was seen at an outpatient clinic 2 days ago and received a right-sided deep intramuscular injection. He had no trouble walking at the time of the appointment but started having problems later that day. The patient has no history of trauma or pain at the injection site. Physical examination is negative for local erythema and swelling, and the patient has full strength against resistance on knee flexion and ankle plantar flexion bilaterally. When the patient is asked to walk across the room, his left hip drops every time he raises his left foot. Which of the following locations is the most likely site of this patient's intramuscular injection?

Superomedial quadrant of the buttock - This patient is presenting with a classic Trendelenburg gait, characterized by hip drop on the contralateral, unaffected side when the foot is raised. This gait is observed when there is injury to the superior gluteal nerve or to the gluteus medius muscle. The nerve is derived from the L4-S1 ventral rami and leaves the pelvis through the greater sciatic foramen above the left piriformis. Injections into the superomedial quadrant of the buttock have a high probability of injuring the superior gluteal nerve.

A 49-year-old man comes to the office due to 4 months of cough and weight loss. His medical history is otherwise unremarkable and he takes no medications. The patient is a farmer who lives in Mississippi. He does not use tobacco, alcohol, or illicit drugs. His temperature is 37.9 C (100.2 F). Examination shows right lobe crackles. Chest x-ray reveals a pulmonary infiltrate in the right upper lobe. Bronchoscopy with transbronchial biopsy shows granulomatous inflammation. HIV testing is negative. Which of the following is most likely causing this patient's symptoms?

Blastomyces dermatitidis - a dimorphic fungus that can cause pulmonary infection in immunocompetent people and should be considered in someone from an endemic area (states adjacent to and east of the Mississippi and Ohio river valleys). Pulmonary blastomycosis is characterized by granuloma formation. Extrapulmonary disease occurs in immunocompromised patients. Examination of tissues or respiratory fluids show round yeast with thick walls and broad-based budding. Treat with itraconazole.

A 56-year-old man comes to the office due to a neck mass, which has been enlarging over the past 2 months. The patient also reports night sweats and a weight loss of 7 kg (15.4 lb). Physical examination shows a 3-cm, rubbery, nontender cervical lymph node. Biopsy of the neck mass reveals non-Hodgkin lymphoma. The patient undergoes systemic chemotherapy with vincristine as part of the regimen. Which of the following adverse effects is most likely to occur with use of this medication?

Finger numbness and tingling - Vincristine, a vinca alkaloid, is often included in chemotherapy regimens to treat numerous malignancies. Vinca alkaloids inhibit microtubule formation by binding to beta tubulin protein and blocking polymerization, which this inhibition then prevents mitotic spindle synthesis leading to a failure in cell division. It also can impact microtubules in neuronal axons, leading to dose-limiting neurotoxicity. The disrupted axonal transport results in peripheral neuropathy.

A 3-year-old boy is brought to the office for a well-child visit. His mother reports that, apart from an ear infection last year, he is in good health. The child started preschool this year. He plays alongside other children and often copies what they are doing but does not play cooperatively with them. The patient knows his age and gender and speaks in 3-word sentences. He can ride a tricycle. The patient cannot use a spoon or fork but enjoys eating with his hands. He scribbles spontaneously but cannot copy a circle. He weighs 14.5 kg (32 lb) and is 96.5 cm (3 ft 2 in) tall. Head circumference is 50.8 cm (20 in). Which of the following developmental milestones is likely delayed in this patient?

Fine motor - 3-year olds should have the fine motor skills to copy a circle and use utensils, the other skills here are normal for a 3 year old

A 49-year-old man comes to the office due to "aching bones." He has a 2-month history of insidious-onset pain that is most pronounced in the back, pelvis, and lower extremities. The pain is dull and increases after weight-bearing activities. The patient has no prior medical conditions and takes no medications. He emigrated from Central Africa 5 years ago and works overnight shifts as a cab driver. Vital signs are within normal limits. Physical examination shows normal muscle strength in the upper and lower extremities bilaterally. A thorough laboratory evaluation establishes the diagnosis. After discussing the likely cause of his condition, the patient starts spending more time outdoors in the sun. Which of the following enzymatic steps will most likely be affected by this change in activity?

7-dehydrocholesterol --> cholecalciferol - on exposure to sunlight, 7-dehydrocholesterol (provitamin D3) in the skin absorbs the UV B rays. This causes a change and forms vitamin D3 (cholecalciferol). It will later be hydroxylated to 25-dehydroxyvitamin D in the liver and then 1,25-hydroxyvitamin D (the active form) in the kidneys. This patient is suffering from osteomalacia from vitamin D deficiency.

A 45-year-old woman comes to the office for a routine follow-up. The patient feels well overall but mentions that climbing the last flight of stairs to her 3rd-floor apartment has become harder. She says, "Sometimes I have to stop at the landing to catch my breath." Medical history includes cocaine use disorder in remission for the past 4 years; it is otherwise unremarkable. The patient does not use alcohol or illicit drugs. She has a 25-pack-year smoking history and has been counseled to quit smoking in the past. The patient says, "I've tried to quit several times. I can cut down to a few cigarettes or half a pack for a day or two, but then it's just too difficult." Which of the following is the most appropriate response by the physician regarding this patient's smoking?

"You were successful in stopping cocaine. How might your experience help you quit smoking?" - motivational interviewing (open-ended questions, give affirmations, reflect and summarize main points)

A study is conducted to evaluate the properties of a new test for diagnosing ovarian cancer. The study enrolled 200 patients who truly have ovarian cancer and 300 patients who truly do not. Study results showed that the test is 80% sensitive and 70% specific. Based on this information, which of the following is the approximate number of false negative test results found in this study?

40 Sensitivity = True positives/(true positives + false negatives) 0.80 = True positives / 200 TP = 160 False Negatives = (TP + FN) - TP 200-160 = 40

A study is conducted to assess whether quantitative measurement of heat shock protein 90 alpha in plasma can improve diagnostic accuracy and help monitor treatment response in liver cancer patients. A total of 300 individuals (100 liver cancer patients and 200 healthy controls) participate in the study. Analysis reveal that 90 of the liver cancer patients have abnormally high plasma Hsp90a level and that 12 of the healthy controls do as well. An abnormally high level indicates a positive test for liver cancer. Which of the following values best represents the specificity of this test?

94% - specificity = true negative/ (true negative + false positive) 188/(188+12) = 188/200 = 0.94

A 23-year-old man is hospitalized for fever, headache, and confusion. He does not use tobacco or illicit drugs and is monogamous with his wife. The patient's cerebrospinal fluid analysis reveals lymphocytic pleocytosis, and a PCR is positive for herpesvirus. The patient is treated with intravenous high-dose acyclovir. Fever and mental status gradually improve, but on the third day of hospitalization the patient's serum creatinine level increases to 3.4 mg/dL from a baseline of 0.9 mg/dL at admission. The observed finding could have been prevented by which of the following?

Aggressive intravenous hydration - The acute elevation of the patient's serum creatinine is likely due to acyclovir nephrotoxicity, which occurs in 5-10% of patients who receive the drug intravenously. This toxic effect can be prevented and treated with adequate hydration and reduction in rate of drug infusion.

A 45-year-old man is referred to an endocrinologist for newly diagnosed diabetes mellitus. A week ago, his primary care physician noted an elevated fasting serum glucose level. The endocrinologist discusses the different treatment options available, including oral and injectable medications. He recommends treatment with a medication that alters glucose metabolism within the liver by increasing the concentration of fructose 2,6-bisphosphate within hepatocytes. Which of the following conversions will be inhibited by high intracellular concentrations of this metabolite?

Alanine --> Glucose - Fructose 2,6 bisphosphate helps control the balance between gluconeogenesis and glycolysis through inverse regulation of phosphofructokinase-1, the main enzyme involved in glycolysis. Insulin increases PFK-2, leading to an increased F26BP levels and augmented glycolysis. High concentrations also inhibit gluconeogenesis, leading to decreased conversion of alanine and other gluconeogenic substrates to glucose.

A 19-year-old woman, gravida 1 para 0, at 29 weeks gestation comes to the emergency department due to severe right upper quadrant pain. The patient suddenly developed the pain this evening after dinner. The pain was associated with acute nausea and vomiting. She has no headaches or visual changes. The patient has no chronic medical conditions and takes no daily medications. Temperature is 36.7 C (98.1 F), blood pressure is 160/100 mm Hg, and pulse is 84/min. The abdomen is tender over the right upper quadrant, but no rebound or guarding is present. Hemoglobin is 8 g/dL. Urinalysis shows 3+ protein. Which of the following associated laboratory changes are most likely to be seen in this patient?

Alanine aminotransferase - increased Indirect bilirubin - increased Platelets - decreased A patient >20 weeks gestation with hypertension, proteinuria, and right upper quadrant pain likely has HELLP syndrome (hemolysis, elevated liver enzymes, low platelet count). It is most often diagnosed with patients with preeclampsia. It is a thrombotic microangiopathic disorder causing microangiopathic hemolytic anemia, thrombocytopenia, and hepatocellular injury.

A 22-year-old woman comes to the hospital due to a 5-day history of nausea, constipation, and severe, poorly localized abdominal pain. She also reports anxiety, difficulty concentrating, poor sleep quality, and tingling of the limbs. The patient has had several similar episodes in the past. She does not take any medications or use tobacco, alcohol, or illicit drugs. The patient has been restricting her diet to lose weight. On examination, the abdomen is soft, nontender, and nondistended. Bowel sounds are decreased. The patient receives an intravenous infusion of a heme preparation that leads to rapid resolution of her symptoms. The improvement in symptoms is most likely due to treatment-induced downregulation of which of the following enzymes?

Aminolevulinate dehydratase - This patient with neurologic symptoms and recurrent episodes of nonspecific abdominal pain likely has acute intermittent porphyria (AIP), an autosomal dominant disease characterized by porphobilinogen deaminase deficiency. Management of AIP attacks includes infusion of hemin, which downregulates hepatic aminolevulinate synthesis.

A 37-year-old woman comes to the office with right-sided abdominal discomfort that has been getting progressively worse over the past few weeks. She has not had any nausea and vomiting. The patient's family moved to the US from rural Kazakhstan when she was a child. The patient has no drug allergies. She does not use tobacco or alcohol. She has a dog at home. Temp is 98.1F. Physical exam is notable for hepatomegaly. Ultrasound reveals a large liver mass with cystic lesions. During surgery to resect the mass, the patient suffers from a procedural complication and dies. An autopsy is performed. Which of the following is the most likely cause of this patient's death?

Anaphylaxis - Echinococcus granulosus is the most common cause of hydatid cysts. Common in those from endemic regions or those in the southwestern US with sheep and dog exposure. Initial infection is often asymptomatic. The liver is often affected. Unilocular lesions are typical of E granulosus and multilocular lesions are typical of E multilocularis. Microscopic examination demonstrates an encapsulated and calcified cyst ("eggshell calcification"). Surgery and adjunctive chemotherapy (albendazole) are the treatment of choice, but cystic manipulation must be performed with caution as spilling of cyst contents can cause anaphylaxis

A 43-year-old man is being evaluated for occasional restrosternal chest pressure that develops with moderate exertion and sometimes occurs when resting. He does not use alcohol, tobacco, or illicit drugs. The patient has an extensive family history of CAD. His temp is 98F, BP is 124/72mmHg, pulse is 81/min, and respirations are 14/min. Physical exam shows no abnormalities. Coronary angiography shows mild luminal irregularities but no significant obstructive lesions. Acetylcholine infusion during the procedure results in dilation of epicardial coronary vessels. A reaction involving which of the following amino acids is most likely responsible for the observed dilation?

Arginine - Acetylcholine, bradykinin, serotonin, substance P, and shear forces mediate vasodilation in vascular endothelium by activation receptors on endothelial cells leading to an increase in cytosolic calcium levels. This causes activation of endothelial nitric oxide synthase (eNOS) which synthesizes nitric oxide from arginine, NADPH, and O2. NO then enters smooth muscle where it activates guanylate cyclase to increase formation of cGMP, which activate protein kinase G, causing reduction of cytosolic calcium levels and relaxation of vascular smooth muscle.

A 38-year-old woman comes to the office due to dissatisfaction with her weight. Although the patient exercises with a personal trainer weekly and tries to eat well-balanced meals, she loses control several times a week, frequently consuming an entire pizza or large bag of chips by herself. The patient says, "I'm not even hungry when it happens, and I feel so guilty afterward. I can't even look at myself in the mirror anymore because I'm disgusting." She smokes marijuana once a week and does not use alcohol or other recreational substances. BMI is 23.4 kg/m2. Which of the following is the most likely explanation for this patient's eating habits?

Binge eating disorder - This patient has recurrent episodes of binge eating with an associated sense of lack of control but an absence of compensatory behaviors seen in bulimia nervosa.

A 52-year-old active smoker comes to the office due to a 2-week history of malaise, shortness of breath, and decreased appetite. The patient has a history of a bicuspid aortic valve and hypertension. His temp is 102.2F, BP is 140/80mmHg, and pulse is 110/min. On exam, the patient has a systolic ejection murmur at the right upper sternal border that is unchanged from previous visits, but there is a new decrescendo diastolic murmur best heard in the third intercostal space at the left sternal border. Lab studies show hemoglobin is 9.3g/dL and leukocytes are 16,500/mm3. Blood cultures are positive for Streptococcus gallolyticus (formerly S bovis). Additional workup for this patient should focus on which of the following conditions?

Colonic neoplasia - S gallolyticus is the main human pathogen among nonenterococcal group D strep. It causes a subacute bacterial endocarditis similar to those of S viridans. It is part of the normal flora of the colon, and bacteremia or endocarditis caused by it are associated with colonic cancer in ~25% of cases.

A 34-year-old industry worker experiences rapid-onset shortness of breath, dizziness, palpitations, and flushed skin after accidental exposure to chemical fumes. Examination by an occupational physician shows that the patient is tachypneic and has a reddish discoloration of the skin without cyanosis. The patient is immediately instructed to inhale amyl nitrite from the safety kit. The success of this antidote is dependent upon its ability to convert hemoglobin into a form with increased affinity for which of the following substances?

Cyanide - The toxicity of cyanide is dependent on its ability to bind ferric iron (Fe3+) with high affinity, inhibiting cytochrome c oxidase in the mitochondria. Cyanide poisoning presents with reddish skin discoloration, tachypnea, headache, and tachycardia, often accompanied by nausea, vomiting, confusion, and weakness. Lab studies indicate severe lactic acidosis in conjugation with narrowing of the venous-arterial PO2 gradient, resulting from the inability of tissue to extract arterial nitrogen. Inhaled amyl nitrite oxidizes ferrous iron (Fe2+) in hemoglobin to ferric iron (Fe3+) generating methemoglobin, which carries oxygen but has a high affinity for cyanide.

A 50-year-old man is brought to the ER with severe dizziness and confusion. He states that he had an episode of chest pain and took several tablets of nitroglycerin. His current medications include a daily aspirin for heart attack prevention, an occasional acetaminophen for headaches and occasionally tadalafil for erectile dysfunction. His blood pressure is 50/20 mmHg and his heart rate is 120 beats/min. Which of the following cellular changes is most likely responsible for this patient's symptoms?

Cyclic GMP accumulation - The interaction between nitrates and phosphodiesterase inhibitors used in erectile dysfunction (tadalafil, sildenafil, vardenafil) is well known and very important to understand. Nitrates are converted to nitric oxide by vascular smooth muscle cells, and nitric oxide causes increased intracellular cGMP as a second-messenger. Increased cGMP concentration leads to vascular smooth muscle relaxation. Additionally, cGMP is metabolized within the cells by phosphodiesterase, and phosphodiesterase inhibitors will lead to increased intracellular cGMP. cGMP accumulation in vascular smooth muscle cells due to both enhanced synthesis (nitrates) and inhibited degradation (PDE inhibitors) is responsible for profound hypotension due to extreme vasodilatation when these drugs are used together.

A 34-year-old man comes to the physician due to fever, myalgias, malaise, and progressive fatigue over a 2-week period. He has not experienced any sore throat. He recently received a blood transfusion while hospitalized for a bleeding duodenal ulcer. Physical examination shows mild splenomegaly. There is no lymphadenopathy and no jaundice. Lymphocytosis is identified in the peripheral blood with 30% atypical lymphocytes. The patient's serum fails to agglutinate horse erythrocytes. The agglutination test is repeated a week later and, again, no agglutination is noted. HIV testing is negative. Which of the following is the most likely cause of this patient's condition?

Cytomegalovirus - Most cases of infectious mononucleosis (IM), caused by EBV, are associated with serum heterophile antibodies that agglutinate with erythrocytes from unrelated species. In immunocompetent patients with a heterophile antibody-negative mononucleosis-like syndrome, the most likely diagnosis is CMV. Lymphadenopathy and pharyngitis is less common in CMV.

A 54-year-old man comes to the office due to severe foot pain. The patient attended a wedding reception where he drank several alcoholic beverages, then woke the following morning with pain. He has a long history of gouty arthritis, and his current symptoms are similar to previous flares of the disease. Past medical history includes type 2 diabetes mellitus and recently diagnosed peptic ulcer disease. Physical examination shows erythema, warmth, and swelling at the left first metatarsophalangeal joint. The patient is started on a new medication for gout that provides significant relief of his symptoms, but he returns to the clinic a week later with diarrhea and persistent nausea. The drug used in this patient most likely affects which of the following cell structures?

Cytoskeleton - First line gout drugs are contraindicated in patients with a recent gastric ulcer. Colchicine binds to tubulin and inhibits its polymerization into microtubules. This disrupts cytoskeletal-dependent functions such as chemotaxis and phagocytosis. It can be used for acute treatment or prophylaxis. Side effects include nausea, abdominal pain, and diarrhea. Should be avoided in the elderly and in severe renal dysfunction

A 41-year-old woman, gravida 3 para 2, at 34 weeks gestation comes to the office for a routine prenatal visit. For the past 2 weeks, the patient has had increasing pain over the pubic symphysis that is unrelieved with acetaminophen or warm compresses. She has had no dysuria, hematuria, contractions, or vaginal bleeding. The patient has hypothyroidism that is well controlled with levothyroxine. Vital signs are normal. Examination shows a tender, slightly widened pubic symphysis. Sensation is intact. Which of the following hormones is the most likely underlying cause of this patient's pain?

Relaxin - This patient has a slightly widened pubic symphysis, a common but often painful adaptation of pregnancy that occurs due to increased relaxin levels. It causes increased ligament elasticity, which is particularly important in the pubic symphysis and sacroiliac joints to facilitate vaginal delivery.

A newborn boy is being evaluated in the nursery. The patient was born at 39 weeks gestation via cesarean delivery to a 30-year-old primigravida. Head circumference, weight, and length are at the 75th to 90th percentiles. The anterior fontanelle is open and soft. The neck is supple. Cardiopulmonary examination is unremarkable, and the abdomen is soft. The back appears unremarkable. Hip examination shows no hip clicks. Both feet are plantar flexed and adducted with the soles pointing medially. There is resistance to range of motion assessment in both feet. Muscle tone is normal and newborn reflexes are intact. The abnormal findings on this patient's physical examination most likely represent which of the following types of congenital anomalies?

Deformation - This newborn presents with talipes equinovarus (clubfoot), a congenital anomaly in which one or both feet are rigidly flexed downward and inward. It is a deformation anomaly caused by extrinsic forces on the developing fetus. Underdevelopment of the talus bone causes subluxation of the surrounding joints and shortening of the adjacent calf muscles and tendon

A 20-year-old woman is brought to the emergency department with intractable vomiting and abdominal pain. She has a several-week history of anorexia and weight loss. The patient has well-controlled primary hypothyroidism which is treated with levothyroxine, but she has been unable to take the medication for 2 days due to vomiting. She has a temp of 99.7F, BP of 80/40 mmHg, pulse of 120/min, and respirations of 18/min. Physical exam shows her mildly lethargic but oriented. Generalized hyperpigmentation is present, especially on the face, neck, and dorsal surface of the hands. Finger stick glucose is 60mg/dL, and urinalysis and chest x-ray are normal. Normal saline (1L) is rapidly infused and high-flow intravenous fluids are continued. The patient also requires immediate treatment with which medication?

Dexamethasone - This patient likely has acute adrenal crisis, as suggested by the weight loss and hyperpigmentation, and an increased risk of autoimmune endocrinopathies, like primary adrenal insufficiency (Addison disease) due to her hypothyroid history. Adrenal crisis is characterized by severe hypotension and shock, with symptoms such as abdominal pain, vomiting, weakness, and fever. Aggressive fluid resuscitation and glucocorticoid supplementation with hydrocortisone or dexamethasone is indicated, even prior to diagnostic study results.

A 23-year-old man is brought to the emergency department by paramedics following a motor vehicle accident. He was an unrestrained passenger in the front seat. Several days after hospitalization, his fluid volume and plasma osmolarity are measured. Osmolarity is increase in intracellular fluid and extracellular fluid, but fluid volume in both is decreased. Which of the following conditions is most likely to cause these findings?

Diabetes insipidus - This patient's recent head trauma is consistent with central diabetes insipidus due to hypothalamic/pituitary damage. The resulting decrease in vasopressin secretion leads to excessive free water excretion by the kidneys, causing hyper osmotic volume contraction. This could also be seen with dehydration or profuse sweating.

A 32-year-old man dies suddenly in his sleep. He had been experiencing easy fatigability for the past 6 months and mentioned that he was feeling tired "'all the time"' and needed to take naps during the day. He did not drink alcohol or use illicit drugs. He smoked a pack of cigarettes per day for 10 years. His father died from a "heart problem" at age 40. On autopsy, the heart appears grossly enlarged. A layered mural thrombus is seen in the left ventricular apex. Coronary atherosclerosis is present, with 20% narrowing of the mid-left anterior descending artery and 25% narrowing of the left circumflex artery. Which of the following is the most likely cause of death of this patient?

Dilated Cardiomyopathy - A primary insult to the myocardium leads to impaired myocardial contractile function with dilation of one or both ventricles. These patients are at risk of sudden cardiac death from ventricular arrhythmias. Stagnation of blood flow can lead to the possibility of developing LV mural thrombus and subsequent systemic embolization. Familial DCM is suggested in this case which is typically autosomal dominant, with truncating mutations in the TTN gene when encodes for titin.

A 14-year-old is brought to the ER due to a suicide attempt where she swallowed several of her grandfather's theophylline tablets after an intense argument with her parents. She reports nausea and headache,. She has no other prior medical issues and takes no medications. Her temp is 98.6F, BP is 114/76mmHg, pulse is 88/min, and respirations are 16/min. The patient should be carefully monitored for which of the following?

Seizures and tachyarrhythmias - In low doses, methylxanthines cause mild cortical arousal and insomnia. Acute theophylline intoxication results in nausea, vomiting, abdominal pain, diarrhea, cardiac arrhythmias, and seizures. Seizures are the major cause of morbidity and mortality, tachyarrhythmias are the other major concern but usually do not cause QT prolongation. Patients should be given activated charcoal, beta blockers are used to tachyarrhythmias, and seizures can be treated with benzodiazepines and barbituates

A 58-year-old man with type 2 diabetes mellitus comes to the emergency department with fever, malaise, lower abdominal discomfort, and left flank pain. The patient has a history of diabetic autonomic neuropathy with bladder dysfunction and has been hospitalized multiple times for recurrent urinary tract infections. His temperature is 39.4 C (103 F), blood pressure is 94/50 mm Hg, and pulse is 118/min. Height is 180 cm (5 ft 11 in), weight is 150 kg (331 lb), and BMI is 46 kg/m2. On examination, there is suprapubic and left costovertebral angle tenderness. After review of culture and sensitivity tests from his prior hospitalization, the patient is started on an empiric antibiotic regimen that includes an aminoglycoside. While calculating the appropriate dosage, the hospital pharmacy uses an adjusted body weight that is lower than the patient's actual body weight. Which of the following best explains the use of this adjusted parameter?

Distribution of the drug is limited to the extracellular fluid compartment - Obesity causes a disproportional increase in adipose mass, which can have a variable effect on the volume or distribution of a drug depending on its solubility. Aminoglycosides cannot cross cell membranes as they are large, charged molecules and their volume distribution is limited to the extracellular space, which is only marginally increased in obese individuals.

A 23-year-old woman, gravida 2 para 1, at 39 weeks gestation comes to the labor and delivery unit with regular painful contractions. The patient's first pregnancy was uncomplicated and resulted in a full-term, spontaneous vaginal delivery 3 years ago, and her current pregnancy has been uncomplicated. She takes a prenatal vitamin daily and has no chronic medical conditions. In response to rising estrogen levels, the patient's myometrial cells increase the expression of genes that encode the oxytocin receptor and connexin 43. These molecular changes most likely result in increased formation of which of the following?

Gap junctions - Communicating gap junctions are especially important during labor and delivery, which require coordination and synchronization of individual myometrial (uterine smooth muscle) cells

A 43-year-old man comes to the office with a several-month history of fatigue, rash, flushing, and abdominal cramps. His rash is worse with rubbing or scratching, and he has diffuse itching after hot showers. The patient is frequently dizzy and light-headed after prolonged standing and had an episode of syncope while working in the hot sun. He used to be healthy and physically active but has had to reduce his normal activity. The patient does not use tobacco, alcohol, or illicit drugs. Vital signs are normal. Skin examination shows a maculopapular rash. Skin biopsy shows large clusters of mast cells that are positive for KIT (CD 117). Which of the following additional findings are most likely present in this patient?

Gastric hyper-secretion - Gastric acid secretion by parietal cells in the fundus and body of the stomach is stimulated by histamine, acetylcholine, and gastrin. In systemic mastocytosis, clonal mast cell proliferation occurs in the bone marrow, skin, and other organs. Mast cell proliferation is often associated with mutation in the KIT receptor tyrosine kinase. These cells are characterized by prominent expression of mast cell tryptase. Excessive histamine release from degranulation of mast cells mediated many of the symptoms, including syncope, flushing, hypotension, pruritus, urticaria, and gastric acid secretion.

A 32-year-old woman comes to the office due to several days of fatigue and malaise. She has no significant medical history and has had no recent sick contacts. The patient does not appear to be in acute distress. Evaluation reveals severe thrombocytopenia. Thrombotic thrombocytopenic purpura is suspected. Which of the following findings is required to make the diagnosis?

Hemolytic anemia - Thrombotic thrombocytopenic purpura (TTP) is a thombotic microangiopathy characterized by the formation of platelet rich thrombi in the microvasculature. It is caused by severe deficiency of ADAMTS13, an enzyme that cleaves ultra large von Willebrand factor multimers on the endothelial surface. Loss of this enzyme triggers spontaneous platelet aggregation and activation, which results in thrombocytopenia due to platelet consumption and microangiopathic hemolytic anemia (MAHA) due to mechanical shearing of passing red blood cells. Schistocytes will be seen on blood smear. Plasma exchange is the treatment.

A 9-year-old girl is brought to the emergency department following a 3-min generalized seizure. She has had a fever with a worsening headache for 3 days. The girl has no medical problems and is not taking any medications. She lives with her parents and pet cat and has traveled to Canada and England. On physical exam, she is disoriented and having difficulty staying awake. Her speech is difficult to understand. She is admitted to the ICU for further workup. The patient becomes unresponsive the following day and dies overnight. Autopsy reveals bilateral, hemorrhagic necrosis of the inferior and medial temporal lobes. This patient's condition was most likely caused by infection with what?

Herpes simplex virus - HSV-1 encephalitis is the most common cause of fatal sporadic encephalitis. It results from primary oropharyngeal infection that travels via the olfactory tract or from the reactivation of latent virus in the trigeminal ganglion with subsequent spread to the cerebral vault. CT, MRI, and post-mortem macroscopic brain exam reveal edema and hemorrhagic necrosis of the temporal lobe.

A 5-month-old boy is brought to the office for right-sided scrotal enlargement. The enlargement has been present since birth and increases when he cries or strains to pass a bowel movement. There is no history of trauma or infection. The boy's parents do not believe he is in pain and have not noted discoloration of the area. On ultrasonography, the enlargement is found to be a fluid collection around the right testis. The specific embryologic defect giving rise to this patient's condition can also lead directly to which of the following?

Indirect inguinal hernia - failure of obliteration of the process vaginalis leads to a persistent connection between the scrotum and peritoneal cavity through the inguinal canal. If the opening is small and allows for fluid leakage only, a communicating hydrocele develops. If the opening were large enough for passage of abdominal organs, an indirect inguinal hernia develops.

A large prospective study evaluates the relationship between alcohol consumption and breast cancer. A total of 4000 middle-aged women are enrolled in the study through a random selection of residential addresses. Daily alcohol consumption and breast cancer incidence are assessed through the use of periodic questionnaires. On five-year follow-up, the investigators report the association between alcohol consumption and breast cancer as a relative risk of 1.32 (95% confidence interval = 0.90-1.85); 800 subjects were lost to follow-up by the end of the study, the majority of whom were moderate to heavy alcohol consumers. According to this information, which of the following biases is most likely to be present and may have affected the results?

Selection bias - If loss to follow-up occurs disproportionately between the expose and unexposed group, attrition bias can result. Attrition bias is a form of selection bias.

A 44-year-old woman, gravida 2 para 2, is being evaluated after having several episodes of painless postcoital bleeding. She has not received any preventative health care in many years. Her menstrual periods have been regular, and she uses barrier contraceptives. Speculum examination reveals an ulcerative lesion at the external cervical os. Cervical biopsy shows malignant squamous cells invading the underlying stroma. Further analysis of the malignant cells reveals that a double-stranded DNA virus, which encodes several proteins including E6 and E7, has integrated into the host genome. Which of the following is the most likely mechanism by which these viral proteins are involved in this patient's condition?

Inhibition of cell cycle regulatory proteins - This patient has squamous cell carcinoma of the cervix from an HPV infection. The most high risk subtypes are 16 and 18. The oncogenic capability of HPV is dependent on its ability to integrate into the host genome and subsequently produce viral proteins E6 and E7, which interact with cell cycle regulatory proteins p53 and Rb, respectively. The collective effects of E6 and E7 lead to inhibition of cell cycle regulation and evasion of apoptosis, consequently increasing malignant potential

A 44-year-old man is evaluated due to progressive dyspnea over the past several years. The patient has no associated chest pain or palpitations. Physical examination shows a prolonged expiratory phase without wheezes or rhonchi. CT scan of the chest demonstrates bilateral lower lobe-predominant emphysema. Further testing reveals that the patient has a protease inhibitor deficiency, which has led to increased elastin fiber breakdown. Elastin fibers within alveolar walls normally allow the lung to stretch during active inspiration and recoil during passive expiration. Which of the following most likely contributes to this property of elastin?

Interchain cross-links involving lysine - This patient's symptoms are likely due to alpha-1 antitrypsin deficiency. This protein typically inhibits elastase to prevent alveolar wall degradation. Patients with this deficiency consequently develop excessive alveolar elastin degradation, which clinically manifests with early-onset, lower lobe predominant emphysema.

A 5-year-old boy with severe, recurrent respiratory infections is undergoing evaluation. Sputum studies reveal intracellular bacteria. Further testing shows that the patient's T cells lack the IL-12 receptor. Supplementation with which of the following substances would most likely improve this patient's condition?

Interferon-gamma - Helper T cells that have not yet contacted antigens are called naive (Th0) cells and have T-cell receptors (TCRs) and CD4 proteins on their surface. An antigen-MHC class II complex on an antigen-presenting cell (eg, macrophage, dendritic cell) can activate a naive Th cell by interacting with both TCR and CD4. The activated T cell then differentiates into either a Th1 or Th2 cell. If the antigen is presented by a macrophage, the macrophage will produce IL-12, which stimulates differentiation into Th1 cells. A deficiency of IL-12 receptors on naive T cells prevents differentiation into Th1 cells, which produce interferon-gamma (IFN-γ). IFN-γ is responsible for activation of the macrophage and CD8+ cytotoxic response against intracellular organisms, such as mycobacteria. Therefore, individuals with an IL-12 receptor deficiency are susceptible to severe, persistent mycobacterial infections. Administration of IFN-γ improves the immune response to mycobacterium in these patients.

A 58-year-old man is being evaluated for constipation and weight loss over the last several months. During the past 2 weeks, he has had several episodes of rectal bleeding. The patient has never had a screening colonoscopy. Examination shows a soft and nontender abdomen without masses or organomegaly. Digital rectal examination reveals a firm palpable mass in the rectal vault. Flexible rectosigmoidoscopy shows a large ulcerative mass in the middle third of the rectum extending to the rectosigmoid junction. He undergoes surgical resection of the rectosigmoid colon. Frozen section analysis reveals clear margins of the colon specimen with metastases in the pararectal lymph nodes. Which of the following lymph node groups should be sampled in this patient for further assessment of metastatic disease?

Internal Iliac - Lymphatic drainage of the colon generally follows arterial blood supply. The sentinel lymph nodes (first 1-4 lymph nodes draining a specific colonic segment) are usually the first site for colon cancer metastasis. Lymphatic channels proximal to the anal dentate line drain into the inferior mesenteric and internal iliac lymph nodes. Areas distal to the anal dentate line drain primarily into the inguinal nodes, although some lymph can also reach the inferior mesenteric and internal iliac nodes.

A 70-year-old man comes to the emergency department due to severe midback pain that started several hours ago. He describes the pain as excruciating and wants immediate relief. The patient also has nausea, diaphoresis, and lightheadedness. He has a history of hypertension and chronic kidney disease. His medication compliance has been poor. He is an active smoker with a 40-pack-year history. His blood pressure on the right arm is 220/105 mm Hg. ECG shows sinus tachycardia and voltage criteria for left ventricular hypertrophy with secondary ST-segment and T wave changes. After initial evaluation, a transesophageal echocardiogram shows a dissection flap in the descending aorta but no evidence of dissection in the ascending aorta. The dissection flap in this patient most likely originates near which of the following points?

Left subclavian artery - this finding is consistent with Stanford type B aortic dissection, which includes the descending aorta only. These dissections typically originate close to the origin of the left subclavian artery which type A typically originates in the sinotubular junction.

A 63-year-old man is brought to the emergency department due to high fever, confusion, headache, watery diarrhea, and cough. The patient smokes 2 packs of cigarettes daily and has a history of chronic bronchitis. Temperature is 40.1 C (104.2 F), blood pressure is 100/70 mm Hg, pulse is 91/min, and respirations are 28/min. Sputum Gram staining reveals numerous neutrophils but no bacteria. Which of the following is the most likely cause of this patient's current condition?

Legionella pneumophila - Legionella is a common cause of community-aquired pneumonia. Most cases present with high fever and fatigue followed by gastrointestinal, pulmonary, and nonspecific symptoms. Patients who smoke and have chronic lung disease are at greatest risk. Breakouts can occur when it contaminates manmade water reservoir. It is facultative intracellular and requires specialized media (buffered charcoal yeast extract) and stains poorly with gram stain due to its unique lipopolysaccharide in its outer membrane. It is usually diagnosed with PCR or a positive urine test.

An autopsy is performed on an 8-month-old infant after his unexpected death. Prior to death, the patient had a 1-month history of poor feeding and intermittent fever, which progressed to severe lethargy and coma. The infant was adopted soon after birth, and there is limited information about his family history beyond documentation of an uncomplicated pregnancy and delivery at term gestation. Brain biopsy reveals leptomeningeal inflammation and is positive for Enterovirus by PCR. This patient would have been most likely to have which of the following laboratory findings?

Low circulating B lymphocyte count - Enterovirus is a common infection in children, however because neutralizing antibodies are required to clear it, severe life threatening infections can occur in infants with a primary humoral immunodeficiency such as X-linked agammaglobulinemia (XLA). Due to a mutation in the BTK gene, patients pre-B cells cannot differentiate and exit the bone marrow, so the patients have low circulating B lymphocytes and low immunoglobulin levels. These patients typically are at risk for recurrent sinopulmonary infections and gastrointestinal infections.

On physical examination, jugular venous pressure is elevated, breath sounds are decreased at the right lung base with dullness to percussion, and 2+ bilateral lower extremity pitting edema is present. Chest x-ray shows cephalization of the blood vessels, Kerley B lines, and a right pleural effusion. Thoracentesis is performed for pleural fluid analysis. In comparison to plasma fluid, which of the following pleural fluid findings is most likely to be observed?

Low lactate dehydrogenase content - When determining etiology of a pleural effusion, light criteria is used to see if it is transudative or exudative via analysis of the protein and lactate dehydrogenase levels of the fluid. Heart failure causes a transudative pleural effusion which is characterized by low protein and low lactate dehydrogenase levels compared to serum values.

A 68-year-old woman is brought to the emergency department with severe right hip pain after a fall. She tripped on a rug and fell against a wall and then to the floor, landing on her right hip. The patient has a history of depression and polymyalgia rheumatica. Medications include sertraline and low-dose prednisone. She also has a 40-pack-year smoking history. Blood pressure is 145/85 mm Hg and pulse is 96/min. The patient appears to be in significant pain. She is unable to move the right hip, and the right leg appears shorter than the left. Injury involving which of the following arteries is most likely to lead to osteonecrosis in this patient?

Medial circumflex femoral - The x-ray shows a right femoral neck fracture, which is common in elderly patients with osteoporosis who have fallen. The femoral head is at risk for osteonecrosis if the blood supply is disrupted. The blood supply derives mainly from the ascending cervical and retinacular branches of the medial circumflex femoral artery, which are at risk with this fracture.

A mutation in a non-coding DNA sequence is believed to affect expression of the gene coding for a specific fetal enzyme. Liver and bone marrow cells from the fetus and his parents are obtained. Which of the following is the best method to determine if this gene is being transcribed in cultures of the isolated cells?

Northern blot - The best method for determining whether a gene is undergoing transcription is to analyze for the presence of its mRNA using a northern blot. DNA for southern blots, mRNA for northern blots, protein for western blots, and DNA-bound protein for southwestern blots.

A 34-year-old man comes to the emergency department due to recurrent fever, chills, and excessive sweating. The symptoms began a few days ago and seem to recur every 48 hours. The patient recently returned from a trip to Latin America. Temperature is 38.6 C (101.5 F). Physical examination is otherwise normal. Laboratory studies are notable for anemia and thrombocytopenia. A blood smear with Giemsa staining demonstrates red blood cell inclusions. Chloroquine and primaquine are prescribed. The addition of primaquine to the treatment regimen is most likely to have which of the following effects?

Prevent disease relapse - This patient is being treated for malaria. Plasmodium falciparum is most common in sub-Saharan Africa, whereas in South and Central America it is largely caused by Plasmodium vivax. P vivaz forms hyponozoites in the liver that are dormant and reactivate weeks or months later, therefore they need medication that targets the intraerythrocytic phase (chloroquine) as well as the dormant hepatic phase (primaquine) to prevent relapse

A 45-year-old woman comes to the office due to colicky abdominal pain, diarrhea, and nausea 15-20 minutes after meals. These episodes are accompanied by light-headedness and diaphoresis. She feels normal between the episodes and at night. The patient underwent Roux-en-Y gastric bypass surgery 4 weeks ago for obesity. She is otherwise healthy, and her only medication is a daily multivitamin. On physical examination, the patient has a well-healed midline abdominal scar. Mild epigastric tenderness to palpation is present. The remainder of the examination is normal. Which of the following is the most likely cause of this patient's symptoms?

Rapid transition of hyperosmolar food into the small bowel - These symptoms are consistent with dumping syndrome, a common complication of gastric surgery. When the pyloric sphincter is removed or circumented, as in Roux-en-Y gastric bypass, incompletely digested food particles enter the small intestine rapidly. The undigested material is often hyperosmolar and causes fluid movement from the intravascular space into the intestinal lumen. Dumping syndrome is characterized by the following symptoms 10-30 minutes after eating - abdominal cramps, diarrhea, and nausea (excessive intestinal fluid); diaphoresis, palpitations, and flushing (vasomotor hyperactivity); hypotension and tachycardia (intravascular volume depletion)

A 63-year-old woman is hospitalized because of dyspnea on exertion and generalized fatigue. She has been hospitalized 3 times during the past year for heart failure exacerbations. Her other medical problems include essential hypertension for 20 years, MI at the age of 58, and hypercholesterolemia. She has a 36-pack-year smoking history in addition to a 10-year history of alcohol abuse. Physical examination reveals tachycardia, distended neck veins, bilateral crackles on lung auscultation, a third heart sound on cardiac auscultation and pedal edema. The appropriate therapy is initiated. On the third day of hospitalization, her hematocrit level increases to 50% from 44% on admission. An arterial blood gas analysis shows an arterial O2 partial pressure (PaO2) of 70mmHg. A51Cr-tagged red blood cell infusion indicated normal red blood cell mass. Which of the following is the most likely cause of this patient's polycythemia?

Relative erythrocytosis - Polycythemia (erythrocytosis) is defined as a hematocrit above 52% in med and above 48% in women. Absolute erythrocytosis is an absolute, true increase in RBC mass, whereas relative erythrocytosis is a normal RBC mass, but a decrease in plasma volume, often from aggressive diuretic therapy for heart failure.

A 54-year-old woman is evaluated in the clinic for exertional dyspnea and easy fatiguability. The patient has no chest pain, cough, or wheezing. She does not use tobacco, alcohol, or recreational drugs. On physical exam, her gait is unstable when her eyes are closed and there is impaired vibratory sensation in the lower extremities. Marked pallor of the conjunctivae, nail beds, and palms is present. Which lab test would help confirm the most likely diagnosis in this patient?

Serum methylmalonic acid level - This patient presents with anemia and deficits highly suggestive of a vitamin B12 deficiency Since B12 serves as a cofactor for methylmalonyl-CoA mutase and methionine synthase, a B12 deficiency will show elevated levels of methylmalonic acid and homocysteine. Increases in methylmalonic acid levels can disrupt myelin synthesis and result in subacute combined degeneration of the dorsal columns and lateral corticospinal tract. Axonal degeneration of peripheral nerves can be seen as well.

A 54-year-old man comes to the physician with recent-onset mild headaches and left leg weakness. Physical exam shows many lesions on the patients back. Which of the following inheritance patterns is this patient's disorder most likely to exhibit?

Single-gene autosomal dominant - Neurofibromatosis type 1 (NF-1) autosomal dominant condition is caused by a mutation in the NF1 gene, located on chromosome 17. NF-1 has high penetrance (most patients who inherit the gene develop the disease), but the presentation is highly variable. All or none of the following symptoms may be present in an individual who suffers from NF-1: Skin: Café-au-lait spots are hyperpigmented lesions with either smooth or irregular borders. The presence of axillary or inguinal freckles is another cutaneous feature of NF-1. Neurofibromas: Short, sessile, or pedunculated lesions that vary in size. They are commonly multiple and distributed throughout the body. Eye: Optic nerve gliomas may occur and cause visual loss. Lisch nodules are pigmented, asymptomatic hamartomas of the iris. Bony abnormalities: These include sphenoid dysplasia, congenital pseudoarthrosis, and scoliosis. Other associated tumors: Meningiomas, astrocytomas, gliomas, and pheochromocytomas. This patient's headaches and left leg weakness are probably the result of a brain tumor.

A 10-year-old boy is brought to the clinic due to stiffness and weakness in his hands. A few weeks ago, the parents first noticed that the child was having trouble letting go of his toothbrush after brushing his teeth. The patient's teacher has also mentioned that the boy seems to have difficulty maintaining a firm grip on his pencil when doing schoolwork. He has a history of intellectual disability and takes no medications. Examination shows bilateral cataracts. Laboratory evaluation shows unstable cytosine-thymine-guanine trinucleotide repeats, resulting in the transcription and accumulation of untranslatable mutant mRNAs, which negatively affect cellular function. Which of the following cell types is likely to be most affected by this patient's condition?

Troponin - The contractile mechanism in skeletal muscle depends on proteins (myosin II, actin, tropomyosin, and troponin) as well as calcium ions. The thick filaments in skeletal muscle are comprised of myosin molecules, with the heads of the myosin molecules forming cross-links with actin during muscular contraction. Two actin chains comprise the thin filaments in skeletal muscle. Tropomyosin molecules sit in the groove between the two actin chains, covering the myosin binding sites on actin when the muscle is at rest. Troponin molecules are small globular proteins situated alongside the tropomyosin molecules. Troponin is composed of three subunits: troponin T, troponin I, and troponin C. Troponin T binds the other troponin components to tropomyosin, troponin I binds the troponin-tropomyosin complex to actin, and troponin C contains the binding sites for Ca2+. During excitation-contraction coupling, Ca2+ is released from the sarcoplasmic reticulum. When Ca2+ binds troponin C, tropomyosin shifts to expose the actin binding sites for myosin, allowing contraction to occur.

A 35-year-old woman, gravida 1, para 0, at 12 weeks gestation comes to the office for evaluation of genital warts. She first noticed the warts last week and is concerned that it has persisted. The patient has occasional mild pruritus but no pain. She is currently sexually active with her boyfriend and engages in oral and vaginal intercourse. Examination shows several verrucous, skin-colored lesions over the labia majora. The patient asks whether the lesion could affect her future child. The virus involved in this patient's condition also has an affinity to infect which of the following structures?

True vocal cords - This patient has a presentation consistent with condylomata acuminatum, caused by human papillomavirus (HPV), a double-stranded DNA virus. Types 6 and 11 can result in warts. It infects basal epithelial cells and has a predilection for stratified squamous epithelium, which is found in the anal canal, vagina, cervix, and in the respiratory tract can be found in the true vocal cords. Infants can acquire HPV during birth and can get these growths on the true vocal cords, leading to a weak cry, hoarseness, and stridor.

A 48-year-old woman comes to the office due to a 6-month history of involuntary passage of urine when sneezing and coughing. She has experienced urine leakage even with normal daily activities, causing her to limit certain activities. She has no weakness, numbness, or fecal incontinence. She has hypertension and type 2 diabetes mellitus. She has had 4 vaginal deliveries. Supine BP is 126/82mmHg and upright is 120/80mmHg. Pelvic exam is normal. A small about of urine leaks from the urethra when the patient is asked to cough. Neurologic exam is normal. What is the most likely cause of this condition?

Urethral sphincter dysfunction - The patient is experiencing stress incontinence, which is the leakage of urine with increased abdominal pressure, such as coughing. This is due to urethral sphincter dysfunction which is common in women with prior vaginal deliveries. The sphincter is controlled by the pudendal nerve.

A 26-year-old woman comes to the office due to fever, malaise, and joint pain. A week ago, the patient had pain in the small joints of her righthand, which improved spontaneously but was quickly followed by pain in the right ankle and left wrist. She has no prior medical problems and takes no medications. Temperature is 38 C (100.4 F), blood pressure is 124/82 mm Hg, and pulse is 90/min. The extremity joints are not erythematous or swollen, but tenderness is present along the tendons of the left wrist and right ankle. Several small, non-tender pustules are seen on the extremities. There is no other rash. Blood culture in a selective growth medium consisting of chocolate agar and multiple antibiotics yields the causative pathogen. Which of the following strategies would most likely have prevented this patient's current condition?

Using condoms consistently - disseminated gonococcal infection occurs when neisseria gonorrhea spreads from genitourinary tract to the bloodstream. It typically presents with the triad of polyarthralgia, tenosynovitis, and dermatitis or purulent arthritis. The diagnosis is confirmed when the organism is isolated on Thayer-Martin media. This can be prevented with consistent use of condoms.

A 72-year-old man is evaluated due to a scaly lesion behind his right ear. A diagnosis of squamous cell carcinoma is confirmed on biopsy, and the patient undergoes excision of the lesion with clear margins. The wound is left to heal by secondary intention. Three weeks later, the patient returns to the office with a 2.5-cm raised, soft, granular lesion at the excision site. Which of the following is most likely responsible for this patient's postoperative changes?

VEGF-induced tissue proliferation - The lesion has abundant capillaries and fibroblasts, which is characteristic of granulation tissue. This is a normal part of wound healing. If VEGF-induced tissue proliferation continues unchecked, the resulting hypergranulation tissue prevents would epithelialization and remodeling. These lesions would most often occur at the site of nasogastric tubes or wounds left to heal by secondary intention.

A 30-year-old man comes to the emergency department due to severe headache and neck pain following a motor vehicle collision. The patient has no blurry or double vision, dizziness, slurred speech, numbness or weakness. Neuroimaging is unremarkable except for evidence of a right transverse foramina fracture at the level of C2. The patient is placed in a cervical collar and admitted to the hospital. While undergoing further evaluation, he suddenly develops dizziness, right-sided facial numbness, and hoarseness. Temp is 98.1F, BP is 160/90mmHg, pulse is 80/min, and respirations are 12/min. Neurologic exam shows partial ptosis and miosis of the right eye, as well as nystagmus. Hearing is intact bilaterally. There is reduced sensation to pain and temperature on the right side of the face and the left side of the body. The patient is ataxic and displays past-pointing with his right hand on finger-to-nose testing. Vascular imaging will most likely reveal an acute dissection in which of the following arteries?

Vertebral - Lateral medullary (Wallenberg) syndrome is most commonly caused by occlusion of the vertebral artery or one of its branches, vertigo without hearing loss, loss of pain and temperature on ipsilateral side of the face, loss of sensation on the contralateral side of the body due to dysfunction of the spinothalamic tract, unilateral ataxia of the upper limb, hoarseness, Horner syndrome

A 2-year-old girl is brought to the office due to concerns about her development. The girl says approximately 40 words and does not string words together. She can jump, walk upstairs slowly, build a 6-block tower, and follow short commands. She feeds and helps to dress herself. The girl's mother says, "My daughter is such a happy child. She loves playing with the other kids at day care. She is affectionate and caring toward us and her dog. I don't know why she won't speak. Her sister was using sentences by age 2. Is there something wrong with her?" Which of the following is the most appropriate response by the physician?

Your child may have a language disorder and could benefit from further assessment - At age 2, a child should have a vocabulary of 50-200 words and be able to use 2-word phases. This child has an isolated language disorder and language is also the most commonly delayed milestone. The child should also have a hearing evaluation.


Kaugnay na mga set ng pag-aaral

Introductory Logic Midterm Exam (VPSA) Study Guide Units 1 - 4 (WIP)

View Set

Course 3: Module 9. Portfolio Management

View Set

Quiz 7 3rd Grade History Abeka Our American Heritage pp. 232-251

View Set

Science; Inclined Plane, Wedge, and Screw

View Set